PharmD

Clinical Case Studies with Answers

This section includes clinical case studies* with answers for the theoretical practice of Doctor of Pharmacy ( Pharm.D ) students. Also, it is equally useful for Pharm.D PB students. These Pharm.D case studies are regularly discussed in the telegram group .   You can join there and participate in active case discussions. Case studies help you revise your Pharmacotherapy syllabus and increase knowledge of disease in an innovative way apart from practical knowledge. 

You can access these clinical case studies with answers with no charge, which are contributed by various seniors from Doctor of Pharmacy stream only. In some cases practical references are also taken from certified & specialized practitioner from the respective field along with standard treatment guidelines.

*Clinical Case Studies included here are collected from various sources and are only for study purpose while maintaining the privacy of patients.

1. Pharm.D Case Studies for second year and Pharm.D PB first year 

A) cardiovascular system case studies with answers: .

  • Hypertension    :   Case Study 1      and Case Study 2  
  • Congestive Heart Failure   :  Case Study 3 and Case Study 4
  • Angina Pectoris  :   Case study 5 and Case Study 6
  • Myocardial Infarction :  Case Study 7 and Case Study 8
  • Hyperlipidaemias :   Case Study 9 and Case Study 10  
  • Electrophysiology of heart and Arrhythmia :   Case Study 11 and Case Study 12

b) Respiratory System Case Studies with answers:

  • Asthma:   Case Study 13 and Case Study 14
  • COPD : Case Study 15 and Case Study 16

c) Endocrine System Case Studies with answers:

  • Diabetes:   Case Study 17    and  Case Study – 18 
  • Hypothyroidism :  Case Study -19 and Case Study -20
  • Other Thyroid Disorders :  Case Study -21    and Case Study -22
  • Oral Contraceptive Use : Case Study- 23 and Case Study- 24
  • Hormone Replacement Therapy:   Case Study- 25    and  Case Study- 26
  • Osteoporosis: Case Study – 27 and Case Study – 28

d) Opthalmology Case Studies with answers: 

  • Glaucoma: Case Study – 29    and Case Study – 30 
  • Conjunctivitis: Case Study – 3 1 and Case Study – 32

e) General prescribing guidelines, Small Case Study with answers:

  • Pediatrics : Case Study – 33

Case Studies for Doctor of Pharmacy students :

Case study-1 (hypertension with cardiovascular comorbidities).

JR is a 58-year-old man with a medical history of elevated low-density lipoprotein levels and well-controlled chronic stable angina (experiencing <1 angina attack per month) secondary to coronary artery disease (CAD). He presents to his primary care physician for a follow-up appointment after his blood pressure (BP) was found to be 165/94 mm Hg at his annual physical exam. At today’s visit, JR’s BP is found to be 166/93 mm Hg, resulting in a diagnosis of hypertension. JR is currently on  Atorvastatin 40 mg daily  and  Metoprolol Tartrate 100 mg twice daily.  And he reports no adverse effects from either medication. He has no other medical history of note, and his resting heart rate is 65 to 70 beats per minute.

Questions :

  • What is the target goal for BP in this patient?
  • What are the main classes of anti-hypertensives that can be used in this case ?
  • Prepare a therapeutic regimen for this patient.

For Case Study-1 Answer, Click here.

Case Study-2 (Hypertension with Type-2 Diabetes Mellitus)

Mr. MK a 55-year-old man, having history of  hypertension and type-2 diabetes mellitus  for past 10 years with non-compliance to medication and poor diet control referred to the clinic for further management of poorly controlled diabetes and hypertension.

Socio-demographics:

Age :55                                   Sex : Male BMI : 35kg/m²                     Weight : 98 kg Occupation : Salesman

Family History :

Mother : Diabetic ( on dialysis). Father : Stroke ( residual left hemiparesis).

Subjective and Objective Evidence :

  • Minimal bilateral leg edema.
  • Bilateral proliferative retinopathy. Vitals –         Blood Pressure : 160/90 mm/Hg                      Pulse rate. : 88 /min

Investigation results :

A1c                                       :      9.2% FBS.                                     :      11.8 mmol/L Serum creatinine               :      1.2 mg/dl eGFR                                    :      88 ml/min/1.73m² 24 hr urinary protein         :      200mg/24 hr. ECG                                      :       Left Ventricular Hypertrophy.

Past medication :

Metformin          1000 mg   BD Gliclazide           160 mg      BD Amlodipine        10 mg        OD

  • What might be reasons for his poorly controlled diabetes and hypertension?
  • What would be the A1c and BP target?
  • How would you manage both HTN and T2DM ?
  • Is there any Drug interaction, that should be taken in consider?
  • What are the major patient counselling points?

For Case Study-2 Answer,  Click here

For more Pharm.D Case studies. Click here.

Case Study-3 ( Congestive Cardiac Failure)

Mrs. JE a 70-year-old woman, was admitted to the medical unit with complaints of increasing dyspnea on exertion.

Subjective data:

  • Had a severe MI at 58 years of age
  • Has experienced increasing dyspnea on exertion during the last 2 years
  • Recently had a respiratory tract infection, frequently cough and edema in legs 2 weeks ago
  • Shortness of breathe while having average walking
  • Has to sleep with head elevated on 3 pillows
  • Does not always remember to take medication

Objective Evidence :

  • In respiratory distress, use of accessory muscles.
  • Heart murmur.
  • Moist cracle in both lungs.
  • Cyanotic lips and extremeties.
  • Skin cool and diaphoretic. Vitals –  Blood Pressure : 130/80 mm/Hg                Pulse rate. : 70 /min                Respiratory rate : 36 / min

Chest X-Ray : Cardiomegaly with right and left ventricular hypertrophy, fluid in lower lung fields.

Current Medication/ Collaborative Care

Digoxin                               0.25 mg            PO            qd Furosemide                        40 mg               IV              bid Potassium                         40 mEq             PO            bid Enalapril                             5 mg                 PO            qd Sodium diet                       2 gm Oxygen                               6 L/min

  • What is the significance of the findings of the chest X- Ray?
  • Are there any collaborative problems that has been not considered?
  • Is Mrs. JE provided with appropriate drug regimen? Justify.

For  Case Study-3 Answer, Click here .

For more Pharm.D Case studies. Click here.

Case Study-4 ( Case of Congestive Heart Failure in Pediatrics patient)

A 6 weeks old female presented to the emergency room with the chief complaints of lethargy, poor feeding, and respiratory distress. Her parents reported that she sweats a lot on her forehead when feeding. Her parents have also noted her to be increasingly lethargic, with tachypnea, and retractions.

  • She is the product of a G3P2, full term, uncomplicated pregnancy. Delivery was unremarkable except for meconium stained fluid. Her pediatric follow-up has been poor.
  • Developed a febrile illness with cough, rhinorrhea, and emesis prior 2 weeks
  • Subsequently developed progressive respiratory distress.

Objective Evidence:

  • Acyanotic, Lethargic, tachypneic, mildly cachectic
  • Mild to moderate subcoastal & intercoastal Retractions

                                    Heart rate                : 160 / min  

                                    Respiratory rate     : 72 / min   

                                    Temp.                       : 98.24 ᵒF

HEENT exam : unremarkable

  • Neck is supple without lymphadenopathy.
  • Skin is clear with no rashes or other significant skin lesions.
  • Lungs have scattered crackles with slightly decreased aeration in the left lower lobe.
  • Precordium is mildly active.
  • Heart is of regular rate and rhythm, with a Grade II/VI holosystolic murmur at the mid lower left sternal border with radiation to the cardiac apex.
  • S1 is normal and the S2 is prominent. An S4 gallop is noted at the cardiac apex. There are no rubs or valve clicks.
  • Her abdomen is soft, non-distended, and non-tender.
  • The liver edge is palpable 3 to 4 cm below the right costal margin. Bowel sounds are hypoactive.
  • Capillary refill is 4 to 5 seconds (delayed).
  • Chest x-ray:  Moderate cardiomegaly with a moderate degree of pulmonary edema.  No pleural effusions.
  • 12 lead ECG:    Sinus tachycardia, normal PR and QTc intervals, and a left axis deviation. Voltage evidence of biventricular hypertrophy is present. No significant Q-waves or ST segment changes are noted.

A large peri-membranous ventricular septal defect with non-restrictive left to right shunting. All cardiac chambers are dilated. Left ventricular contractility is at the lower range of normal. There is no pericardial effusion.

  • Suggest the best therapeutic regimen to control/overcome the current situation.
  • Should she be referred for surgical correction of Ventricular Septal Defect after the drug regimen has been started?

Go to  Case Study-4 Answers  |  Other  Case Studies

Case Study-5 ( Case of Angina Pectoris)

A 62-year-old male smoker with Type-2 Diabetes Mellitus and Hypertension presents with a 4-month history of exertional chest pain. 

Physical examination shows a  blood pressure  of 152/90 mm Hg, but is otherwise unremarkable. 

The  ECG  is normal, and laboratory tests show a  fasting blood glucose  value of 110 mg/dL,  glycosylated hemoglobin  6.0%,  creatinine  1.1 mg/dL,  total cholesterol  160,  LDL  120,  HDL  38, and  triglycerides  147 mg/dL. 

He exercises for 8 minutes, experiences chest pain, and is found to have a 2-mm  ST-segment depression  in the inferolateral leads at the end of exercise. 

The patient is diagnosed with  chronic stable angina .

  • What is the treatment goal and strategy for this case?
  • Suggest the best follow-up for this case.
  • What are the conditions which worsens the symptoms of angina (in general)?

Case Study-6 (Case of Angina Pectoris)

A Mr SW is a 48-year-old man going through a stressful time at work, who for the past 6 months, has been increasingly short of breath while walking to the bus. He has put this down to his ‘unhealthy ‘ lifestyle. Although he has cut down from two packets to one packet of cigarettes a week, reduced his alcohol intake from about 40 units to 25 units a week and is trying to lose weight (currently 1.8 m tall, weighing 100 kg). He sometimes finds himself short of breath with mild chest tightness, especially when he is running late. He has a strong family history of cardiovascular disease with his father having a stroke in his early 50s and his older brother having had a CABG (coronary artery bypass graft) 2 years ago. Both have encouraged Mr. SW to see his general physician due to his worsening symptoms.

Mr. SW was seen by a physician 2 years ago who prescribed  aspirin  75 mg daily,  atenolol  50 mg daily and a  GTN spray  to be used if he experiences chest pain. Since this time, he has stopped taking the aspirin, because he feels that he does not need it and he has stopped the atenolol for more than a year because he was feeling tired and read that it can cause impotence. He has used the GTN once but, after experiencing a headache and facial flushing, he has not used it since and he does not carry it with him.

After his current physician visit, he is prescribed with:

Aspirin                                              75 mg                                    daily

Simvastatin                                     40 mg                                    at night

Amlodipine                                      5 mg                                      daily

GTN spray                                                                                      when required

  • What information and counselling points would you include?
  • How is stable angina managed?
  • What options are there, if Mr. SW experiences further symptoms despite the use of amlodipine?

 Go For  Case Study-6 Answer  | Go back to  Case Studies | Go to  Home

Case Study-7 (Myocardial Infarction)

A 50 year-old male ( Height -154 cm , weight – 70 kg ) who was auto driver visited to the clinic with chief complaints of chest pain. The patient was apparently alright till 3 h back, when he suddenly felt a vague chest pain present at the center of the chest. Pain was located in the substernal location and was radiating to the right side of the shoulder. The quality of the pain was dull aching, which was increasing in severity rapidly over few hours. Chest pain was aggravated by exertion. Pain was not relieved even during rest. There was profuse sweating associated with the chest pain, and also a sense of doom or impending death. There were mild dyspnea and palpitation associated with the chest pain. There was no history of pedal edema, abdominal distension and facial puffiness, loss of appetite/fullness, right hypochondriac pain, or increased neck pulsations. Chest pain  was  diffuse in nature and  not  localized. There is no relation of the chest pain to food intake. There were no associated vomiting and hematemesis. There was no history of trauma and no history of any psychological disorders in the past. Review of other systems was normal.

Past Medical History:

The patient is not a known diabetic, or hypertensive. He has had no similar history in the past.

Past Medication History:

The patient is not on any medications. There was no history of any intake of any cardiotoxic drugs (cancer chemotherapy or prolonged steroids).

Personal History:

The patient takes mixed diet and smokes 1 packet cigarette/day for last 15 years. There was no alleged history of any alcohol or illicit drug abuse.

Pulse rate                             : 120/min      with    regular rhythm

Blood pressure                    : Left hand – 138/94 mm Hg

                                                 Left leg   -. 144/90 mm Hg.

Lab Findings:

Hb                                           : 15 g/dL

WBC                                       : 10,000 u/mcL

Creatinine clearance        : 90 ml/min

ECG Finding :

There is ST elevation >2 mm in v2–v6, and >1 mm ST-elevation in lead 1 and aVL with some minimal reciprocal changes seen in lead 3 suggestive of anterior wall + lateral wall MI due to complete left anterior descending (LAD) occlusion. Likely in the proximal LAD.

Diagnosis : Acute Myocardial infarction

Treatment :

T. Aspirin                               75 mg            OD

T. Ticagrelor                        90 mg           BD (after initial loading dose of 180 mg)

T. Atorvastatin                    80 mg            OD HS

T. Metoprolol                      50 mg            OD

T. Ramipril                            2.5 mg           BD

T. Lasix + Spironolactone (20/50) mg   OD

1. What is the latest definition of STEMI?

2. What are the classical ECG criteria for diagnosing STEMI ?

3. What are the different types of MI ?

4. Justify the treatment given to this patient.

5. Determine the duration of DAPT In this patient ?

For Case Study-7 Answers | Go to Guidelines | Go to other Case Studies

Case Study-8 (Myocardial Infarction)

Mr TR, a 54-year-old man, presented to his general physician with sudden-onset epigastric pain that started the previous night and ‘felt like trapped wind’ . This radiated through his back, up to his neck and into both shoulders/arms. He was extremely flatulent.

The patient was referred for coronary angiogram/percutaneous coronary intervention (PCI), which was carried out on site the same day. It showed the presence of a thrombus in the OM (obtuse marginal coronary artery). The distal LAD (distal left anterior descending coronary artery) and the RCA (right coronary artery) were 80% and 60- 70% stenosed, respectively.

Echocardiogram: normal LV function.

Diagnosis: NSTEMI

Two drug- eluting stents were inserted into the OM. The following drugs were prescribed post-PCI:

  • Aspirin                                              75 mg                        daily
  • Clopidogrel                                     75 mg                        daily
  • Bisoprolol                                        2.5 mg                      daily
  • Ramipril                                           1.25 mg                     at night
  • Atorvastatin                                   40 mg                        at night
  • Lansoprazole                                  30 mg                        daily

Mr TR was discharged after 3 days, after an uneventful outpatient stay, with the drugs listed above and a GTN spray to be used sublingually. Arrangements were made for an exercise tolerance test (ETT) in 6 weeks to determine whether further PCI was indicated, and for Mr TR to enter the local cardiac rehabilitation programme.

  • Is the diagnosis & treatment given to Mr TR justified? Explain.
  • Is there a need of counselling?  

Go for Case Study-8 Answers | Explore more Case Studies | Go to Guidelines

Case Study-9 (Hyperlipidemia)

A 62-year-old male is referred for management of elevated cholesterol. He has history of obesity, hypertension, and hyperlipidemia. He had a non–ST-segment elevation myocardial infarction (NSTEMI) one year ago with drug-eluting stent placement in his right coronary artery. His current medications include aspirin 81 mg daily, lisinopril 20 mg daily, and metoprolol XL 50 mg daily. His physical exam is notable for a body mass index (BMI) of 32 kg/m 2  but is otherwise unremarkable. His blood pressure is 135/85 mm Hg.

A recent lipid panel shows the following:

  • Total Cholesterol: 226 mg/dL
  • Triglycerides: 154 mg/dL
  • High-Density Lipoprotein Cholesterol (HDL-C): 39 mg/dL
  • Low-Density Lipoprotein Cholesterol (LDL-C): 190 mg/dL
  • He has a normal creatinine and normal liver enzymes. His TSH and vitamin D levels are within normal limits.
  • What would be the target goal for LDL-C in this patient?
  • What is the drug of choice in this patient to treat LDL-C?
  • What would be the treatment plan, if the patient’s LDL-C goal is not reached even after initiation of statin therapy?
  • What would be choice of drug in this patient, if he is intolerant to statin therapy?

Go for Case Study-9 Answers | Explore more Case Studies | Refer Guidelines

Case Study-10 ( Hyperlipidemias)

Four months ago, a 46-year-old man was admitted to hospital with acute chest pain. A subendocardial inferior MI was diagnosed and he was treated with thrombolytics and aspirin. After discharge, he complained of angina, and coronary angiography was performed. This showed severe triple-vessel disease not suitable for stenting, and coronary artery bypass grafting was performed. He is attending a cardiac rehabilitation clinic and he has had no further angina since his surgery.

Family History:

He has a strong family history of ischaemic heart disease, with his father and two paternal uncles having died of myocardial infarctions in their 50s; his 50-year-old brother has angina. He is married with two children.

Social History:

He smokes 25 cigarettes per day and drinks at least 40 units of alcohol per week.

Medication History:

He is taking atenolol and aspirin.

Examination :

He is slightly overweight (85 kg; body mass index ) 28). He has tar-stained nails. He has bilateral corneal arcus, xanthelasmata around his eyes and xanthomata on his Achilles tendons. He has a well-healed midline sternotomy scar. His pulse is 64/min regular, blood pressure 150/84 mmHg. He has no palpable pedal pulses. His respiratory, gastrointestinal and neurological systems are normal.

Lab Investigation:

case study examples pharmacology

Urinalysis : no abnormality detected.

  • What is the metabolic abnormality present? 
  • Discuss the Patient counselling for this case?

Go to Case Study-10 Answers | Explore more Case Studies | Refer Guidelines

Case Study-11 ( Arrthymias )

(Arrhythmia) A 2 month-old male who presents to the emergency room with a chief complaint of fever, lethargy, and poor feeding for the past 36 hours. His parents began noticing increasing lethargy and tiring with feeding and increased work of breathing for about 12 hours prior to presentation.

He is the product of a G2P1, full term, uncomplicated pregnancy and spontaneous vaginal delivery. Nursery course was uneventful.

Vital Sign: Temp: 99.32 ᵒF,

        Heart Rate: 240 per min,

        Respiratory Rate: 72 per min,

        BP: 87/64mmHg, oxygen saturation 98% in room air.

He is well developed, well nourished, but pale, lethargic and tachypneic, with mild subcostal retractions. HEENT exam is normal . Neck is supple without adenopathy.

Lungs have good aeration with fine crackles and mild retractions. His heart is tachycardic with a regular rhythm. No murmur, rub, or valve clicks are heard. His abdomen is soft, non-distended, non-tender, and without masses. His liver is 2 to 3 cm below right costal margin. His feet and hands are cool.

His peripheral pulses are 1+ to 2+ (out of 4+) throughout. Capillary refill time is 3 to 4 seconds. He has no rashes or other significant lesions.

Chest x-ray shows mild cardiomegaly and mild pulmonary edema. A 12 lead electrocardiogram shows a narrow complex tachycardia (rate of 240 bpm) with no visible P-waves (rhythm strip below).

The patient is felt to be in supraventricular tachycardia and mild congestive heart failure.

A peripheral IV is started and he is given a rapid IV bolus dose of adenosine. The patient immediately becomes briefly bradycardic followed by resumption of a normal sinus rhythm at a rate of 140 beats per minute. He is admitted for overnight observation and initiation of an anti-arrhythmic medication.

A 12-lead electrocardiogram (ECG) following conversion shows no evidence of a delta-wave, so he is started on digoxin .

  • What medicine used to treat Supraventricular tachycardia is contraindicated specifically in Wolff-Parkinson-White syndrome?
  • What would be the possible differential diagnosis?

Go For Case Study-11 Answers | Explore More Case study | Go to Guidelines

Case Study-12 ( Atrial Fibrillation )

A 67-year-old man presents to the emergency department with palpitations and dyspnea which began approximately 4 hours ago. He has a history of hypertension, diabetes, and gastroesophageal reflux disease, and LVH. On further questioning, he reports drinking 1 cup of coffee daily and 1-2 beers on the weekends.

He denies binge drinking and the use of herbal or alternative medications. He quit smoking 10 years ago. His urine drug screen is negative. His body mass index is 36 and he admits to snoring and daytime sleepiness. He is at high risk for obstructive sleep apnea (OSA). His current medications include lisinopril, metformin, and omeprazole. He has no history of congestive heart failure, stroke, or transient ischemic attack (TIA).

He appears to be in mild respiratory distress.

Blood pressure is 88/60 mmHg, pulse rate is 140 bpm, respiratory rate is 24/min, and temperature is normal. Oxygen saturation is 90% on 40% oxygen by face mask. Cardiac exam reveals tachycardia with an irregularly irregular tachycardic rhythm. There are crackles in the lower lung fields.

Electrocardiogram demonstrates atrial fibrillation (AF) with rapid ventricular rate.

  • What would be the most appropriate initial management in this patient?
  • Calculate the risk of thromboembolism and risk of bleeding in this patient?

Go for Case Study-12 Answers | Explore more Case Study | Go to G uidelines

Case Study-13 ( Case study of Asthma )

A 29-year-old man with mild persistent asthma presented to an outpatient office for a follow-up visit.  He was originally referred 6 months ago by his primary care provider after having an asthma exacerbation which required treatment in an emergency room.

At his initial visit, he reported wheeze and cough 4 days a week and nocturnal symptoms three times a month.  Spirometry revealed forced vital capacity (FVC) 85% predicted, forced expiratory volume in 1 second (FEV 1 ) 75% predicted, FEV 1 /FVC 65%, and an increase in FEV 1  of 220 ml or 14% following an inhaled short-acting bronchodilator.  He was placed on a low-dose  inhaled corticosteroid  twice a day and a short-acting  inhaled beta-agonist  as needed. 

He returned 4 weeks later improved, but with continued daytime symptoms 2 days a week.  He also had symptoms of rhinitis; therefore he was referred to an allergist for evaluation. Skin testing was positive for trees, ragweed, dust mites, and cats, and he was prescribed a nasal steroid spray and nonsedating oral antihistamine.   He presents today and reports no asthma exacerbations since his last visit. 

Furthermore, during the past 4 weeks, he has not been awakened by his asthma, experienced morning breathing symptoms, missed work, had any limitations in activities due to asthma, or required the use of rescue albuterol.  He currently denies shortness of breath or wheezing.  He performs aerobic exercise 4 days a week for 45 minutes per session without symptoms, provided he premedicates with a short-acting inhaled beta-agonist.  His review of symptoms is otherwise unremarkable. 

His current medications include low-dose inhaled corticosteroid, 1 puff twice a day; steroid nasal spray, 2 puffs each nostril daily; a nonsedating antihistamine, 1 tablet daily; and inhaled beta-agonist, 2 puffs as needed.   

His  past medical history is significant for intermittent asthma diagnosed at age 13 and frequent “colds.”  He has never required hospitalization for an asthma exacerbation.  He works as a hospital microbiologist and does not smoke, drink alcohol, or use illicit drugs.  He recently moved to a pet-free apartment complex and instituted dust mite protective barriers for his bedding.  His family history is noncontributory.

Physical Exam

On physical exam, he is an age-appropriate man in no acute distress.  His height and weight are proportionate and resting oxygen saturation as measured by a pulse (SpO 2 ) is 98% on room air.  A Head and neck exam revealed mild erythema of the nasal mucosa.  A heart exam revealed normal heart tones, no murmurs, gallops or rubs, and the lungs were clear to auscultation.  Extremities were free of oedema, cyanosis, or clubbing.

Lab:  In the office, spirometry is completely normal. He states he feels great and inquires about stopping his inhalers, particularly his inhaled steroid.

1. Based on current evidence, which of the following would be the most appropriate recommendation regarding his asthma medication regimen?

A. Maintain current medication regimen; no adjustment is indicated.

B. Discontinue the inhaled corticosteroid; maintain on an Inhaled beta-agonist as needed.

C. Decrease the inhaled corticosteroid to 1 puff daily.

D. Discontinue the inhaled corticosteroid; start a leukotriene modifier at bedtime.

E. Discontinue the inhaled corticosteroid; start low-dose inhaled corticosteroid/long-acting beta-agonist, 1 inhalation at bedtime.

2. Which of the following should be done routinely with each follow-up visit?

A. Methacholine challenge testing

B. Sputum for eosinophils

C. In-office peak flow recordings

D. Review of proper inhaler technique and adherence

E. Measurement of exhaled nitric oxide (NO)

3. What findings would suggest that the patient requires a step-up in asthma medication?

A. Two or more nighttime awakenings per month due to Asthma.

B. Two or more interruptions in daytime activities per month due to asthma .

C. Peak flow readings 85% of personal best

D. Short-acting beta-agonist for rescue once a week

E. Short-acting beta-agonist 4 days/week prior to exercise

4. The patient was provided with an asthma action plan to follow at home. Which component of the asthma action plan is considered the most critical element for improving asthma outcomes?

A. A list of the patient’s controller and rescue medications

B. A list of symptoms indicative of worsening asthma

C. Medication changes based on personal-best peak flow readings

D. Instructions describing when, how, and how long to increase medications when symptomatic

E. Medication changes based on symptoms

5.  How often is spirometry testing recommended if the previous readings are normal and the patient’s asthma is well controlled?

A. Every 1 to 2 years

B. Only if asthma controller medications are changed

C. Only if symptomatic

D. Every 4 to 6 months

E. Every follow-up visit

Image result for asthma

Explore more  Case Studies  | Go to  Case Study-13  Answers  |  Guidelines

Case Study-14 ( Case Study - II of Asthma )

A 14-year-old girl with a history of asthma requiring daily inhaled corticosteroid therapy and allergies to house dust, mites, cats, grasses, and ragweed presents to the emergency department in mid-September, reporting a recent “cold” complicated by worsening shortness of breath and audible inspiratory and expiratory wheezing.

She appears frightened and refuses to lie down but is not cyanotic.

Her pulse is 120 bpm, and respirations are 32/min.

Her mother states that she has used her albuterol inhaler several times a day for the past 3 days and twice during the previous night. She took an additional two puffs on her way to the emergency department, but her mother states that “the inhaler didn’t seem to be helping so I told her not to take any more.

1. What emergency measures are indicated?

2. How should her long-term management be altered?

Case Study-14 Answer | More Case Studies | Guidelines

Case Study-15 ( Chronic Obstructive Pulmonary Disease : COPD Case Study)

Mrs. Glenda is a 60-year-old and has recently retires from her job working for a firm that manufactures fabrics. She is a thin lady who appears older than her stated age. Glenda visits her GP as she beginning to get short of breath while climbing the stairs and is struggling to walk to the end of her road. She explains to the GP that for the past six months she has become increasingly short of breath while carrying out daily activities.

Glenda takes no regular medications and has no history of drug allergies .

She says she has not had any acute changes in her breathing, but she does have a chronic cough that produces around one or two tablespoons of clear sputum daily. Her cough has not changed recently, and the colour and volume of her sputum have also remained unaltered. She says she is not suffering from chest pains or wheezing and has not been coughing up blood.

She has smoked at least ten cigarettes a day since she was 20 but has recently cut down to five a day because of her shortness of breath. In the last few years, she has had at least two chest infections each year requiring treatment with antibiotics.

The GP suspects COPD and conducts spirometry testing, the results of which are:

FEV1 1.2 L
FEV1 predicted 2.2 L
FEV1 % 55%
FVC 1.79 L
FEV1/FVC 0.67

1. What clinical features and risk factors of COPD does glenda exhibit?  What grade of severity does glenda’s COPD fall into?

2. What initial treatment would you recommend for glenda?

3. Glenda continues to report that her breathlessness is getting worse. Her medical research council dyspnea score is now four and in the last few days she has been producing more sputum than usual. Her sputum has turned a yellow green colour. What does these changes indicate & what treatment would you recommend?

Explore more Case Studies | Guidelines | Case Answers of this COPD Case Study-15

Case Study-16 ( Case of COPD with comorbidity )

A 63-year-old woman, 67 kg, is admitted to hospital with chest pain, shortness of breath and sweating. She is seen in casualty and treated using a salbutamol nebuliser. She looks obese. She has been a life-long smoker who stopped one day ago.

Her previous medical history includes chronic obstructive pulmonary disease (COPD) for 10 years, the last admission to hospital was two weeks ago; ischaemic heart disease since 1995, myocardial infarction 4 years ago; osteoporosis diagnosed 3 years ago; hypertension diagnosed 9 years ago; and pulmonary embolism two months ago.

On examination:

Blood pressure                    105/90 mmHg

Heart rate                             90 bpm

Respiratory rate                 20 breaths per minute.

Arterial blood gases on admission:

pH                                           7.388 on 35% O2

PCO2                                      9.67 kPa

PO2                                        6.5 kPa.

Oxygen saturation: SpO2 89%. Lungs were hyperinflated, no wheeze, few right base crepitations.

Laboratory tests at admission  :

WCC                 16.5 × 109/L                    (4–11 × 109/L)

Na+                   140 mmol/L                     (135–145 mmol/L)

K+                      4.4 mmol/L                      (3.5–5 mmol/L)

Creatinine     75 micromol/L            (59–104 micromol/L)

Urea                 7.8 mmol/L                       (1.7–8.3 mmol/L)

Hb                     11.6 g/dL                            (13–17 g/dL)

Medication on admission:

Prednisolone                   10 mg                    o.d.

Fluticasone inhaler         500 micrograms  b.d.

Aspirin                              75 mg                    o.d.

Bumetanide                    1 mg                        o.d.

Combivent nebs             2                              q.d.s.

Enalapril                          5 mg                        o.d.

Uniphyllin Continus       200 mg                    bd

Senna                              2 tablets                  nocte

Warfarin                          5 mg                        o.d.

Zopiclone                        7.5 mg                    nocte

Diclofenac                       50 mg                     p.r.n.

Oxygen  (O2)                                     2 L nasal specs.

1.  How the sign, symptoms & pathophysiology of COPD, relates to the patient?

2.  Comment on the current drug therapy and describe the role of O2 in this patient.

3.  What are the social issues in treating this patient at home?

Case Study-16 Answer  | Explore more  Case Studies  |  Guidelines

Case Study-17 ( Type-1 Diabetes in Pregnancy )

Mrs Jaya is a 36-year-old married lady who has type 1 diabetes. She undertook a home pregnancy test because she was feeling particularly nauseated in the mornings and her period was late. The  test was positive  confirming that she was pregnant.

However, at 8 weeks, she experienced  vaginal bleeding  and  abdominal pain . She attended the Accident & Emergency department, where a miscarriage was confirmed.

Upon questioning, it was discovered that she had been taking  folic acid 400 μcg daily  for the previous 6 months but had not received any pre-conception diabetes care.

Her most recent HbA1c was 7.3% (56 mmol/mol). Her regular medications are  ramipril 10 mg daily ,  simvastatin 40 mg daily ,  insulin glargine at night  and  insulin aspart three times daily  with meals.

  • Why should women of childbearing age be offered advice about pregnancy?
  • Was she taking appropriate dietary supplements prior to conception?
  • What advice should she be given with respect to her regular medication?

Go to Case Study-17 Answers

Case Study-18 (Type-2 Diabetes)

Case scenario:.

Mk a 68-yr-old woman who has previously diagnosed with type 2 diabetes 15 years ago came to the clinic for her regular follow-up. she reports paraesthesia’s in her feet at night but her sleep was not disturbed by these symptoms. She checked her blood sugars at home two to four times each day and found them to range from 100-250 mg/dl.

She reported compliance with her medications that included lisinopril 20 mg OD, hydrochlorothiazide 25 mg OD, diltiazem 180 mg OD, glargine insulin 0.4 units/kg , atorvastatin 40 mg , and aspirin 81 mg OD.

Past medical history & medication history :

  • Type 2 diabetes, hyperlipidaemia and hypertension diagnosed at the age of 47. For these conditions she was treated with metformin, atorvastatin, and hydrochlorothiazide and maintained Hb A1c below 7.0 %, LDL below 100 mg/dl, and blood pressure below 135/80 mmHg for many years.
  • At the age 59, she was noted to have an increase in her urine albumin to 54 mg/g of creatinine and lisinopril was added to her regimen.
  • Three years later at the age 62, her serum creatinine increased to 1.56 mg/dl, and her GFR was estimated to be 36 ml/min. Her Hb A1c increased to 7.2% and she began to experience paraesthesia’s in her feet. The metformin was discontinued, and she was started on glargine insulin.
  • At age 63, she developed proliferative retinopathy in her right eye and underwent laser photocoagulation. Bilateral macular edema developed at age 65.

Examination – Present visit:

  • BMI – 38.2 kg/m² (height – 151 cm, weight – 87 kg)
  • Blood pressure – 142/90 mmHg
  • Heart rate – 68 bpm
  • Retinal exam – significant for panretinal photocoagulation changes with scattered dot haemorrhages and macular edema in both eyes.
  • Heart, lung and abdominal exams were unremarkable.
  • Extremities – Pulses in her feet were reduced, but her feet were warm and without ulcers. She was unable to detect a Semmes Weinstein 5.07 monofilament on the soles of her feet.

Lab parameters:

  • Hb A1c : 7.9%.
  • Serum creatinine : 2.45 mg/dl.
  • GFR : 20 ml/min.
  • Calcium : 9.8 mg/dl. Phosphorus                                                  : 4.6 mg/dl.
  • PTH : 75 pg/ml
  • Total cholesterol : 188 mg/dl,
  • Triglycerides :82 mg/dl,
  • High-density lipoprotein cholesterol :42 mg/dl,
  • LDL cholesterol  : 131 mg/dl
  • Hb  : 10.8 g/dl.
  • Prepare pharmaceutical care plan for this case study.

Go to Case Study-18 Answers

Case Study-19 (Thyroid Disorder)

A 55-year-old man presents to his general practitioner, complaining of lack of energy. He has become increasingly tired over the past 18 months. He works as a solicitor and describes episodes where he has fallen asleep in his office.

He is unable to stay awake after 9:30 pm, and sleeps through until 7:30 am. He finds it difficult to concentrate at work, and has stopped playing his weekly game of tennis. He had an episode of depression 10 years ago related to the break-up of his first marriage. He has no current personal problems. He has had no other major illnesses.

His brother developed type 1 diabetes mellitus at the age of 13. On direct questioning, he has noticed that he has become more constipated but denies any abdominal pain or rectal bleeding. He has put on 8kg in weight over the past year.

Examination:

On examination he is overweight. His facial skin is dry and scaly. His pulse is 56/min, regular and blood pressure 146/88 mmHg. Examination of his cardiovascular, respiratory and abdominal systems is unremarkable. Neurological examination was not performed.

Haemoglobin                                               10.3 g/dL                              13.3–17.7 g/dL

Mean corpuscular volume (MCV)          92 fL                                       80–99 fL

White cell count                                         4.3 x 10⁹/L                            3.9–10.6 x 10⁹/L

Platelets                                                        154 x 10⁹/L                           150–440 x 10⁹/L

Sodium                                                          140 mmol/L                         135–145 mmol/L

Potassium                                                    4.4 mmol/L                          3.5–5.0 mmol/L

Urea                                                               6.4 mmol/L                          2.5–6.7 mmol

Creatinine                                                    125 μmol/L                        70–120 μmol/L

Glucose                                                         4.7 mmol/L                          4.0–6.0 mmol/L

Calcium                                                         2.48 mmol/L                        2.12–2.65 mmol/L

Phosphate                                                    1.20 mmol/L                        0.8–1.45 mmol/L

Cholesterol                                                  6.4 mmol/L                          3.9–6.0 mmol/L

Triglycerides                                                1.4 mmol/L                          0.55–1.90 mmol/L

Urinalysis: nothing abnormal detected (NAD)

  • What is the likely diagnosis? Main differential diagnosis?
  • How would you further manage this patient?

Go to Case Study-19 Answers

Case Study-20 (Thyroid Disorder)

A 40-year-old woman (51kg) presents with complaints of left side chest pain since 4 months that last for 1-2 hours and radiates to back, increase on exertion; associated with palpitation, perspiration. Headache since 1 month in occipital region that lasts 30-40 min. Generalised weakness, easy fatigue, facial swelling, peri-orbital swelling, bilateral lower limb swelling.

Past medication history :

Blood transfusion 2 times 4 months back.

Temp : Afebrile                               Pulse: 86 beats/min

BP: 130/84mmHg (pattern for hypotension seen for 2 days)

Lab details:

Hb                                           10.1 g%                                 13-17g%

WBC                                      4600 cumm                        4000-11,000 cumm

PCV                                        31.3%                                    36-47%

MCV                                       85.2fL                                   82-92fL

MCH                                       27.6pg                                   27-31pg

MCHC                                    32.3%                                    32-36%

Platelet                                1.55 lac/cumm                    1.5-4.5 lakh/cumm

RBC                                        3.67 millions/ μ L              4.4-5.9 millions/ μ L

RDW                                      17.7%                                    13-15 %

Pus & epithelial cells        1-2/hpf

Reticulocyte count            0.5%                                       0.5-2.5%

Na                                           135 mmol/L                        135-145mmol/L

K                                              4.2mmol/L                           3.5-5.5mmol/L

Cl                                            100mmol/L                          98-110mmol/L

Creatinine                           0.8mg%                                0.6-1.3 mg%

Glucose : RBS                      103mg%                               70-140mg%

RBC Smear : Mild anisocytosis                                                                                  

Stool Test ( Occult Blood) : Positive

TSH                                         >100 miu/ml                       >15miu/ml

Total T4                                0.10 ng /dL                          0.82- 2ng/dL                      

Free T4                                  0.10ng /dL                           0.9- 2.3 ng/dL        

ECG : normal sinus rhythm , poor P-wave progression

LVEF: 60%; Grade 1 diastolic function; mild MR; mild TR; mild PAH; Concentric LVH

            Local part

Findings: Left lobe P/O  — hyperplastic nodule

                  Both lobe P/O – inflammatory etiology thyroiditis

Inj pantoprazole    40mg                                                                         12hrly

Infusion 0.45% NS 500ml + 2 ampoule Optineuron at 60cc/hr

Tab Fdson MP Forte                                                                                0-1-0

Tab febac XT                                                                                             1-0-1

Tab Thyronorm      25mg                                                                        1-0-0

  • What is the likely diagnosis in this case? And are lab finding clinically justified?
  • What are the Pharmacist Intervention points in this case?
  • Patient counselling regarding drug & disease?
  • Write the Generic Names of above advised drugs.

Go to Case Study-20 Answers

Case Study-21 (Thyroid Disorder)

A 34-year-old pregnant woman came to the emergency department with complaints of fetal movements not felt since 1 day ago.

At present the patient is the third pregnant at the age of 23-24 weeks, so far the regular control to the midwife and specialist doctors.

The patient has never experienced bleeding or trauma before. A history of previous labour was normal with a healthy baby. The patient has been suffering from Grave’s disease for the past 10 years and has not been treated for one year.

During pregnancy, the patient feels palpitations and tightness during activity. The patient also complaint about weight loss and fatigue.

The patient’s general condition appeared weak with a blood pressure of 197/87 mmHg and pulse 148 times per minute regular.

The exophthalmos and the thyroid gland feel soft in the neck without pain.

Laboratory tests results:

 An increase in FT4 levels of 75.62 pmol / L and low TSHs levels of 0.005 µIU / mL. Wayne index with a value of 23 or found signs of toxicity and the Burch Wartofsky scale with a value of 45 or impending thyroid storm.

Q1 . What is the likely diagnosis ?

The patient was taken to the endocrine section and given methimazole 30 mg twice daily, propranolol 30 mg twice daily, and Lugol 5 drops per 6 hours .

The termination of pregnancy is carried out by vaginal delivery and administration of oxytocin postpartum.

The patient complained of shortness of breath and anxiety after two hours postpartum.

Patient was consulted in the Cardiology division with pulmonary oedema and advised giving diuretics and vasodilators .

Patient was given nitroglycerin at a dose of 5 meq per hour and furosemide 30 mg per hour intravenously to improve the condition of pulmonary oedema.

Close monitoring is carried out on the patient for several days until the patient’s condition is stable.

Q2. Was the Treatment given to the patient justified according to clinical condition?

Q3. Widely used anti-thyroid drugs in pregnant women with hyperthyroidism?

Patient is planned to go home for outpatient care after monitoring side effects and postpartum complications. Hemodynamic condition is stable with blood pressure 110/70 mmHg and pulse 92 times per minute. Patient was given anti-thyroid therapy and beta-blockers on discharge. During treatment at home, patient is advised to monitor thyroid function to the clinic every once a month.

Q4. What should be the Patient Counselling points ?

Go to  Case Study-21 Answers

Case Study- 22 (Thyroid Disorder)

A 55-year-old woman presents with complaints of pus discharge per vagina since 8-9 months. Decreased urine output. Palpitation 3-4 months. Shortness of breath 4-6 months

Past medical history :

Hysterectomy 10-12 years ago

Social History :

“ Chulha ” user

Temp : Afebrile                               Pulse: 140 beats/min

BP: 126/80mmHg

Provisional Diagnosis :

Atrial Fibrillation w/ hemorrhoids w/ hyperthyroidism w/ perianal fissure.

Hb                                           13.2 g%                                 13-17g%

WBC                                       8100 cumm                         4000-11,000 cumm

Platelet                                  2.11 lac/cumm                    1.5-4.5 lakh/cumm

Pus cell                                  5-6

Epithelial cells                     1-2/hpf

Na                                           142 mmol/L                        135-145mmol/L

K                                              3.8mmol/L(3 rd day: 2.9)          3.5-5.5mmol/L

Cl                                             106mmol/L                         98-110mmol/L

Creatinine                             0.6mg%                                0.6-1.3 mg%

Urea                                       24

Mg                                          2.1

Ca                                          8.6

Protein                                 6.1 g %                                    6.0-8.0 g %

Albumin                                3.4g%                                    2.7-5.0 g %

Globulin                                2.7 g%                                   2.5-4.0 g %

SGOT                                     59

SGPT                                      27

Bilirubin                                1.6

TSH                                         0.01                                        0.39-5.0

Free T3                                  7.01                                        2.1-3.8

Free T4                                  4.28ng /dL                            0.9- 2.3 ng/dL         

CRP                                         <10                                         0-6.0

ECG : Flat T- wave (V5) atrial fibrillation.

ECO : Mild MR; Mild TR; LVEF: 60%

Inj  Dilzem                            12.5mg                                 stat w/ 10ml NS

Tab Dilzem                           30mg                                     1-1-1

Inj Lasix                                20mg                                     1-0-0

Inj PAN                                  40mg                                     24 hrly

Tab Metro                           400mg                                  1-1-1             

Syp Duphalac                     2tsp                                       0-0-1             

Tab Ciplox T2                      500/600                               1-0-1             

Tab Neomercazole            10mg                                                 1-1-1             

Tab Propanolol                  10mg                                    1-0-1

Syp Potklor                          15ml                                     1-1-1 

Metrogyl ointment                                                           1-1-1-1 

Inj NS + KCl 1 amp                                                             12hrly                       

  • Are there any drug-drug interactions?
  • Is the dosing of medication for hyperthyroidism, according to standard treatment guidelines?
  • What should be Patient counselling in this case, regarding drug & disease?

Go to Case Study-22 Answers

Case Study-23 (OB/GYN: Oral Contraceptive Use)

An 18-year-old female presents with an absence of periods for 6 months. This has occurred twice before in the past but on both occasions menstruation returned so she was not too concerned. Her periods started at the age of 12 years and were initially regular.

She has no medical history of note and denies any medication. She is currently in her first year at university. She sometimes follows inconsistent diet plan. However, She runs most days and reports a ‘healthy ‘ diet avoiding carbohydrate foods and meat. She is the oldest of three siblings and her parents separated when she was 12 years. She has minimal contact with her father and lives mainly with her mother who she says she gets on well with. She has had a boyfriend in the past but has veered away from any sexual relationships.

The woman is tall and thin with a body mass index (BMI) of 15.5 kg/m².

There is evidence of fine downy hair growth on her arms.

Heart rate is 86/min and blood pressure 100/65 mmHg.

Abdominal examination reveals no scars or masses, and genital examination is not performed.

INVESTIGATIONS:

Follicle-stimulating hormone                  1.0 IU/L                     Day 2-5

 Luteinizing hormone                                0.8 IU/L                     Day 2-5

0.5-14.5 IU/L

Oestradiol                                                    52 pmol/L                 70-600 pmoI/L

Prolactin                                                       630 mu/L                  90-520 mu/L

Testosterone                                               1.6 nmol/L                0.8-3.1 nmol/L

Diagnosis : Hypogonadotrophic hypogonadism

  • What should be the further investigation ?
  • How, this patient would be managed ?
  • What Patient counselling points should be included in this case?

Go to Case Study-23 Answers

Case Study-24 (OB/GYN: Oral Contraceptive Use)

A 19-year-old female was referred by her general practitioner with increased hair. She first noticed the problem when she was about 16 years old and it has progressively worsened such that she now feels very self-conscious. It also affects her forming relationships.

The hair growth is noticed mainly on her arms, thighs and abdomen. Hair has developed on the upper lip more recently. She has tried shaving but this seems to make the problem worse. She feels depilation creams are ineffective. Waxing is helpful but very expensive and she has bleached her upper-lip hair.

Her GP has not prescribed any medication in the past.

There is no significant previous medical history of note. Her periods started at the age of 13 years and she bleeds every 30-35 days. The periods are not painful or heavy and there is no intermenstrual bleeding or discharge. She has never been sexually active.

Examination

On examination she has an increased body mass index (BMI) of 29 kg/m². The blood pressure is 118/70 mmHg. There is excessive hair growth on the lower arms, legs and thighs and in the midline of the abdomen below the umbilicus. There is a small amount of growth on the upper lip too.

The abdomen is soft and no masses are palpable. Pelvic examination is not indicated.

INVESTIGATIONS

Follicle-stimulating hormone (FSH)                  7 IU/L                         Day 2-5

                                                                                                                        1-11 IU/L

Luteinizing hormone (LH)                                    12 IU/L                      Day 2-5

                                                                                                                        0.5-14.5 IU/L

Prolactin                                                                   780 mu/L                  90-520 mu/L

Testosterone                                                           3.2 nmol/L                0.8-3.1 nmol/L

Thyroid-stimulating hormone                            4.9mu/L                    0.5-5.7 mu/L

Free thyroxine                                                         14.7pmol/L              10-40pmol/L

Provisional Diagnosis : Polycystic ovarian syndrome(PCOS)

Clinical features: Hirsutism , acne, increased BMI and slight menstrual irregularity

Go to Case Study-24 Answers

Case Study-25 (Hormone Replacement Therapy)

History of present illness :.

A 51-year-old newly postmenopausal woman suffering from intense hot flashes and night sweats, as well as anxiety, insomnia, and stress, presents to the hospital.

The doctor presents a breakdown of the patient’s self-reported symptoms before going into the details of her test results – where she highlights the biochemical factors that explain both her symptoms and her observed results.

Symptoms she rated as severe , included, depression, anxiety, and sleep disturbances , moderate symptoms included hot flashes, night sweats, foggy thinking, vaginal dryness, and mood swings. In addition, she had multiple symptoms that she rated as mild in severity.

The Patient had already tried multiple supplements in an attempt to address her own symptoms – these included elements like magnesium, calcium, selenium, zinc, and copper, as well as L-theanine and Rhodiola, vitamin D, fish oil, and some probiotics.

Investigation:

Saliva Hormone Test Results:

Name                                                      Lab value                     Normal range

Estrogen hormone                                     0.3                                0.9-3.1 pg/ml

Progesterone hormone                            5                                   12-100 pg/ml

DHEA                                                           1                                   2-23 ng/ml

Testosterone                                              90                                 16-55 pg/ml

           

Serotonin metabolite 5-HIAA

DOPAC

 

Neurotransmitter Test revealed:

LOW levels of:

Nor Epinephrine

  • What should be the further investigation for this patient ?
  • Give the provisional diagnosis for this case.

Go to Case Study-25 Answers

Case Study-26 (Hormone Replacement Therapy)

A 25 year-old woman with menarche at 13 years and menstrual periods until about 1 year ago.

Complains of hot flushes, skin and vaginal dryness, weakness , poor sleep and scanty and infrequent menstrual periods of a year duration.

She visits her gynecologist, who obtains plasma levels of follicle-stimulating hormone and luteinizing hormone, both of which are moderately elevated.

She is diagnosed with premature ovarian failure, and estrogen and progesterone replacement therapy is recommended.

A dual energy absorptiometry scan (DEXA) reveals bone density t-score f <2.5 SD, ie., Frank osteoporosis.

  • How should the ovarian hormones she lacks be replaced?
  • What extra measures should she take for her osteoporosis while receiving treatment?

Go to Case Study-26 Answers

Case Study-27 (Osteoporosis)

History of present illness:.

     A 73-year- old woman presenting for a physical examination who looks and feels healthy and well.

Medication history:

Takes a multivitamin daily plus a calcium tablet

History of low-trauma Colles’ fracture (11 years ago)

Physical Examination:

Weight:  55 kg (121 lbs.)

Height: 157 cm (5’2”)

Body Mass Index (BMI): 22.3 kg/m2

*Changes in height and weight can be signs of vertebral fractures

INVESTIGATION:

BMD :                 Value                       

Spine                -3.6;

Hip                     -2.0

Provisional Diagnosis : Osteoporosis with moderate risk of fracture

  • What all investigations are needed for the further management?
  • How this patient would be managed ?
  • How will you do the Patient counselling in this case?
  • Mention the indications for BMD testing.

Case Study-28 (Osteoporosis)

A 64-year-old retired firefighter Retired nine years ago; now doing contract carpentry Presents for physical examination, complaining his back has been “worse than usual” the past three weeks.

On no medications

Prior smoker (45 pack/year history)

Quit smoking one year ago

Height: 180 cm (5’11”)

Patient recalls being 185.5 cm (6’1”)

Weight: 80 kg (up 5 kg from one year ago)

Body mass index (BMI): 24.7 kg/m2

*Other indicators of vertebral fracture in physical examination: Rib-pelvis distance and occiput-wall distance

INVESTIGATION:      

  Screening for osteoporosis with dual energy X-ray absorptiometry (DXA) is  T-score -1.9 at femoral neck

Lateral thoraco-lumbar spine X-ray is ordered to rule out vertebral compression deformities

*The radiologist makes note of two vertebrae being wedge shaped and just meeting the criteria for vertebral compression fracture

  • What is the significance of T- Score?

Go to Case Study – 28 Answers

Case Study-29 (Glaucoma)

The patient, a 61 year-old female retired school teacher, has not had an eye exam in 10 years. She reports no problems with driving, watching TV, computer or reading. She uses over-the-counter readers for close work. She admits to mild eye burning feeling after reading for long periods of time or in the afternoons. She denies any flashes, floaters, pain, redness or double vision.

Past Ocular History:

Presbyopia. No prior eye surgeries, hx of eye trauma, amblyopia or strabismus.

Ocular Medications:

Hypertension

Surgical History:

Cesarean delivery x 1

Past Family Ocular History:

Cataract surgery in her mother and father. Negative for macular degeneration, glaucoma or blindness.

Never smoked

Medications :

Hydrochlorothiazide

Allergies :

Denies any recent illness or any new CNS, heart, lungs, GI, skin or joint symptoms.

Ocular Exam:

Visual Acuity (cc):

IOP (tonoapplantation):

OD: 21 mmHg

OS: 23 mmHg

Equal, round and reactive to light, no APD

Extraocular Movements:

Full OU, no nystagmus

Confrontational Visual Fields:

Full to finger counting OU

Normal, both sides

Lids and Lashes:      Normal OU

Conjunctiva/Sclera:         Normal OU

Cornea:                 Clear OU; no krukenberg spindle or embryotoxon

Anterior Chamber: Deep and quiet OU

Iris:              Normal, no neovascularization or atrophy

Lens: 1+ nuclear sclerotic cataracts OU

Anterior Vitreous: Clear OU

Dilated Fundus Examination:

OD:    Clear view, CDR 0.7 with sharp optic disc margins (no obvious rim thinning or disc hemorrhage); flat macula with normal foveal light reflex; normal vessels and peripheral retina.

OS:    Clear view, CDR 0.8 with sharp optic disc margins (no obvious rim thinning or disc hemorrhage); flat macula with normal foveal light reflex; normal vessels and peripheral retina

Gonioscopy: Open angles with minimal pigmentation in the trabecular meshwork, no synechiae OU

Automated visual field test: superior arcuate defect in both eyes

Pachymetry: 560 OD; 551 OS (within normal range)

  • What findings are needed for the diagnosis of POAG?
  • Explain the result of the above mentioned investigations of the ocular exam?
  • Enlist the Patient counselling points in this case?

Go to Case Study – 29 Answers

Case Study-30 (Galucoma)

The patient is a 51 year-old stay-at-home mother who presented to the ED with severe R brow, R cheek and R eye pain/pressure that started 2-3 hrs prior. She also noticed blurry vision from that eye and rainbow-colored halos around lights around the same time. 

Accompanying symptoms include acute nausea. She has vomited twice since feeling eye pain. Denies prior episodes. Denies flashes, floaters or diplopia. There is mild redness in the R eye.

Hx myopia OU No prior eye surgeries, trauma, amblyopia or strabismus

Degenerative disc disease – lower back

Father: chronic angle-closure glaucoma. No FHx of macular degeneration or other blinding diseases.

30 pack/year smoking history Drinks alcohol on occasion No illicit drug use

Medications:

Multivitamins Vicodin prn (uses few days/month for back pain)

Denies recent illnesses, new medications, CNS, lungs, GI, skin, joint problems except for above.

Ocular Exam

Visual Acuity (cc): OD: 20/70 OS: 20/20 IOP (tonoapplantation): OD: 62 mmHg OS: 11 mmHg Pupils: OD: pupil mid dilated, sluggish to respond to light. OS: pupil round and reactive to light No obvious APD Extraocular Movements: Full OU No nystagmus Confrontational Visual Fields: Full to finger counting OU External: Normal, both sides

Lids and Lashes: Normal OU Conjunctiva/Sclera: Mild diffused injected conjunctiva OD, Normal OS Cornea: Hazy cornea OD, Normal OS Anterior Chamber: Shallow anterior chamber 360 OD – hazy view, Deep and quiet OS Iris: Mid dilated iris OD, Normal OS Lens: Trace nuclear sclerosis OU Anterior Vitreous: Clear OU

OD: Dilation not performed, examination through undilated pupil showed hazy view, CDR 0.5 with sharp optic disc margins; flat macula OS: Dilation not performed, examination through undilated pupil showed clear media, CDR 0.4 with sharp optic disc margins; flat macula

Gonioscopy: Performed with Abraham 4 mirrored lens, shallow angle and no view of the angle structures 360 degrees OD, shallow angle with view of the trabecular meshwork 360 degrees OS without synechiae, mild pigmentation 360 degrees OS

1. What is the Diagnosis in this case? 2. What is the mechanism of angle closure in an episode of acute angle closure glaucoma resulting from a pupillary block? 3. Mention in brief the surgical options for the disease? 

Go to Case Study – 30 Answers

Case Study - 31 (Conjunctivitis)

The patient is an 31 year-old male who reports a 4-day history of irritation and itching, first in the L eye followed by the R eye one day later. Both eyes have also had a mild yellow discharge and mattering of the eyelids making it difficult to open the eyes in the morning. There is minimal eye redness but no foreign body sensation, flashes, floaters, decreased vision or diplopia. Not using any drops. No environmental exposures to the eyes.

No history of eye trauma, surgery, amblyopia or strabismus. No history of contact lens use.

Born at term without complications

No history of glaucoma, macular degeneration or other blinding diseases

No smokers at home

Exposure to the common cold (neighbor friend). No history of environmental allergies, recent cold, CNS, heart, lung, GI, skin or joint problems.

OD: 17 mmHg

OS: 14 mmHg

Equal, round and reactive OU, no APD

Normal-appearing orbital structures; no redness or swelling either eye

Lids and Lashes:           Crusted dry flaky material on eyelashes OU, no follicles in inferior or superior fornix OU. No foreign body in fornices OU

Conjunctiva/Sclera               Mild conjunctival injection OU, no chemosis

Cornea                           Clear OU, no infiltrates

Anterior Chamber               Deep and quiet OU

Iris                       Normal OU

Lens                   Normal OU

Anterior Vitreous            Clear OU

Dilated Fundus Examination :

OD     Clear view, CDR 0.2 with sharp optic disc margins, flat macula with normal foveal light reflex, normal vessels and peripheral retina

OS      Clear view, CDR 0.2 with sharp optic disc margins, flat macula with normal foveal light reflex, normal vessels and peripheral retina

No preauricular or submandibular lymph node enlargement

  • What are the patient counselling points in these case?
  • What is not a typical exam finding of conjunctivitis?

Go to Case Study-31 Answers

Case Study - 32 (Conjunctivitis)

History: A 35 year old female presented with complaints of sticky eyelids, watery and green ocular discharge, redness, soreness and slightly blurred vision in both eyes.

The symptoms started 2 weeks ago; the right eye was affected first.

Initially prescribed with Chloramphenicol (0.5%), no improvement was reported.

Chloramphenicol was replaced with Tobramycin, improvement was noticed only on 1 st day.

·         Ocular swab taken for PCR and was advised to stop tobramycin.

Again, on the visit, she reported that the left eye now felt worse and noticed an increase in green discharge, however a decrease in redness since the drop discontinuation.

Past medical history : A metal foreign body removal from the right eye – 10years ago.

She had flu, 3 weeks ago but is in good health now (was not atopic and she was not taking any medications.)

Vision: OD :    6/6                                                      OS :     6/5

Slit lamp examination:

OD: Mild conjunctival injection. Mild follicular change of the inferior palpebral conjunctiva. No corneal surface involvement. No anterior chamber inflammation.

OS: 360 degree conjunctival injection grade 3. Multiple white nodules at the limbus. Mild follicular change of the inferior palpebral conjunctiva. No anterior chamber inflammation.

Other Investigation:

PCR test : negative for Adenovirus, Varicella-Zoster virus, Herpes Simplex virus and Chlamydia trachomatis.

Provisional Diagnosis: Bacterial Conjunctivitis

For left eye:

Instill one drop of ofloxacin                     q2h

* reduce the dosage to “qid” the next day if the condition improves.

For right eye:

Ofloxacin                                                      qid     

Revisit after 1 week.

  •   What are the possible differential diagnosis in this case?
  •   What is the role of Ofloxacin in this Patient’s case?
  •   Why Tobramycin wasn’t readvised after the PCR result, is there any evidence suggestive for the change?

 Go to Case Study -32 Answers

Case Study - 33 ( Pediatrics)

Leanne is a 6-year-old girl whose teachers have suggested that her parents take to her GP. They have noticed that she seems to have problems listening and to be daydreaming a lot in class. The GP asks whether her parents have also seen her daydream. Her mother has, but has not thought much about it. However, more recently, it seems to have been happening more frequently. On direct questioning by the GP, Leanne’s mother thinks that these daydreams or ‘trances’ as she calls them sometimes occur when Leanne is in the middle of doing or saying something, and they interrupt her activity. Leanne’s birth and early medical history, including her development, have been normal. There was a history of epilepsy on her father’s side of the family. Her younger brother and older sister are well.

1.What should Leanne’s GP consider as a possible diagnosis?

2. What further investigations should the paediatrician or paediatric neurologist Request, and what considerations should be taken into account?

3. What should the discussion around medication include, and what medications may be Prescribed?

Go to Case Study- 33 Answers

case study examples pharmacology

Answers for Case Studies for Pharm.D / PharmD students:

Case study-1 answers.

  • According to AHA guidelines the target BP goal for hypertensive patients with CAD is <130/80 mmHg.
  • Beta blockers, ACE inhibitors, ARB’s, thiazide diuretics, and calcium channel blockers (CCB). ( Beta – blocker , first line agent to treat HTN in patients with Angina).

Explanation : stepwise                                 Go to CASE STUDY 1

Step 1:  In HTN patients with angina as comorbidity Beta blockers are the first line agents. Here metoprolol ( beta- blocker is already prescribed). Check whether target BP goal is achieved or not. Here target BP goal is not achieved in this patient (noted as 166/93 mmHg on his today’s visit to clinic). Follow step-2 if target Bp goal is not achieved. Step-2 :  Prescribe the maximum dose of metoprolol (100 mg twice daily). In this case already maximum dose is prescribed. Follow step 3 if target BP goal is not achieved. Step-3 :  Add another anti-hypertensive agent from ACEI’s / ARB’s / Thiazide diuretics/ CCB’s to the Beta -blocker. Note :  Here the patient is tolerant to METOPROLOL , however if the patient is intolerant to METOPROLOL ,Consider ACEi/ARB as first line agent and add CCB/thiazide as adjuvant.

       Go to CASE STUDY 1  

Case Study-2 Answers

Poor diet control, non-compliance to medication and obesity were considered to be the main reasons for poorly controlled hypertension and diabetes in Mr. MK

  • HbA1C target value should be less than 6.5%
  • Target value for BP should be 130/80 mm Hg.
  • Achieving these targets found to be beneficial in decreasing the Cardiovascular morbidities and prevention or delay of CKD progression.

3.Ans)   FOR HTN:                                                         Go to  Case Study 2.

For diabetic patients whose blood pressure levels are ≥160/100 start with two anti-hypertensive agents. Diabetic and hypertensive Patients with albuminuria consider ACEI /ARB as first line agent and add adjunctive drug from classes calcium channel blockers or Diuretic. Eg : Telmisartan (ARB) + Furosemide (diuretic)

FOR DIABETES: Metformin is the preferred initial pharmacologic agent for the treatment of type 2 diabetes. Once initiated, metformin should be continued as long as it is tolerated and not contraindicated; If the A1c levels are not achieved within the target levels, consider other class of agents to metformin. LIRAGLUTIDE (GLP1 AGONISTS) has the benefit of decreasing both the HBA1c values as well as body weight. So replacing GLICLAZIDE with LIRAGLUTIDE increases the effectiveness of treatment. Rx : Metformin + LIRAGLUTIDE .

4. Ans) DRUG INTERACTION:

AMLODIPINE+METFORMIN  – Amlodipine decreases effect of Metformin by pharmacodynamic antagonism. Management  – Monitor closely after withdrawal of Amlodipine whether the blood sugar levels are decreased or any hypoglycemic condition is observed.

5.Ans) Patient counselling:

  • Counsel the patient regarding importance of medication adherence.
  • Suggest to maintain regular exercise or at least do walking for 1 hour as it will be more beneficial for the weight loss.
  • Follow  DASH  diet to control HTN and reduce cardiovascular comorbidities.
  • Reduce intake of saturated fat and trans-fat; increase of dietary n-3 fatty acids, viscous fiber, and plant stanols/sterols.
  • Counsel the patient for regular checkup of A1c levels at least four times yearly.

Considerations:                                         Go to  Case Study 2.

  • LIRAGLUTIDE -1.2MG SC as GLP 1 agonists mainly decrease the HbA1c value (~0.8% -1.6%)and it also decreases body weight to (1-3kgs). Monitor closely while using liraglutide as it is contraindicated in medullary thyroid carcinoma and Monitor the calcitonin levels .
  • Metformin is contraindicated in patients with an eGFR ,30 mL/min/1.73 m2;
  • eGFR should be monitored while taking metformin;
  • The benefits and risks of continuing treatment should be reassessed when eGFR falls ,45 mL/min/1.73 m2;
  • Metformin should not be initiated for patients with an eGFR ,45 mL/min/1.73 m2; and
  • Metformin should be temporarily discontinued at the time of or before iodinated contrast imaging procedures in patients with eGFR 30 60 mL/min/1.73m2

Within these constraints, metformin should be considered the first-line treatment for all patients with type 2 diabetes, including those with CKD.

  • If  cost of medication  was the reason for  non-compliance  replace Liraglutide with insulin therapy basal insulin with lowest acquisition cost. If replaced with insulin there must be strict diet control and exercise that helps the patient to loose weight.
  • If Furosemide was given as an adjunctive drug advice the patient to observe closely for hypoglycemia, if needed switch to other anti-hypertensive agent.

Go to  Case Study 2.  | For more  CASE STUDIES .

Case Study-5 Answers

1.   what is the treatment goal and strategy for this case.

Ans :  Goals of therapy of CSA are :

  • Amelioration of anginal symptoms and improved angina-free exertion capability
  • Prevention or reduction of subsequent acute MI, UA, or ISD and there by increasing quality of life.

1.  Vasodilating – Beta-blockers as initial therapy:

Vaso-dilating beta blockers like Carvedilol and nebivolol can be used as first line agents, as they don’t have negative metabolic effects as compared to non-vasodilating beta-blockers like metoprolol.

Usually non-vasodilating  beta-blockers like metoprolol, propranolol etc., have high risk for new onset diabetes or masking of hypoglycemia.

{ Usually beta-blockers are preferred as initial therapy in the absence of any contraindications or chances for severe side effects.

In case of contraindications to beta blockers → use long acting calcium channel blockers  as initial therapy. }

2.  Lipid lowering agents  :

Lipid lowering agents like atorvastatin can be used as prophylactic therapy to prevent or minimizing risk of CAD.

3. Anti-platelet agents :

Adding  Aspirin  to the treatment shows a good evidence of Preventing MI and Death and Reducing Symptoms.

2. Suggest the best follow-up for this case.

Ans:     Monitoring of Symptoms and Antianginal Therapy:

During the first year of therapy, evaluations every 4 to 6 months are recommended. After the first year of therapy, annual evaluations are recommended if the patient is stable and reliable enough to call or make an appointment when anginal symptoms become worse or other symptoms occur.

  • Has the patient decreased the level of physical activity since the last visit?
  • Have the patient’s anginal symptoms increased in frequency and become more severe since the last visit? If the symptoms have worsened or the patient has decreased physical activity to avoid precipitating angina, then he or she should be evaluated and treated according to either the unstable angina or chronic stable angina guidelines, as appropriate.
  • How well is the patient tolerating therapy?
  • How successful has the patient been in reducing modifiable risk factors and improving knowledge about ischemic heart disease?
  • Has the patient developed any new comorbid illnesses or has the severity or treatment of known comorbid illnesses worsened the patient’s angina.

3.What are the conditions which worsens the symptoms of angina (in general)?

  • Tobacco use,
  • high blood pressure,
  • high cholesterol,

Go to  Case 5  | Explore more  Case Studies  | Go to  guidelines

Case Study-6 Answers

1 .  what information and counselling points would you include.

Ans:  Mr SW  has already been taking steps  to address his unhealthy lifestyle and reduce his cardiovascular risk.

  • Furthermore, he can be counselled for risk factors for cardiovascular disease that are commonly distinguished as modifiable and non-modifiable. Modifiable risk factors are those that can be controlled, treated or modified. These include  smoking, diet  (especially cholesterol and lipids),  weight and obesity, physical exercise, blood sugar levels and hypertension .

Non-modifiable risk factors are those that cannot be changed, such as age, gender, ethnicity and family history.

  • However, Mr SW should be counselled about how primary and secondary prevention helps in delaying the progression of disease by early detection of any further onset of disease .
  • He should be given proper information about the side effects (like muscle spasm, headache, flushing, breathlessness on physical work) & use, storage of medications; and how the benefits of treatment are much more as compared to side effects. So that, Mr SW will understand the rational use of medication and develop better medication adherence.
  • Also, if Mr SW founds any side effects that he is concerned about ( like impotence in previous ), rather quitting the drug by himself, consult his physician for the same. Drugs can be changed/further treatment option will be suggested based on the situation, the physician finds.

2. How is stable Angina managed ?                          Go to CASE STUDY-6

Ans: Managing stable Angina:

Treatments can be divided into those that reduce the risk of future cardiovascular events, thereby reducing mortality ( secondary prevention ) and those that prevent symptoms ( which can be further subdivided into short- and   long-term relief ).

For Secondary Prevention:

Aspirin  75 mg daily   (reduction in non-fatal MI & vascular events)

Simvastatin  40 mg daily  (starting dose)

  • Alternative / lower dose can be used if contraindication / interactions found.
  • If total cholesterol <4 mmol/L or LDL <2 mmol/L not achieved at initial dose, titration of simvastatin or alternative should be used.

For Short-Acting relief:

GTN      sublingually  (rapid onset of action: within 1-5 min)                                            Go to CASE STUDY-6

  • Minimizes the discomfort
  • Choice of dosage form should be discussed with patient
  • Pt should be counselled for side effects, use & storage
  • Can cause difficulties in Pt with significant arthritis or reduced hand dexterity

For Long-Acting symptom control:

  • 1 st  line treatment should be with either beta-blocker or CCBs.
  • If symptoms are not controlled, the next step is to swap or add the other (usually avoiding the combination of  beta – blocker & verapamil ).
  • Long-acting nitrate
  • Response to treatment should be reviewed 2-4 weeks after starting or changing drug treatment.               Go to CASE STUDY-6

3. What options are there, if Mr. SW experiences further symptoms despite the use of amlodipine?

Ans :   If further symptoms are experienced, i.e., symptoms not adequately

controlled with medication, the relative merits and risks of coronary artery bypass grafts (CABG) versus percutaneous coronary intervention to alleviate symptoms should be discussed. A multidisciplinary discussion should take place when the coronary artery disease is more complex.

REFERENCES:

1. European Society of Cardiology (2006). Guidelines on the management of stable Angina Pectoris.

2. national institute for health and clinical excellence (2011). management of stable angina. clinical guideline 126. london.

Go to CASE STUDY-6

Explore more  Case Study  |  Guidelines  |  Home

Case Study-7 Answers

Ans. the fourth universal definition of mi (udmi) was released in 2018. according to the udmi, mi is defined as myocardial injury in the clinical setting of myocardial ischemia. there are two components:.

  • Myocardial injury which is defined as raise of troponins above the 99th percentile
  • Clinical Setting of myocardial ischemia-symptoms and signs of myocardial ischemia, ECG changes with new ischemic changes or pathological Q waves, imaging evidence like loss of viability, regional wall motion abnormality (RWMA), thrombus in angiography, evidence of thrombus in autopsy, sudden cardiac death.

2. What are the classical ECG criteria for diagnosing STEMI ? Go to Case Study-7  

Ans.  There should be two contiguous leads with ST elevation (measured at J point).

>2.5 mm for men <40 years

>2 mm for men >40 years           in leads V2, V3

>1.5 mm for women.

And/or       >1 mm in other leads in absence of LVH, LBBB.

For posterior MI, in leads v7–v9, 0.5 mm is itself enough to diagnose posterior wall MI in inferior MI.

3. What are the different types of MI ? Go to Case Study-7  

Ans.  UDMI classifies MI in five types:

a) Type 1 MI due to thrombosis of an atherosclerotic plaque

b) Type 2 MI due to myocardial oxygen supply demand imbalance in the context of another acute illness.

c) Type 3 MI presenting as sudden death

d) Type 4 post-percutaneous coronary intervention (PCI)

e) Type 5 postcoronary artery bypass grafting.

4. Justify the treatment given to this patient.  Go to Case Study-7  

  • Aspirin  – ISIS 2 was the landmark trial in 1988 after which aspirin was considered as mainstay in ACS patients
  • DAPT  – benefit of DAPT in ACS setting and post-PCI setting were shown in CURE and PCI CURE trials, where they used Clopidogrel as the p2y12 inhibitor of choice. There was a significant reduction in composite primary endpoint of cardiovascular (CVS) mortality, non-fatal MI and stroke (9.3vs. 11.4%) with number needed to treat (NNT=48)

Ticagrelor was compared to clopidogrel in PLATO trial. Addition of ticagrelor to aspirin was able to further reduce composite primary endpoint of cardiovascular mortality, non-fatal MI, and stroke from 11.7% to 9.8% (P < 0.001) with an insignificant increase in major bleeding (11.6% vs. 11.2%)

  • In this case BMI of the patient was found to be 29.2 which is overweight. This puts patient at high risk for increased LDL, Cardiovascular comorbidities etc.

Statins  have a huge body of evidence both for early initiation and intensive treatment strategy mainly from meta-analysis data by cholesterol treatment trialist. Most of the trials utilized atorvastatin in the STEMI setting. With high intensity statin defined as the atorvastatin 40-80 mg with an ability to decrease LDL cholesterol by >50

  • Beta blockers  found to have robust benefit in people with MI and post MI reduced LVEF.
  • ACE inhibitor  trials are the SAVE, AIRE and TRACE trials which demonstrated mortality benefit of starting ACE inhibitors within first 24 h. AIRE trial used the drug ramipril.

#   Discontinue  Lasix + Spironolactone  :

Early initiation of Lasix and Spironolactone was not was not found to beneficial in Acute Myocardial infarction. ( As MI is one of the main cause for reduced ejection fraction or systolic heart failure consider using Mineralocorticoid receptor antagonists when reduced ejection fraction is less than 40% ).

(These can be used if the patient is having post-MI heart failure or LV dysfunction or Ejection fraction < 40% or diabetic with symptoms of heart failure.)

5. Determine the duration of DAPT In this patient ? Go to Case Study-7  

Ans.  In this case patient has had an acute coronary event (STEMI). He is a 56-year-old male, with only smoking as risk factor. He does not have any features of high bleeding risk. He is a candidate with a Low Bleeding risk with a moderate ischemic risk. PRECISE DAPT SCORE in this patient is <25. Hence, strategy for DAPT duration will be till 12 months with Aspirin 75 mg dose and ticagrelor 90 mg BD dose.  Reassess the patient at the end of 12 months , with DAPT score. If the patient is found to have high ischemic risk features, prefer continuing the patient on ticagrelor 60 mg BD till 3 years’ duration if cost is not an issue for greater benefit .

( Refer : high bleeding risk criteria and Precise DAPT scoring)

Go to Case Study-7  

Explore more  Clinical Case Studies  |  Guidelines  |  Home

Case Study-8 Answers

1. is the diagnosis & treatment given to mr tr justified explain.  case 8  .

Ans:  The diagnosis and treatment given to patient is  appropriate .

             Justification:

Diagnosis  :  The most likely diagnosis was confirmed by angiogram, which identified thrombosis and stenosis in the coronary arteries obtuse marginal, LAD& RCA, requiring deployment of drug-eluting stents. Also, Echocardiogram shows normal LV function.

Treatment  :

  • Drug eluting stents(DES) – for stenosed coronary artery to restore blood flow.
  • Dual antiplatelet therapy (Aspirin+Clopidogrel) – required as DES is inserted, until re-endothelisation occurs. However, treatment courses of 2 nd  antiplatelet agent(clopidogrel) should be kept to a minimum(1-3 months or 1 year)
  • Beta blocker & ACEI started with minimum required dose which is required.
  • High-intensity statins are recommended after an acute coronary syndrome.
  • GTN helps in venodilatation and arterial dilatation & reduction of cardiac ischaemia.

***Major and intermediate drugs interactions are there. However, drugs dosage is already  maintained minimum as recommended , if  used together .

2. Is there a need of counselling?  Case 8  

Ans:  Approx. 50% or more cardiovascular patients do not take their medications as intended. Poor adherence can lead to increase in recurrence, hospitalization, mortality. So, counselling is required to increase positive adherence.

Pt should be informed of drugs effects and side effects and the benefits of medication adherence.

Also, he should be counselled for lifestyle modification.

The patient should be encouraged to report any adverse effects from the drugs.

**He may have to take Aspirin lifelong to prevent secondary cardiovascular events.

Case 8   |  Clinical Case Studies  |  Guidelines  |  Home

Case Study-9 Answers

  • As the patient is at High risk with known CVD, the target goal for LDL-C would be < 70 mg/dl.
  • High-intensity statins (Atorvastatin 40-80 mg , Rosuvastatin 20 – 40 mg ) would be choice of treatment in this patient.
  • If on maximal statin & LDL-C ≥70 mg/dL (≥1.8 mmol/L), add ezetimibe.
  • Using non-statin therapy like niacin would be helpful in the patients who are intolerant to statins. (prior to withdrawing statin therapy check whether the patient is having true intolerance to statin or not).

References :

  • https://www.acc.org/~/media/Non-Clinical/Files-PDFs-Excel-MS-Word-etc/Guidelines/2018/Guidelines-Made-Simple-Tool-2018-Cholesterol.pdf
  • https://www.lipid.org/sites/default/files/6-_lipid_u-_guidelines-_jones.pdf

Case 9 | Clinical Case Studies | Guidelines | Home

Case Study-10 Answers

1. what is the metabolic abnormality present  case 10.

Ans: The obvious abnormal investigation is a very high serum cholesterol with high

LDL and low HDL levels. He has many clinical features to go with the high cholesterol and premature vascular disease. The patient has familial hypercholesterolaemia . He has presented with premature coronary artery disease. His absent pedal pulses suggest peripheral vascular disease.

** The metabolic defect is a result of a reduced number of high-affinity cell-surface LDL receptors. This leads to increased LDL levels. Increased uptake of LDL by macrophage scavenger receptors leads to increased oxidized LDL, which is particularly atherogenic. Triglyceride and VLDL levels are normal or mildly elevated. HDL levels are low. The other major causes of hypercholesterolaemia are familial combined hyperlipidaemia and polygenic hypercholesterolaemia. Familial combined hyperlipidaemia differs from familial hypercholesterolaemia by patients having raised triglycerides. Patients with polygenic hypercholesterolaemia have a similar lipid profile to familial hypercholesterolaemia but they do not develop xanthomata.

2. Discuss the Patient counselling for this case?  Case 10

Ans: The patient is at extremely high risk for further vascular events and especially

occlusion of his coronary artery bypass grafts. His risk depends on the combination of his risk factors, and all of these need attentions.

He should be counselled about following points:

  • To stop smoking,
  • Reduce his alcohol intake
  • Take more exercise and
  • Eat a strict low-cholesterol diet.
  • He should be suggested that diet alone will not control this level of cholesterol. So, a proper adherence with the pharmacological treatment with a statin or combined treatment for this level of hyperlipidaemia, is needed.
  • Importance of periodic lab check-ups.
  • His children should have their lipid profile measured so that they can be treated to prevent premature coronary artery disease.

Case 10 | Clinical Case Studies | Guidelines | Home  

Case Study-11 Answers

1. what would be the possible differential diagnosis  case 11  .

Ans:  The differential diagnosis of a pediatric patient who presents with a  narrow complex tachycardia  includes atrioventricular reentrant tachycardia  (AVRT ), AV nodal reentrant tachycardia ( AVNRT ),  sinus tachycardia ,  atrial flutter ,  atrial fibrillation ,  junctional ectopic tachycardia ,  atrial ectopic tachycardia , and  multifocal atrial tachycardia/chaotic atrial tachycardia .

Provisional Diagnosis: SVT ( supraventricular tachycardia)

2. What is the treatment goal and strategy for this case?   Case 11  

Ans: Treatment goals include:

  • Reducing the symptoms and stabilizing the patient,
  • Terminating the SVT and
  • Stablishing a mechanism to prevent any cardiac emergency condition.

Treatment Strategy:

If the patient is clinically stable , vagal maneuvers may be initially attempted to convert the tachycardia. Such vagal maneuvers may include bearing down (as though having a bowel movement (i.e., Valsalva maneuver)), blowing in a straw or inducing the diving reflex using an ice bag to the face for infants. Other vagal maneuvers such as eyeball pressure and unilateral carotid massage are harmful and should not be performed.

If the patient appears clinically unstable , urgent electrical cardioversion is indicated using 0.5-1 J/kg. If an intravenous line is already in place (antecubital preferred over a hand vein), an  IV bolus of 0.1-0.2 mg/kg adenosine  may be given prior to cardioversion. Adenosine causes a transient AV block and sinus bradycardia, thus interrupting the reentrant circuit involving the AV node and accessory pathway. It must be remembered that this medication is metabolized by the red blood cells and has a very short half-life (approximately 5 seconds), therefore it should be administered via bolus injection followed by an immediate bolus of saline (rapid push and flush). A 12-lead ECG should be obtained before and after conversion, if possible, and a rhythm strip should be continuously run during attempted conversion.  External pacing equipment should be available since some patients go into sinus arrest following administration of adenosine .

If adenosine initially fails to convert the SVT , but the patient is hemodynamically stable, they may be started on a medication such as  propranolol, digoxin or verapamil  (digoxin should be avoided in WPW, verapamil should be avoided in infants) followed by a repeat dose of adenosine. 

3. What medicine used to treat Supraventricular tachycardia(SVT) is contraindicated specifically in Wolff-Parkinson-White syndrome(WPW)?  Case 11  

Ans:    Digoxin .

** In the setting of WPW, digoxin can facilitate impulse conduction via accessory pathway and increase risk for ventricular arrhythmias (i.e., ventricular fibrillation).

Case 11  |  Clinical case Studies  |  Guidelines  |  Home 

Case Study-12 Answers

1. acute management of af with hemodynamic instability     case 12.

Given his presentation of AF with hypotension and pulmonary edema, considering AF with hemodynamic instability, sinus rhythm must be rapidly restored by synchronised electrical cardioversion. After achieving normal sinus rhythm, consider anti-coagulation as the patient is at risk for thromboembolism. Anticoagulation must be initiated and should be continued for at least 4 weeks.

(The decision to continue anticoagulation beyond the initial 4 weeks is based on the long-term risk for thromboembolism associated with nonvalvular AF. This risk is estimated by determining his CHA 2 DS 2 -VASc score, which is calculated by assigning points for congestive heart failure (1 point), hypertension (1 point), age (1 point for 65-74 years and 2 points for >75), diabetes (1 point), stroke, TIA, or thromboembolism (2 points), vascular disease defined as history of myocardial infarction, peripheral vascular disease, or aortic atherosclerosis (1 point), and female gender (1 point) )

2. CHA 2 DS 2 -VASc score (estimating risk of stroke & thromboembolism):    3 points       Case 12

Stroke risk was 3.2% per year in >90000 patients ( the Swedish atrial fibrillation cohort study) and 4.6%risk of stroke/TIA/systemic embolism.

  HAS-BLED score ( for estimating major bleeding risk) : 2 points

Risk was 4.1% in one validation study (Lip 2011) and 1.88 bleeds per 100 patient-years in another validation study      (Pisters 2010).Anticoagulation can be considered, however patient does have moderate risk for major bleeding (~2/100 patient-years).

Case 12 | Clinical Case Studies | Guidelines | Home

Case Study-13 Answers

1. based on current evidence, which of the following would be the most appropriate    recommendation regarding his asthma medication regimen   case study-13.

A. Maintain current medication regimen; no adjustment is indicated

B. Discontinue the inhaled corticosteroid; maintain on an inhaled beta-agonist as needed

C. Decrease the inhaled corticosteroid to 1 puff daily

D. Discontinue the inhaled corticosteroid; start a leukotriene modifier at bedtime

E. Discontinue the inhaled corticosteroid; start low-dose inhaled corticosteroid/long-acting beta-agonist, 1 inhalation at bedtime

ANS:  correct answer = option E

EXPLANATION : The goal of asthma therapy is to minimize risk and maintain asthma control with the least amount of medication (1).  In patients with mild persistent asthma, recent studies have demonstrated several options for “step-down therapy.”  The American Lung Association Asthma Clinical Research Centers network study found that patients who stepped down from twice daily low-dose fluticasone to once daily combination therapy with fluticasone/salmeterol had equivalent asthma control scores, FEV 1 , and frequency of exacerbations compared with continued therapy with twice daily fluticasone (2).  Once-daily montelukast demonstrated a slightly higher treatment failure compared with either of the regimens containing inhaled steroids.  Despite the slight increase in treatment failure with montelukast, each of the treatment groups had equivalent symptom-free days and rates of clinically significant asthma exacerbations.  Thus, while either regimen would be appropriate, stepping down to once-daily combination therapy with fluticasone/salmeterol appears to be more beneficial.

Recent studies also suggest that those with mild persistent asthma taking inhaled corticosteroids in combination with either a long-acting beta-agonist or a short-acting beta-agonist when symptomatic, had no increase in adverse outcomes compared with those taking scheduled daily inhaled doses.  Boushey et al. (3) compared patients with mild persistent asthma using twice-daily budesonide versus twice-daily zafirlukast verses placebo. All three groups used budesonide as-needed following a symptom-based action plan. The study found that in comparison with patients on a daily controller (budesonide or zafikulast), participants using only as-needed budesonide had no significant difference in morning peak expiratory flow, postbronchodilator FEV 1 , quality of life, or frequency of asthma exacerbations. Results of this study raise the possibility of treating mild persistent asthmatics with as-needed inhaled corticosteroids. More recently, Papi et al. (4) found as-needed use of an inhaler containing both beclomethasone and albuterol for symptom relief was associated with fewer exacerbations and higher morning peak flow readings than using an inhaler with albuterol alone.  The morning peak flow readings in the as-needed combination beclomethasone/albuterol group was equivalent to those taking scheduled daily doses of beclomethasone alone, or scheduled daily doses of beclomethasone/albuterol combined.  The combination of an inhaled steroid and a short-acting beta-agonist in a single inhaler is not currently available in the United States.

In the mild persistent asthmatic there is now strong evidence to support multiple treatment approaches which provide good asthma control.  Matching the drug regimen with the patient’s preferences, lifestyle, comorbidities, and financial limitations will help ensure drug adherence and maintain asthma control.

2. Which of the following should be done routinely with each follow-up visit?   Case Study-13

ANS: correct answer = option D

EXPLANATION: Assessing inhaler technique at each office visit allows the provider an opportunity to assess compliance, reinforce proper use, and identify motor or physical limitations affecting technique (8).

When studied, only approximately 25% of patients are able to properly demonstrate use of a meter dose inhaler when asked.  The remaining 75% improved with specific instruction and practice which reinforces the need to incorporate proper inhaler use during the office visit (9,10).  The use of a spacer significantly improves accuracy and dose delivery, particularly in patients with poor coordination skills (9,10).

Assessing patient adherence is best approached with a non-judgmental attitude.  Adherence to inhaled corticosteroids is estimated at < 50% (11).  Causes of nonadherence are multifactorial but may be improved by providing asthma education, encouraging self management through use of an asthma action plan, and facilitating open communication (11).  Financial barriers often transcend all other efforts to improve adherence and must be taken into account when prescribing asthma therapy (11).

Methacholine challenge testing is useful to demonstrate airway hyperresponsiveness in those with normal spirometry and a suspicion of asthma, but is not recommended as a serial procedure.  Biomarkers for inflammation such as eosinophils or nitric oxide are being investigated in clinical trials but currently have no indication in routine asthma care (1).  Peak flow monitoring is useful for long-term home assessment of asthma control and medication response, but is not indicated for regular office assessment or diagnostic purposes (1).

3. What findings would suggest that the patient requires a step-up in asthma medication?   Case Study-13

A. Two or more night time awakenings per month due to Asthma.

B. Two or more interruptions in daytime activities per month due to asthma

ANS:  correct answer = option A

EXPLANATION: Inadequate asthma control and a need for step-up therapy is based on two or more daytime symptoms per week, two or more nighttime symptoms per month, interference with activities of daily living, use of short-acting beta-agonist > 2 days/week (excluding use for prevention of exercise-induced bronchospasm),or peak flow or FEV 1  <80% predicted/personal best (1). 

Asthma symptoms should be assessed at each office visit to determine asthma control.  Validated self-assessment tools such as the Asthma Control Test (ACT), Asthma Therapy Assessment Questionnaire (ATAQ), or Asthma Control Questionnaire (ACQ) can facilitate consistent measurement and documentation of asthma symptoms during office visits (1, 8).  All asthmatics are at risk for a severe asthma attack regardless of their asthma classification; therefore, providers are encouraged to teach patients to recognize symptoms of inadequate asthma control and provide them with specific instructions for adjusting their medications or seeking medical care (1)

4. The patient was provided with an asthma action plan to follow at home. Which component of the asthma action plan is considered the most critical element for improving asthma outcomes?   Case Study-13

ANS:  correct answer = option D

EXPLANATION : All of the elements are important components of an asthma action plan. However, Gibson and Powell (5) found a 40% reduction in hospital admissions and a 20% reduction in emergency room visits when the plan contained personalized instructions regarding the medications to add, criteria for adding the medication, duration of use, and when to seek medical help when patients are symptomatic. An asthma action plan serves as a patient guide for early recognition and treatment of an exacerbation. Treatment guidelines may be based on symptoms, peak flow readings, or both. When peak flow readings are used, personal-best readings were consistently associated with improved health outcomes compared with percentage-predicted readings (5).

5. How often is spirometry testing recommended if the previous readings are normal and the patient’s asthma is well controlled?   Case Study-13

EXPLANATION: Spirometry is a simple test that can be performed in-office and can be used to assist the provider in determining the degree of airway obstruction (1, 6).  There are no widely accepted data correlating frequency of spirometry with clinical outcomes in asthmatics, thus one must rely on expert opinion and individual patient needs.  Spirometry is recommended during the initial evaluation after treatment is initiated and the patient’s symptoms have stabilized during periods of progressive or prolonged loss of asthma control and at least every 1-2 years (1).

When spirometry is used to diagnose or confirm asthma, testing must include pre- and post-bronchodilator readings (1).  A change in FEV 1  of >200 ml  and  ≥ 12% from the baseline measure following the administration of a short-acting bronchodilator is indicative of significant airway reversibility which has been shown to correlate with airway inflammation (7).  

The Expert Panel (1) classifies asthma severity by FEV 1 , FEV 1 /FVC, short-acting beta-agonist use, or frequency of asthma symptoms.   Parameters are measured at baseline with asthma severity determined by the worse parameter, e.g., daily symptoms with normal FEV 1  is classified as moderate persistent asthma.  Correct identification of asthma severity guides the provider in choosing the appropriate type and amount of therapy

REFERENCES :

  • Expert Panel Report 3 (EPR 3). Guidelines for the Diagnosis and Management of Asthma. Bethesda, Md: National Institutes of Health; 2007. NIH Publication No. 08-4051.
  • The American Lung Association Asthma Clinical Research Centers. Randomized comparison of strategies for reducing treatment in mild persistent asthma. N Engl J Med 2007;356:2027-2039.
  • Boushey HA, Sorkness CA, King TS, et al. Daily versus as-needed corticosteroids for mild persistent asthma. N Engl J Med 2005;352:1519-1528.
  • Papi A, Giorgio GW, Maestrelli P, et al. Rescue use of beclomethasone and albuterol in a single inhaler for mild asthma. N Engl J Med 2007;356:2040-2052.
  • Gibson PG, Powell H. Written action plans for asthma: an evidence-based review of the key components. Thorax 2007;59:94-99.
  • Miller MR, Hankinson J, Brusasco V, et al. Series ATS/ERS Task Force: Standardization of lung function testing. Eur Respir J 2005;26:319-338.
  • Pellegrino R, Viegi G, Brusasco V, et al. Interpretative strategies for lung function tests. N Engl J Med 2005;26:948-968.
  • Global Initiative for Asthma. Pocket guide for asthma management and prevention. Bethesda, Md: National Institutes of Health; 2006.
  • Giraud V, Roche N. Misuse of corticosteroid metered-dose inhaler is associated with decreased asthma stability. Eur Respir J 2002;19(2):246-251.
  • Johnson DH, Robart P. Inhaler technique of outpatients in the home. Respir Care 2000;45(10):1182-1187.
  • Elliott RA. Poor adherence to anti-inflammatory medication in asthma reasons, challenges, and strategies for improved disease management. Dis Manage Health Outcomes 2006;14(4):223-233.Top of Form

Case Study-14 Answers

1. what emergency measures are indicated   case study- 14.

Ans: In this case patient demonstrates the destabilizing effects of a respiratory infection on asthma, and her mother’s comments demonstrate the common (and dangerous) phobia about overuse of bronchodilator or steroid inhalers. The patient has signs of imminent respiratory failure, including her refusal to lie down, her fear and her tachycardia, which cannot be attributed to her minimal treatment with albuterol.

#Critically important immediate steps are

  • to administer high flow oxygen and to start albuterol by nebulisation.
  • Adding ipratropium to the nebulized solution is recommended.
  • A corticosteroid (0.5 – 1.0 mg/kg of methylprednisolone) should be administered intravenously.

#Alert the intensive care unit, because a patient with severe bronchospasm who tires can slip into respiratory failure quickly, and intubation can be difficult.

2. How should her long-term management be altered?    Case Study- 14

Ans: Presuming this patient recovers, she needs adjustments to her therapy before discharge. The strongest predictor of severe attacks of asthma is their occurrence in the past. Thus, this patient therapy needs to be stepped up to a higher level, like a high dose inhaled corticosteroid in combination with a long acting beta agonist. Both the patient and her  parents need instructions on the importance of regular adherence to therapy, with reassurance that it can be stepped down to a lower dose of inhaled corticosteroid( although still in combination with long acting beta agonist) once her condition stabilizes.

They also need instruction on an action plan for managing severe symptoms. This can be as simple as advising that if the patient has a severe, frightening attack, she can take up to four puffs of albuterol every 15 minutes, but if the first treatment doesn’t bring significant relief, she should take next four puffs while on her way to an emergency department or urgent care clinic.

She should also be given a prescription for prednisone, with instructions to take 40-60 mg orally for severe attacks, but not to wait for it to take effect if she remains severely short of breath even after albuterol inhalations.

# Asthma is a chronic disease, and a good care requires close follow up and creation of a provider-patient partnership for optimal management. If she has had several previous exacerbations, she should be a candidate considered for monoclonal anti-IgE antibody therapy with omalizumab, which effectively reduces the rate of asthma exacerbations _ even those associated with viral respiratory infection.

Reference : Text book Bertram G. katzung Basic and clinical pharmacology 14 th edition page number 365.

Go to Case Study- 14

For more Case Studies | Guidelines

All subject, PharmD books in pdf free download

Case Study-15 Answers

1. what clinical features and risk factors of copd does glenda exhibit  what grade of severity does glenda’s copd fall into    case study -15.

Ans: Glenda presents with a clinical scenario suggestive of COPD based on her age, smoking history, weight, frequent chest infections and gradual worsening of respiratory symptoms – breathlessness on exertion, a reduction in exercise tolerance, chronic cough and regular sputum production. Working in a fabric factory is also one of the identified occupational risk factors for COPD. Glenda is considered to have moderate COPD based on her spirometry results and breathlessness score.

(refer NICE classification of severity of airflow obstruction)

2. What initial treatment would you recommend for glenda?     Case Study -15

Ans: Start a short acting beta agonist such as salbutamol or a short acting muscarinic antagonist, such as ipratropium bromide, to alleviate symptoms as required. Short acting bronchodilators should be used as needed; their onset of action ranges from approximately five minutes (beta2 agonists) to 30 minutes (muscarinic antagonists) and effects lasts for between 3 and 6 hours.

Refer: NICE clinical guideline for COPD

3. Glenda continues to report that her breathlessness is getting worse. Her medical research council dyspnea score is now four and in the last few days she has been producing more sputum than usual. Her sputum has turned a yellow green colour. What does these changes indicate & what treatment would you recommend?     Case Study -15

Ans: Glenda’s symptoms suggest that she is experiencing exacerbations of COPD. She must be prescribed an antibiotic for 5 days at a therapeutic dose with Oral prednisolone 30 mg every morning for 7-14 days.

(Systemic corticosteroids are beneficial in the management of exacerbations of COPD. They shorten recovery time and improving lung function (FEV10 and hypoxemia (PaO2).  Antibiotics should be used to treat exacerbations of COPD associated with a history of more purulent sputum. Initial empirical treatment should be amoxicillin, doxycycline or clarithromycin depending on a local resistance pattern.

Go to Case Study -15

Explore more Case Studies | Guidelines

Case Study-16 Answers

1. how the sign, symptoms & pathophysiology of copd, relates to the patient   case study-16.

Ans:   Clinical signs and symptoms of COPD, the pathophysiology and how this relates to the patient.

Airflow obstruction, due to airway and parenchymal damage as a result of chronic inflammation that is progressive, not fully reversible. Shortness of breath
Due to increased hydrogen ion concentrations and possible increased metabolic rate stimulating the respiratory centre. RR 20 per min
Airway narrowing, mucosal damage, oedema of airway and increased sputum production increases the ventilation/ perfusion mismatch. PCO2 9.672 kPa
The duration of expiration is insufficient to allow the lungs to deflate (due to airway resistance or increased breathing rate).   Lungs hyperinflated

2. Comment on the current drug therapy and describe the role of O2 in this patient.     Case Study-16

 
10 mg od Need to review the duration and need. Will need bisphosphonate for bone protection as osteoporotic
500 micrograms BD Check which device? Accuhaler or Diskhaler. Patient not taking a long acting beta agonist, so why is she on an inhaled corticosteroid?
75 mg od NICE guidelines post MI
1 mg od Loop diuretic, why? BP is low, what evidence of heart failure is there?
2 qds Salbutamol 2.5 mg, ipratropium 500 micrograms in 2.5 mls. Is there a need for regular short acting beta2-agonist? Change to long acting. Why use nebulisers?
5 mg od Low dose for heart failure, or is it for high BP, but patient has a low BP!!! On admission. So it should be changed to other antihypertensives.
200 mg bd Theophylline normally used after a trial of short and long-acting bronchodilators. Needs plasma levels monitored. Increased risk of low potassium when given with prednisolone and bumetanide.
2 tablets nocte Review the patient’s intake and need for laxatives.
5 mg od INR will need to be checked, 3–6 months duration. Care with drug interactions, anticoagulant effect may be affected by prednisolone, and aspirin. Patients Hb is already at the lower end of normal.
7.5 mg nocte Normally only used for 4 weeks, as a hypnotic.
50 mg prn Need to change to different pain control as high risk of bleeding when on warfarin and aspirin. Must not be taken as required.
2 L nasal specs Nasal specs: difficult to predict the amount of oxygen inspired Need to investigate LTOT for home use.

Role of oxygen:

As this patient has a PaO2 of less than 7.3 kPa and oxygen saturation of arterial blood of less than 90%, she is eligible for Long-Term Oxygen Therapy (LTOT). She will need to use oxygen at least 15 hours a day and needs to be counselled on the importance of smoking cessation. Ambulatory and short-burst oxygen therapy should also be considered, as per NICE guidelines (NICE, 2004).

3. What are the social issues in treating this patient at home?    Case Study-16

Ans: Patients may be anxious and refuse such support. The cultural and social setting of the patient needs to be taken into account. As patients lose their mobility and increase dependence on others for help with day-to-day living, anxiety increases. Patients can become hesitant to seek help because of the perception that their condition was self-inflicted. Poor populations tend to have a higher risk of developing COPD, other factors include poor nutrition, crowding, exposure to pollutants, poor access to healthcare and early respiratory infections. Some evidence suggests women are more susceptible to COPD development than men. A multidisciplinary team should be involved in the support of the patient at home. Additional use can be made of nebulisers, compressors, oxygen, visiting respiratory nurses and increased social service input. The patient remains under care of the hospital but the GP is made aware of the extra support. Health status is better in home-treated patients. COPD is linked with other co-morbid conditions. Patients are more likely to have ischaemic heart disease, pneumonia and diabetes, making treatment more complicated and requiring a holistic approach to care.

This patient demonstrates five co-morbidities. These in turn impact on the medication load she has to cope with, so concordance is important.

Book: Pharmacy case studies by Rebekah     Raymond

Go to Case Study-16

Explore more Case Studie s | Guidelines

PharmD 1st year, 2nd year, 3rd year, 4th year books pdf free download

Case Study-17 Answers

1. Why should women of childbearing age be offered advice about pregnancy?  Case Study-17

Ans:  Mrs Jaya or other women like her should have been offered preconception advice prior to becoming pregnant because glucose control needs to be optimal to reduce the risks of miscarriage, congenital malformation, stillbirth and neonatal death associated with diabetes in early pregnancy. Preconception advice should also include information for the patient on how diabetes affects pregnancy and how pregnancy affects diabetes, what dietary supplements to take and advice on diabetes-related medicines that are unsafe to take during pregnancy.

2.  Was she taking appropriate dietary supplements prior to conception?     Case Study-17

Ans:  Mrs Jaya was taking the appropriate dietary supplement; however, the recommended dose for women with diabetes is 5 mg daily, rather than 400 μcg daily. The 5-mg strength tablets are available on prescription.

3.  What advice should she be given with respect to her regular medication?     Case Study-17

Ans:  Mrs Jaya should have been advised to stop her ramipril and simvastatin since both have been associated with an increased risk of birth defects.

Case Study - 18 Answers

You can get Case -18th answers with explanation by this link.

Go back to Case Study-18 

Case Study - 19 Answers

Answers :        (CORRECTED)

1.     What is the likely diagnosis? Main differential diagnosis?          Case Study-19

Ans:   Diagnosis: Hypothyroidism.

The differential diagnosis is extensive.

However, Main Differential Diagnosis : Depression and Hypothyroidism

** Fatigue is a very common symptom of both physical and mental illness. The differential diagnosis is extensive and includes cancer, depression, anaemia, renal failure and endocrine diseases. He has a past history of depression, but currently has no obvious triggers for a further episode of depression. He is not waking early in the morning or having difficulty getting to sleep, which are common biological symptoms of severe depression.

There are a number of clues in this case to the diagnosis of hypothyroidism . Insidious onset of fatigue, difficulty concentrating, increased somnolence, constipation and weight gain are features of hypothyroidism. As in this case there may be a family or past medical history of other autoimmune diseases such as type 1 diabetes mellitus, vitiligo or Addison’s disease. Hypothyroidism typically presents in the fifth or sixth decade, and is about five times more common in women than men. Obstructive sleep apnoea is associated with hypothyroidism and may contribute to daytime sleepiness and fatigue. On examination the facial appearances and bradycardia are consistent with the diagnosis

2.     How would you further manage this patient?                        Case Study-19

Ans:    For the management of the patient in this case, patient should be advised for Thyroid Function Test . Level of TSH, T ₃ , T ₄ will lead to decide the accurate therapy for him.

**However, a starting dose of levothyroxine 50-70 μ g/day will be sufficient. Clinical benefits begin in 3-5 days.

Response is measured clinically and biochemically by the return of TSH to the normal range.

*Elderly patients or those with coronary heart disease should be started cautiously on T4 because of the risk of precipitating myocardial ischaemia.

The most common cause of hypothyroidism is autoimmune thyroiditis and the patient should have thyroid autoantibodies assayed.

Patients should be advised to avoid or do heavy physical labor with caution (hypotonia may be there sometimes).

·         No specific diets are required for hypothyroidism . WHO recommends a daily dietary iodine intake of 150 μ g for adults.

CORRECTION :                    Case Study-19

1.      Starting Dose of Levothyroxine should be 1.6 – 1.8 mcg/kg/day of lean body mass.

2.       If started with levothyroxine, then, soyabean, walnuts, dietary fiber, calcium fortified juices should be avoided, if possible. When levothyroxine is given during continuous enteral nutrition for more than 7 days, the tube should be interrupted for at least one hour before and one hour after the dose of levothyroxine.  

Ref-2: Recommendations for the use of medications with continuous enteral nutrition , “ American journal of health-system pharmacy: AJHP: official journal of the American Society of Health-System Pharmacists ”  

Case Study - 20 Answers

1.     What is the likely diagnosis in this case?  & Are the Lab finding s clinically justified?    Case Study- 20

Ans: Diagnosis: Hypothyroidism with Anemia.

           !  Ix Hashimoto Thyroiditis .

Yes , the lab findings are clinically justified.

·         High TSH & low levels of T4 suggestive of Hypothyroidism.

·         Stool occult blood positive: Reason to justify blood loss, decrease in levels of RBC, Haemglobin.

·         High RDW with normal MCV indication for anemia further evalualtion needed for iron/B12/FA deficiency/anemia of chronic disease.

·         Decrease in reticulocyte:(borderline to decrease) indication for iron deficiency/anemia of chronic disease.

·         USG: indicates Thyroditis further lab values needs for antibody testing(thyroxine peroxidase).

2.     What are the Pharmacist Intervention points in this case?          Case Study- 20

Ans:   Overdose: According to TSH value,  Tab Thyronorm 75 mcg should have been  started instead of Tab Thyronorm 25mg on the basis of wt. of patient (1.6-1.8mcg/kg/day).

Inappropriate Dose:  There is no electrolytic imbalance seen therefore, isotonic (0.90% NS) should have been administered.

Untreated Indication: No medication for headache has been prescribed (antipyretic SOS can be prescribed)

Drug- Drug Interaction: Moderate Interaction

( Pantoprazole : Iron supplements ), ( Levothyroxine: Pantoprazole ), [ Levothyroxin: Iron Supplements ( Theoritical )]

Drug-Food Interaction: Is seen with soyabean products

3.     Patient counselling regarding drug & disease?          Case Study- 20

Ans:    Disease: It is a disorder in which under activity of Thyroid glands occur(in local language).

Signs & Symptoms : related to slow metabolism (functioning) of the body .

·         Fatigueness, dry skin, constipation, feeling cold, slow heart rate, weight gain

     *Avoid soyabean & its products; cruciferous vegetables.

·         Don’t chew the tablet

·         Don’t split the tablet

·         Tightly place the cap of container after use

·         In the case of missed dose, Don’t take double dose; If it is time for next dose, take the single dose only.

·         Don’t take the dose immediately, when you recall anytime, after you missed the dose

·         If case, vomiting occurs immediately after taking the dose, consume it after sometime. If another episode occurs, consult your physician. Don’t stop taking the medication [this further may lead to untreated condition, resulting in myxedema (emergency condition)]

ü The Improvement of symptoms may not be evident for several weeks after the starting of the therapy. So, she should be counselled for proper medication adherence.

Drugs Counselling Points:

·         Take tab Thyronorm on an empty stomach (30min to 1 hour prior to breakfast) once daily.

·         Take tab Thyronorm 4 hours apart from Pantoprazole and Febac XT, Fdson MP Forte.

·         Take tab Fdson MP Forte once daily after lunch

·         Take one tab Febac XT after breakfast and another after dinner.

·         For Tab Fdson MP Forte & Febac XT, you may observe metallic taste in the mouth. Therefore, don’t stop taking the medication.

·         Take Tab Pantoprazole once a day 1 hour before lunch.

4.     Write the Generic Names of above advised drugs.      Case Study- 20

Ans:                                                                                         Generic

Pantoprazole                                                  Pantoprazole

            Optineuron                                                   Vit. B-Complex

            Fdson MP Forte                                          Folic Acid 5mg + Methylcobalamin

        1500mcg + Pyridoxine 20mg

            Febac XT                                                       Ferrous Ascorbat 100mg + Folic Acid

          1.5mg + Zinc Sulphate 22.5 mg

            Thyronorm                                                  Levothyroxine Sodium

Case Study - 21 Answers

Q1 . What is the likely diagnosis ?          Case Study-21

Ans: Diagnosis : Grave disease in pregnancy with impending thyroid storm and

        IUFD at 23-24 weeks gestation.

Q2. Was the Treatment given to the patient justified according to clinical condition?            Case Study-21

Ans: The dosing and the regimen advised to the patient is according to the prescribed guidelines and in the limit according to the lab reports.

***Methimazole 30mg BD,                     recommended 60-80mg/day (impending

thyroid storm cond. )

*** Lugol solution can be given 5 drops 2 times a day, but should not be more than 1 week.

*** The combination of propranolol 40 mg every 6 hours with iodide usually results in clinical improvement within 2 to 7 days.

Q3. Widely used anti-thyroid drugs in pregnant women with hyperthyroidism?              Case Study-21

Ans: Propylthiouracil (PTU) and methimazole

Q4. What should be the Patient Counselling points?                  Case Study-21

·         Pt should be instructed to report any new symptoms occurring.

·         She should be counselled about the necessity of medication adherence.

·         Next optimal time to conceive is once a euthyroid state is reached, should consult the practitioner for the same.

·         Pre- pregnancy counselling for all patients with hyperthyroidism or a history of hyperthyroidism is imperative and use of contraception until the disease is controlled.

·         Prior to conception, ablative therapy (radioiodine or surgery) or medical therapy may be offered when the thyroid gland is overactive.

·         May experience the previous symptoms of hyperthyroidism along with new symptoms, consult the practitioner

·         Avoid caffeine; stress reduction therapy should be suggested to relieve the symptoms of anxiety, nervousness, poor conc. May occur due to the miscarriage.

·         It is normal to women to experience some Vaginal bleeding (light, menstrual-like bleeding) for several weeks after an abortion.

·         Some pain is normal after an abortion, as the uterus is contracting.

·         In this case, she should be specially suggested of the recommended interval to next interval to next pregnancy is at least 6 months (checking the hyperthyroid condition)

·         She should be counselled with care and one to one interactions and the family members should be counselled to maintain a better social environment for her.

ADDITIONAL BASIC POINTS TO CONSIDER IN COUNSELLING:

·         If case, vomiting occurs immediately after taking the dose, consume it after sometime. If another episode occurs, consult your physician. Don’t stop taking the medication [this further may lead to untreated condition , resulting in myxedema (emergency condition)]

Case Study - 22 Answers

1.     Are there any drug-drug interactions?          Case Study – 22

Ans:    Major:

Diltiazem + Propranolol ::  Increases the toxicity of other by unspecified mechanism. Can increase the risk of bradycardia.

**In this case, alternative drugs also have the major/serious interaction (alternatively: closely monitor)

            Moderate:

            Propranolol + Furosemide :: Decreases serum potassium.

            Propranolol + Diltiazem  ::  Both increase anti-hypertensive channel blocking.

            ** Monitor Closely                              *Minor interactions are also there.

2.     Is the dosing of medication for hyperthyroidism, according to standard treatment guidelines?          Case Study – 22

Ans:    Dose of carbimazole in hyperthyroidism; depends on the

FT4 levels. If the levels are raised 2 times higher than the upper limit of normal values then the dose is : 10-20 mg. So, correct in this case.

Given in split doses as duration of action is less than 24 hours, Twice a day is acceptable.

If HR >90 BPM Beta adrenergic blocker are considered, Propranolol dose :10-40 mg 3-4 times / day

For atrial fibrillation: diltiazem IV 15-20 mg is considered.

Maintenance dose is: 5-10 mg Carbimazole

3.     What should be Patient counselling in this case, regarding drug & disease?                Case Study – 22

Ans:  Drugs:

·         If case, vomiting occurs immediately after taking the dose, consume it after sometime. If another episode occurs, consult your physician. Don’t stop taking the medication.

            Disease:

Hyperthyroidism

·         It is a disorder in which thyroid hormones level are increased. So, they may cause some alarming symptoms related to increase in the metabolism of the body such as wt loss, over active bowel movement, heat intolerance, tremors, palpitations, nervousness, fatigue , weakness.

·         Patient should be advised to follow medication adherence, Otherwise may increase the risk of cardiovascular problems.

·         Pt should be informed that improvement in the symptoms may be seen in 3-4 weeks.

·         A sightly elevated liver function test are commonly seen in some hyperthyroidism pts.

·         Patient should be informed of side effects of ATDs and the necessity of informing the physician promptly if they should develop pruritic rash, jaundice, acolic stools or dark urine, arthralgias, abdominal pain, nausea, fatigue, fever, or pharyngitis. Preferably, this information should be in writing.

·         Before starting ATDs and at each subsequent visit, the patient should be alerted to stop the medication immediately and call their physician if there are symptoms suggestive of agranulocytosis or hepatic injury.

·         Patient should be informed about the condition like, Sometimes, after apparently successful treatment, the condition may return, and further treatment may be needed.

Hemorrhoids/ Anal fissure

·         May be advised to sit over Hot water bag/ tub/brick not more than 15 min.

Note: As much as one can tolerate

·          Keep a check on your regular bowel movement, avoid any straining.

·          Pt should be advised to Eat less, oily less spicy easily digested food items

Case Study -23 Answers

1.     What should be the further investigation ?                          Case Study- 23

Ans:     Females with primary ovarian insufficiency-related estrogen deficiency are

at risk of osteopenia, osteoporosis, and fracture, especially if hypoestrogenism occurs early in life and before accrual of peak bone mass. Therefore, Dual-energy X-ray absorptiometry has been recommended for the evaluation of bone mineral density in women diagnosed with primary ovarian insufficiency.

Flow-mediated brachial artery diameter for endothelial dysfunction as there are cardiovascular risks .

[** The woman has evidence of hypogonadotrophic hypogonadism- she has low oestradiol

levels associated with low gonadotrophin stimulation from the anterior pituitary. This

may be due to various pituitary or hypothalamic causes, but in this case clearly relates to

anorexia nervosa and possibly excessive exercise . The raised prolactin is consistent with

stress and does not need to be investigated further. At a BMI below 18 kg/m2, menstru-

ation tends to cease, returning once the BMI increases again.]

-Furthermore, CBC, Liver & renal function test should be

  monitored.

-Cognition, Mood, and Psychosocial Functioning

-Vasomotor Symptoms and Quality of Life

-Also include USG abdomen if needed!

2.     How, this patient would be managed ?                Case Study- 23

Ans: The combined oral contraceptive pill should be prescribed, which will prevent osteoporosis and bring on periods. However, this anorexia and primary ovarian insufficiency condition can be refractory to treatment, albeit pharmacologically induced.

Recommended dose:

 

1-2 mg micronized

17 beta-estradiol (oral)

2.5mg medroxyprogesterone acetate daily (oral)

 

10mg medroxyprogesterone acetate daily (oral) for 12 days each month

0.625-1.25 mg conjugated equine estrogen (oral)

100mg micronized progesterone daily (oral)

200mg micronized progesterone daily (oral) for 12 days each month

* One of the estrogen options to be combined with one of the progestogen options.

Also, vitamin supplementation and calcium should be started (if evident)

3.     What Patient counselling points should be included in this case?      Case Study- 23

Ans: Following Counselling points will help in effective recovering:

·         Encouraging the woman to eat a more normal diet and to avoid exercising is the ideal management.

·         Explanation that her periods will return if she increases her BMI may possibly encourage her to put on weight.

·         Proper counselling should also be given to her parents, to help in stress reduction.

·         She should be advised to consult dietician & nutritionist also, to maintain regular food habits with minimal or moderate physical exercise.

·         Combined oral contraceptive pills are to be taken daily at approximately the same time each day.

·         Avoid taking them greater than 24 hours apart as this could affect efficacy. 

·         When you initiate the contraceptive pills you are not protected from pregnancy prevention in the first 7 days and an alternative method of birth control is recommended during this time period (not required in this case)

·         If she, miss a tablet, just take the missed tablet as soon as she remembers and the next tablet at the usual time (taking 2 tablets in 1 day). 

·         If she, miss 2 tablets in a row in the first or second week, then, take 2 tablets the day she remembers and 2 tablets the next day, then resume 1 per day.

·         The most common adverse effect of combined oral contraceptive pills is break through bleeding. 

·         She may feel of nausea, headaches, abdominal cramping, breast tenderness, and an increase in vaginal discharge or decreased libido. 

·         Nausea can be avoided by taking the medication at night before sleep.

Ref: 1. https://www.acog.org/en/Clinical/Clinical%20Guidance/Committee%20Opinion/Articles/2017/05/Hormone%20Therapy%20in%20Primary%20Ovarian%20Insufficiency

2. Oral contraceptive pills, NCBI, National Library of Medicine,USA

Case Study -24 Answers

1.      What should be the further investigation ?                  Case Study – 24

Ans: Further recommended investigations are :

Lipid panel , 2 hr OGTT, questionnaire for Depression, USG Abdomen (transabdominal)

2.      How, this patient would be managed ?                      Case Study – 24

Ans : For acne: First line: hormonal contraception, and topical cream (benzoyl

         peroxide/ tretinoin/adapalene/ antibiotic cream like clindamycin.

            ** depending upon the staging nd grading of acne

Obesity: First line is Lifestyle Modifications

Insulin resistance: First line METFORMIN (Dose 1500-2250 mg twice daily )

Hirsutism: First line:  Hormonal contraception with or without anti androgen therapy

Menstrual irregularities : Clomiphene 50-100mg / day

3.      What Patient counselling points should be included in this case?                      Case Study – 24

·         Encouraging the woman to eat a more normal diet and to avoid leg exercising is the ideal management.

·         Explanation that how increased BMI can also affect the normal menstrual cycle.

·           For Acne: Wash the face twice daily with medicated face wash and can spill water during day time.

·         Adapalene is UV, light sensitive, to be used at night. And wash your face before stepping out. It should be instructed that cold temperatures or wind may also increase skin irritation during drug therapy.

·         Benzoyl peroxide : May cause dry skin and peeling, so avoid using higher concentration. Should be advised to use a test dose for over-the-counter (OTC) products due to potential hypersensitivity reactions. Apply small amount to skin for 3 days and if no discomfort occurs, use product as directed.

·         Clomiphene citrate: Drug may cause decreased visual acuity. Patient should be instructed to report visual symptoms, such as blurred vision. Advise patient to take drug exactly as ordered, as administration is timing-sensitive.

Case Study - 25 Answers

1.     What should be the further investigation for this patient ?              Case Study – 25

Ans : The following further investigation should be suggested for her:

                                    i.             Thyroid profile

                                  ii.              HbA1C test

                               iii.              Diurnal epinephrine test

2.     Give the provisional diagnosis for this case.                                             Case Study – 25

     Ans : Provisional Diagnosis: Menopausal syndrome

3.     How, this patient would be managed ?                                Case Study – 25

Ans : Following management plan is recommended for the patient:

Estradiol low dose transdermal patch is to be started with min possible dose

of : 0.025 mg with size 5 cm ² patch. (Duration : 3 weeks of therapy + 1 week off)

An oral progestin is also added for only 10-14 days every month this will promote sleep.

( Dose : 10mg dydrogesterone OR Micronised progesterone capsule 100mg Frequency : at night 1 tab)

For glycine and glutamate:  glutathione 20mg is recommended Or NAC(n-Acetyl cysteine) if needed

For PEA:  vit B6 is advised daily, at lunch time.. 1tab, MVBC most preferably

4.     What Patient counselling points should be included in this case?                            Case Study – 25

Ans:    About disease:

  •     Give information to menopausal women and their family members (as appropriate) that includes:
  •     That a change in their menstrual cycle they may experience a variety of symptoms associated with menopause, including:

·         Vasomotor symptoms (for example, hot flushes and sweats)

·         Musculoskeletal symptoms (for example, joint and muscle pain)

·         Effects on mood (for example, low mood)

·         Urogenital symptoms (for example, vaginal dryness)

·         Sexual difficulties (for example, low sexual desire)

·         Benefits and risks of treatments for menopausal symptoms

·         Long-term health implications of menopause

·          Hormone replacement therapy (HRT) aids for Psychological symptoms , Consider HRT to alleviate low mood that arises as a result of the menopause

·         Non-pharmaceutical, for example cognitive behavioral therapy (CBT),  Consider CBT to alleviate low mood or anxiety that arise as a result of the menopause

About drugs: (general points)

Transdermal patch:

Application : It is to be applied to non hairy skin, below waist on upper quadrant of abdomen.

Replace the patch in every 3-4 days using a different site.

·         Gradually reducing HRT may limit recurrence of symptoms in the short term

·         Gradually reducing or immediately stopping HRT makes no difference to their symptoms in the longer term

**Long-term benefits and risks of hormone replacement therapy–

·          Venous thromboembolism:The risk of venous thromboembolism (VTE) is increased by oral HRT compared with baseline population risk

·          The risk of VTE associated with HRT is greater for oral than transdermal preparations

·          Consider transdermal rather than oral HRT for menopausal women who are at increased risk of VTE, including those with a BMI over 30 kg/m ²

·          Consider referring menopausal women at high risk of VTE (for example, those with a strong family history of VTE or a hereditary thrombophilia) to a haematologist for assessment before considering HRT

Lifestyle changes:

·         She should be suggested to exercise regularly

·         She can take cold water bath as required.

·         She should be counselled to avoid stress and enjoy day to day activity.

·         Advise her to deep her feet in mild-salted, tolerable cold water, to ease her in hot flashes.

·         Consult your gynecologist and ask her about vaginal wash products to prevent dryness, if needed.

·         isoflavones or black cohosh may relieve vasomotor symptoms advice to consume soy products, nuts raisins, legumes, sesame, peanuts, Meat, pork, chicken, egg , banana.

Note: Ensure that menopausal women and healthcare professionals involved in their care understand that there is no clear evidence for SSRIs or SNRIs to ease low mood in menopausal women who have not been diagnosed with depression. (NICE Guidelines)

1.        Indian Menopause Society ( https://indianmenopausesociety.org/prescription-writing-module-for-hormone-therapy/ )

2.        Essentials of medical Pharmacology, Tripathi.KD, 6 th edition.

3.        https://www.acc.org/latest-in-cardiology/ten-points-to-remember/2020/02/10/12/13/hormone-therapy-for-postmenopausal-women

4.        https://dailymed.nlm.nih.gov/dailymed/lookup.cfm?setid=c714974b-766f-42f2-a846-b0c1f5a60560

5.        https://www.jmidlifehealth.org/article.asp?issn=0976-7800;year=2013;volume=4;issue=2;spage=77;epage=106;aulast=Meeta%2C

Case Study -26 Answers

1.      How should the ovarian hormones she lacks be replaced?            Case Study- 26 

Ans: The patient should be advised to start daily transdermal estradiol (100

mcg/day) along with oral natural progesterone (200mg/day) for the last 12 days of each 28 day cycle. On this regimen, her symptoms should disappear and normal monthly uterine bleeding resume.

2.      What extra measures should she take for her osteoporosis while receiving treatment?              Case Study- 26 

Ans: She should  also be advised to get adequate exercise and increase her calcium

and vitamin D intake as treatment for her osteoporosis.

Clinical evidence:

Non-pharmacological approaches–

·         A balanced diet, adequate calcium and vitamin D intake, weight-bearing exercise, maintaining a healthy body weight and cessation of smoking and moderation of alcohol intake are primary goals in reducing fracture risk.

·         Calcium is essential for bone health, and there is evidence that calcium supplementation in older women reduces the risk of fracture.

·         The recommended nutritional intake (RNI) for calcium is 1000mg/day, and for vitamin D 800 IU/day 

**Higher calcium intake during growth and early adulthood is associated with higher peak bone mass.

**Based on recent concerns of a potential association between calcium supplement use and increased risk of myocardial infarction, calcium supplements should not be prescribed when dietary calcium intake is adequate (1000 – 1200 mg/day).

Case Study - 27 Answers

1.      What all investigations are needed for the further management?

·         Her 10-year risk of fracture FRAX is the risk assessment tools validated for use

·         She has been taking multivitamin with calcium so

·         Consider assessing serum 25-OH-D

·         dual-energy X-ray absorptiometry (DXA) and repeat DXA every 1 to 2 years until findings are stable

·         Additional tests for Clinical Identification of Vertebral Fracture :

2.      How this patient would be managed ?

·         Counsel patients to maintain adequate dietary intake

of calcium, to a total intake (including diet plus supplement, if needed) of 1,200 mg/day for women age ≥50 years

·         Pharmacologic therapy is strongly recommended for Patients with osteopenia or low bone mass and a history Of fragility fracture of the hip or spine

·          Pharmacologic therapy is strongly recommended for Patients with a T-score of −2.5 or lower in the spine, femoral neck, total hip, or 1/3 radius

Calcium: 1200mg/day is recommended so as per the patients need either 500mg twice daily OR 1000mg once daily is given with food items rich in calcium.. 

Vit D3: If serum25[OH]D deficiency is seen then according to individualize patient therapy is given as per this case: vit D3 5000IU / day for 8-12 weeks is recommended…. With the maintenance therapy of 1000-2000 IU/day

3.      How will you do the Patient counselling  in this case?

·         Counsel patients to maintain adequate dietary intake of calcium rich products.

·         Counsel patients to avoid or stop smoking.

·          Counsel patients to maintain an active lifestyle, including weight-bearing, balance, and resistance exercises

·          Provide counseling on reducing risk of falls, particularly among the elderly.

·         Daily sun bath is advised.

·          Consider referral for physical therapy, which may reduce discomfort, prevent falls, and improve quality of life.

·          avoiding use of tobacco And excessive use of alcohol;

·         This “bone healthy” lifestyle is Important for everyone, not only patients with osteopenia And osteoporosis.

·         Weight-bearing exercise includes walking, jogging, Tai Chi, stair climbing, and dancing, among other activities.

·         Muscle-strengthening exercise includes weight training and other resistive exercises.

·         Before initiating an exercise program in an individual with osteoporosis, a clinician’s evaluation is recommended.

·         Physical therapy plays an important role in the effort to mitigate sarcopenia and reduce risk of falls.

·         Measures for Prevention of Falls

·         Anchor rugs

·         Minimize clutter

·         Remove loose wires

·         Use nonskid mats

·         Install handrails in bathrooms, halls, and long stairways

·         Light hallways, stairwells, and entrances

·         Encourage patient to wear sturdy, low-heeled shoes

4.      Mention the indications for BMD testing.

·         All women 65 years of age or older

·         All postmenopausal women

·         With a history of fracture(s) without major trauma

·         With osteopenia identified radiographically

·         Starting or taking long-term systemic glucocorticoid therapy (≥3 months)

·         Other perimenopausal or postmenopausal women with risk factors for Osteoporosis if willing to consider pharmacologic interventions

·         Low body weight (<127 lb or body mass index <20 kg/m2)

·         Long-term systemic glucocorticoid therapy (≥3 months)

·         Family history of osteoporotic fracture

·         Early menopause

·         Current smoking

·         Excessive consumption of alcohol

·         Secondary osteoporosis

Case Study-28 Answers

1. what is the significance of t- score            case study – 28.

• The T-score on your bone density report shows how much your bone mass differs from the bone mass of an average healthy adult.

• DEXA accomplishes with only one-tenth of the radiation exposure of a standard chest x-ray and is considered the gold standard for osteoporosis screening

• Standard X-rays may show weakened bones. But at the point when bone weakness can be seen on standard X-rays, it may be too far advanced to treat. Bone densitometry testing can find decreasing bone density and strength at a much earlier stage when treatment can be beneficial

• The World Health Organization has established the following classification system for bone density:

1. If your T-score is –1 or greater: your bone density is considered normal.

2. If your T-score is between –1 and –2.5: you have low bone density, known as osteopenia, but not osteoporosis.

3. If your T-score is –2.5 or less: you have osteoporosis, even if you haven’t yet broken a bone.

2. How this patient would be managed ?                Case Study – 28

• A daily calcium intake of 1000 mg for men under 70 years is advised(only of dietary calcium is not sufficient enough). So 500 mg twice daily once on morning and other at night is ideal.

• Vit D starting from 800 IU/day (once daily OR 400 IU twice daily)

3. How will you do the Patient counselling in this case?       Case Study – 28

• to not involve yourself in smoking again

• Quit drinking alcohol

• Regular exercise helps make your bones stronger. • physical activity is reviewed and prescribed by an exercise professional, such as a physiotherapist or exercise physiologist. This is because some activities like jumping, running and twisting can be hazardous to weaker bones, particularly if you have had a fracture. • Other forms of exercise such as strength training can actually benefit the bones

• Foods high in calcium include milk products, leafy green vegetables, sardines, salmon, tofu, and almonds.

• You can get enough vitamin D from being in direct sunlight for 10 to 15 minutes, two or three times weekly.

Falls prevention program –

• falls are responsible for the majority of hip and spine fractures in older people.

• A falls prevention program can provide strategies to help you prevent falls occurring

Measures for Prevention of Falls

• Anchor rugs

• Minimize clutter

• Remove loose wires

• Use nonskid mats

• Install handrails in bathrooms, halls, and long stairways

• Light hallways, stairwells, and entrances

Sources Of Calcium :

• Milk & milk products -cheese, yogurt, ice cream, buttermilk

• Turnip greens, Spinach, Kale, Radish, Okra( lady Finger),bottle gourd

• Dry Beans, such as rajma, chole, chana, lobia, other kidney beans, black-eyed peas, kidney beans, black beans

• Kamal gatta(makhana)

• Water chestnuts/cresnuts(singhada)

• Nuts like peanuts, groundnuts, walnuts, cashew nuts ,almonds , and fruit seeds.

• Seeds like melon seeds, watermelon seeds etc

• Fruits like Custard Apple, Guavas, Banana, Jackfruits, Figs, Oragnges & chiku

Sources of Vit D :

1. Sunlight

2. Food – fatty fish (examples are mackerel, salmon and tuna), egg yolks and liver.

Calcium: take the tablet with meals… and better at night…

Vit D: with meals.. Together with calcium supplements.

Case Study - 29 Answers

1.     What findings are needed for the diagnosis of POAG?            Case Study  – 29

§   enlarged cup-to-disc ratio

§   Visual acuity loss

§   Family history of glaucoma

§   Race/ Ethnicity (African American)

2.     Explain the result of the above mentioned investigations of the ocular exam?    Case Study  – 29

1.      evidence of increased IOP ,

2.      optic nerve head abnormality

3.      open anterior chamber angle,

4.      visual field deficits and no history to suggest a secondary glaucoma (glaucoma due to an identifiable cause.

·         Visual field examination shows defects that are consistent with the state of the optic nerve.

·         Measurements of the nerve fiber layer over the optic nerve can confirm an abnormally thin nerve.

3.     How this patient would be managed ?        Case Study  – 29

Ans : **Damage is permanent to eyes—it cannot be reversed. But medicine and surgery help to stop further damage.

However, Latanoprost 0.005% eyedrops                    only once at bedtime

And, If further required,then,

                   Pilocarpine 1-4% eyedrops thrice a day    OR

                   Brimonidine tartarate 0.2% twice daily     OR

                   Dorzolamide 2% eyedrops   2-3 times a day can be advised.

**If patient is not controlled on 2 topical drugs, then consider alternative treatment options with either laser trabeculoplasty or glaucoma filtering surgery.

4.     Enlist the Patient counselling points in this case?        Case Study  – 29

Patient should be counselled as-

Counselling regarding administration of drops

·         It is extremely important to use your glaucoma eye drops exactly as your ophthalmologist tells you to. That includes taking every dose, every day.

·         But remember, glaucoma eye drops won’t cure glaucoma or improve your vision. They prevent your vision from getting worse. If you don’t use them as prescribed, you could lose your vision.

How to put drops In eyes:

Follow these steps to put in your eye drops:

·         Tilt your head back and look up

·         With one hand, pull your lower eyelid down and away from your eyeball — this makes a “pocket” for the drops

·         With the other hand, hold the eye drop bottle upside down with the tip just above the pocket

·         Squeeze the prescribed number of eye drops into the pocket 

·         For at least 1 minute, close your eye and press your finger lightly on your tear duct (small hole in the inner corner of your eye) — this keeps the eye drop from draining into your nose

·         to use more than 1 type of eye drop, like different drops for different eye conditions, wait at least 5 minutes between each type.

Follow these tips to protect your eyes from infection:

·         Wash your hands with soap and water before you use eye drops

·         Don’t touch the tip of the eye drop bottle with your hands

·         Don’t let the tip of the eye drop bottle touch your eye or eyelid

Storage of drops:

·         All eye drops have an expiration date, which refers to the shelf life of a drop which has not been opened.

·         opened Eye Drops can lose potency and even become contaminated.

·         Most eye drops are stored in a cool dry place and should not be used longer than one month after the bottle is opened, unless otherwise stated on the label.

·         Pt should be advised to mention the date of opening of drops on the label itself.

General points:

·         Do not drive or operate machinery if your glaucoma eye drops make you feel tired or drowsy.

·         Blurry vision, stinging, and redness may improve with time. But if the side effects still bother you, call your ophthalmologist.

·         Never suddenly quit taking your medicine unless your doctor tells you to.

Case Study -30 Answers

1. what is the diagnosis in this case          case study – 30.

Ans • Suspected acute angle closure glaucoma R eye

         • Narrow angle L eye

2. What is the mechanism of angle closure in an episode of acute angle closure glaucoma resulting from a pupillary block?

• The apposition of the pupil border against the lens obstructs aqueous humor flow through the pupil and creates a pressure gradient with increased pressure behind the iris. This moves the iris forward with subsequent apposition of the peripheral iris with the trabecular meshwork. • Decreased drainage by the trabecular meshwork causes increased pressure in the anterior chamber and pushes the iris against the lens. • A portion of vitreous humor moves anteriorly and around the lens, blocking the trabecular meshwork. • Increased aqueous humor drainage through the trabecular meshwork causes a decreased pressure in the anterior chamber, causing a pressure gradient that presses the iris forward and blocks the angle.

3. Mention in brief the surgical options for the disease?          Case Study – 30

Ans: laser iridotomy Is recommended as the first line treatment for all patients Laser peripheral iridotomy/ Surgical iridectomy Preferably, a laser peripheral iridotomy (LPI) is done to alleviate pupillary block. It allows the aqueous to bypass the pupil, providing an alternative route for outflow from posterior to anterior chambers of the eye. Surgical iridectomy may rarely be needed in case of failures of laser iridotomy. Laser peripheral iridotomy: Technique: The role and limitations and possible complications of laser iridotomy are explained to the patient. To reduce the risk of post laser IOP spike and inflammation, apraclonidine 1% or brimonidine 0.15/ 0.2% can be used either before or after the procedure. Alternatively, oral/ topical carbonic anhydrase inhibitors or topical glycerine (in case of corneal epithelial edema secondary to raised IOP) can be used in selected patients.          • It is preferable to reduce IOP to a safe level prior to the procedure. To reduce the risk of bleeding, selected patients on oral anticoagulants for systemic diseases should be counseled and may be asked      to stop their anticoagulants for a few days prior to the procedure.         • Usually iridotomy is recommended between 11-1 o’ clock beneath theeyelids avoiding the 12 o’ clock position. However, others prefer 3 and 9 o’ clock positions.         • PI is avoided at lid margins to reduce symptoms of glare formed by tear meniscus. Successful penetration is seen with a gush of pigments in anterior chamber with a visible deepening of anterior chamber. A minimum opening of 150-200 microns is aimed to ensure patency.        • Surgery is usually considered in case of failure of medical/ laser .        • Management for IOP control or progression of glaucoma despite maximum medical Management.        • Trabeculectomy alone or combined with cataract surgery.

Case Study - 31 Answers

Questions/Answers:

1.      What are the patient counselling points in these case?          Case Study-31

With one hand, pull your lower eyelid down and away from your eyeball — this makes a “pocket” for the drops

To use more than 1 type of eye drop, like different drops for different eye conditions, wait at least 5 minutes between each type.

Follow these tips to protect your eyes from infection :

·         Opened Eye Drops can lose potency and even become contaminated.

·         Wash your face 3-4 times per day with plain water

·         Soak your eyes with cotton filled of water if having any discomfort , it will ease in your painful eyes.

·         Don’t touch your eyes with bare hands

·         If accidently touching the infected eye then wash hands .. OR least possibly don’t touch the eyes which is non infected.

·         Avoid any extra strain on eyes with limited activity of gadgets

·         Do not drive or operate machinery if your  eye drops make you feel tired or drowsy.

2.      What is not a typical exam finding of conjunctivitis?                Case Study-31

·         Eyelid erythema

·         Red conjunctiva

·         Subepithelial corneal infiltrates

·         Anterior chamber cell

·         Mucous in the canthus

Case Study - 32 Answers

1. what are the possible differential diagnosis in this case      case study – 32.

Ans: Differential Diagnosis • Viral Conjunctivitis • Hyperacute Bacterial Conjunctivitis • Chlamydial Conjunctivitis • Allergic Conjunctivitis • Superficial Keratitis • Blepharitis • Episcleritis • Scleritis • Acute Angle-closure Glaucoma • Acute Anterior Uveitis

2. What is the role of Ofloxacin in this Patient’s case?

Ans: Ofloxacin was prescribed to replace tobramycin due to suspecting toxic reaction to Tobramycin. Patient was asked to instil one drop of ofloxacin into the left Eye q2h on the day, to reduce the dosage to qid the next day if the condition Improves, the right eye is to be treated with ofloxacin qid. A review appointment was Scheduled in one week time.

3. Why Tobramycin wasn’t readvised after the PCR result, is there any evidence suggestive for the change?   

Case study – 32.

Ans:        1. the patient had an adverse Reaction to the aminoglycoside, this was evident when she noted that the Discontinuation of tobramycin had reduced ocular hyperaemia, but resulted an Increase in ocular discharge.      2. PCR was performed to rule possible viral and Chlamydia infection. This was deemed necessary as her condition was unresponsive To prescribed therapy.      3. The ophthalmologist had a strong suspicion that the condition Was caused by a gram-negative bacteria due to the nature of ocular discharge, Therefore prescribed ofloxacin as it has a relatively strong antibacterial activity Against gram-negative organisms.

Case Study - 33 Answers

1.What should Leanne’s GP consider as a possible diagnosis?       Case Study- 33

Ans: Following points should be considered for the diagnostic purpose-

·         Take a detailed history from Leanne and her parents and explore the ‘trances’ because they have experienced and witnessed them. This should determine whether an epileptic seizure is likely to have occurred.

·         Diagnosis should not be based on the presence or absence of single features.

·         Consider a history of absence seizures. Leanne is the right age

and gender for this relatively common childhood epilepsy syndrome.

·         The frequent occurrence of the daydreams and the fact that they interrupt her activities are suspicious features of childhood-onset absence epilepsy.

·         The positive family history is supportive but not diagnostic of this epilepsy syndrome. Childhood-onset absence epilepsy is classified as an idiopathic (presumed genetic) generalised epilepsy.

·         Confirm the diagnosis in the surgery. Children with typical absence seizures will often experience one of their absences during hyperventilation (over-breathing). However, hyperventilation usually has to be performed well and for at least 3 minutes to induce an absence.

·         Refer Leanne to a general paediatrician with an interest in epilepsy or a paediatric neurologist to establish the diagnosis.

Ans: First, information should be given to Leanne and her parents about the reasons For further tests, and they should be carried out in a child-centred environment.

·         An electroencephalogram (EEG) should be arranged and Leanne should have This test soon after it has been requested. Because the paediatrician or Paediatric neurologist suspects that her seizures are epileptic in origin, the EEG Should be performed to support a diagnosis of epilepsy.

·         It should not be used in Isolation to make a diagnosis of epilepsy.

·         The healthcare professionals carrying Out the EEG should encourage Leanne to hyperventilate, because this is one of The provocation techniques always undertaken during an EEG.

·         The EEG is abnormal, as in the vast majority of children with childhood-onset Absence epilepsy. It ‘captures’ an absence seizure, particularly during Hyperventilation.

3. What should the discussion around medication include, and what medications may be Prescribed?   Case Study- 33

·         The discussion on anti-epileptic medication should include the different . Medications that are available, and specifically ethosuximide, sodium valproate And lamotrigine, the evidence base for using these medications.

·         Their Common and potentially unwanted side effects. Discussion should also include The likely outcome or prognosis of the epilepsy and specifically that it will go into Spontaneous remission (that is, it will ‘go away’).

·         The family should also be Referred to a paediatric epilepsy nurse who can provide information and Guidance on lifestyle and other non-medical issues.

·         Ethosuximide or sodium valproate should be offered as a first-line treatment.

Academia.edu no longer supports Internet Explorer.

To browse Academia.edu and the wider internet faster and more securely, please take a few seconds to  upgrade your browser .

Enter the email address you signed up with and we'll email you a reset link.

  • We're Hiring!
  • Help Center

paper cover thumbnail

100 Cases in Clinical Pharmacology Therapeutics and Prescribing 1st edition

Profile image of Dragutin Petrić

Related Papers

Jeffrey Ginsberg

case study examples pharmacology

American Journal of Emergency Medicine

IJAR Indexing

Tuberculosis has a high prevalence in India but pulmonary embolism is rarely reported in Mycobacterium tuberculosis infection. We describe a case series of pulmonary embolism associated with severe pulmonary tuberculosis. In our first case pulmonary embolism (PE) occurred within 6months of pulmonary tuberculosis diagnosis.Patient took ATT for 6 months and after being diagnosed as a case of PE, low-molecular-weight heparin was prescribed for 3 days and after that he was given Tab. warfarin (5mg) OD. Our second case diagnosed as a case of PE after taking 9months of ATT and he also prescribed Tab.Warfarin and low molecular weight heparin. The association between tuberculosis and pulmonary embolism is rare. Tuberculosis, the disease can itself lead to hypercoagulability, increased venous stasis, and endothelial dysfunction, thus increasing the susceptibility to venous thromboembolism (VTE) particularlyin those with severe pulmonary or disseminated tuberculosis. Among the ATT drugs, most commonly associated with thromboembolic events is Rifampicin, an enzyme inducer, and may alter the balance of anticoagulant and coagulant proteins produced by the liver. But it should be in detailed investigation should be done by respiratory physicians to rule out etiology whether it is due to drug or disease itself.

Turkish Thoracic Journal

Emine Argüder

Current Treatment Options in Cardiovascular Medicine

Doug Humber

Circulation

Gregory Elliott

Henry Eriksson

Revista Española de Cardiología (English Edition)

Jose Luis Pichardo Nieto

Annals of Pharmacotherapy

Munir Pirmohamed

Cor et Vasa

Giulia Renda

Loading Preview

Sorry, preview is currently unavailable. You can download the paper by clicking the button above.

RELATED PAPERS

The Anatolian Journal of Cardiology

marija vavlukis

Indian journal of critical care medicine : peer-reviewed, official publication of Indian Society of Critical Care Medicine

Clinical Cardiology

Curt Bay , Frederic Schwartz

Tuberculosis and respiratory diseases

Sam Schulman

Journal of The American Geriatrics Society

Francesc Formiga

INTERNATIONAL JOURNAL OF SCIENTIFIC STUDY IJSS

Kardiologia Polska

Olga Dzikowska-Diduch

Brown Hospital Medicine

Alisa Pugacheva

Jornal Brasileiro de Pneumologia

Carlos Henrique Miranda

HOW TO ANTICOAGULATE: CLINIC AND MANAGEMENT OF PULMONARY THROMBOEMBOLISM – LITERATURE REVIEW (Atena Editora)

Atena Editora

Davood Mansouri , Ilad Alavi Darazam

Ahmad Alharafsheh

Critical Care

Levent Yamanel , Volkan Inal

Postgraduate Medical Journal

Pier-Luigi Rocco

Journal of blood & lymph

Paolo Conca

Francesco Pistelli

Ammar Al-obaidi

Jeffrey Weitz

Circulation Journal

John McCall

Pulmonary Embolism

Diana Mühl , Subhamay Ghosh

Arquivos brasileiros de cardiologia

Jussara Castelli

Sandra Zelman Lewis

Journal of Academic Emergency Medicine

Mehmet Altuntas

The Kaohsiung Journal of Medical Sciences

Inn-wen Chong

Clinical Colorectal Cancer

M. Copur , Max Norvell

  •   We're Hiring!
  •   Help Center
  • Find new research papers in:
  • Health Sciences
  • Earth Sciences
  • Cognitive Science
  • Mathematics
  • Computer Science
  • Academia ©2024

case study examples pharmacology

Clinical Pharmacology: Current Topics and Case Studies

  • © 2016
  • Latest edition
  • Markus Müller 0

Universitätsklinik für Klinische, Medizinische Universität Wien, Wien, Austria

You can also search for this editor in PubMed   Google Scholar

Updated and revised 2nd edition

Summarizes the latest topics, tools and clinical trials in clinical pharmacology

Added value for employees in drug research and development

61k Accesses

13 Citations

7 Altmetric

This is a preview of subscription content, log in via an institution to check access.

Access this book

Subscribe and save.

  • Get 10 units per month
  • Download Article/Chapter or eBook
  • 1 Unit = 1 Article or 1 Chapter
  • Cancel anytime
  • Available as EPUB and PDF
  • Read on any device
  • Instant download
  • Own it forever
  • Compact, lightweight edition
  • Dispatched in 3 to 5 business days
  • Free shipping worldwide - see info
  • Durable hardcover edition

Tax calculation will be finalised at checkout

Other ways to access

Licence this eBook for your library

Institutional subscriptions

About this book

Similar content being viewed by others.

case study examples pharmacology

Clinical Pharmacology Regulatory Sciences in Drug Development and Precision Medicine: Current Status and Emerging Trends

case study examples pharmacology

Clinical Drug Trials: The Path to the Patient

case study examples pharmacology

Clinical Research in Pharmaceutical Drug Development

  • drug development
  • clinical trial
  • gene therapy
  • pharmacodynamics
  • drug research
  • pharmacotherapy

Table of contents (23 chapters)

Front matter, introduction, the discipline of clinical pharmacology.

Markus Müller

Current Issues in Drug Development

Current issues in drug regulation.

  • Christa Wirthumer-Hoche, Brigitte Bloechl-Daum

Current Topics in Drug Reimbursement

  • Anna Bucsics, Robert Sauermann, Valerie Nell-Duxneuner

Clinical Trials

Ethics in clinical research.

  • Ernst Singer, Christiane Druml

Good Clinical Practice (GCP) and Scientific Misconduct

  • Brigitte Bloechl-Daum

Phase I Studies and First-In-Human Trials

  • Ulla Derhaschnig, Bernd Jilma

Clinical Trials: Interventional Studies

  • Michael Wolzt, Stefan Aschauer

Observational Studies

  • Harald Herkner, Christoph Male

Tools in Clinical Pharmacology

Tools in clinical pharmacology: imaging techniques.

  • Martin Bauer, Oliver Langer

Pharmacokinetics II: 14C-Labelled Microdosing in Assessing Drug Pharmacokinetics at Phase 0

  • Graham Lappin

Current Concepts of Pharmacogenetics, Pharmacogenomics, and the “Druggable” Genome

  • Wolfgang M. Schmidt, Robert M. Mader

Pharmacokinetics I: PK-PD Approach, the Case of Antibiotic Drug Development

  • Sherwin K. B. Sy, Hartmut Derendorf

Epidemiology and Biostatistics

  • Gerhard Garhöfer, Leopold Schmetterer

Placebo Effects and Placebo Control in Clinical Trials

  • Johannes Pleiner-Duxneuner

Topics in Clinical Pharmacology

“The aim is to describe the role of the discipline of clinical pharmacology in drug discovery. … The intended audience is faculty, researchers, and advanced students from academia and the pharmaceutical industry who are learning about or involved in the process of drug discovery. … This book provides a multitude of very useful insights about the successes and failures in drug development for scientists who are, or will be, involved in the process of drug discovery.” (Thomas L. Pazdernik, Doody's Book Reviews, May, 2016)

Editors and Affiliations

About the editor, bibliographic information.

Book Title : Clinical Pharmacology: Current Topics and Case Studies

Editors : Markus Müller

DOI : https://doi.org/10.1007/978-3-319-27347-1

Publisher : Springer Cham

eBook Packages : Biomedical and Life Sciences , Biomedical and Life Sciences (R0)

Copyright Information : Springer International Publishing Switzerland 2016

Hardcover ISBN : 978-3-319-27345-7 Published: 25 March 2016

Softcover ISBN : 978-3-319-80118-6 Published: 25 April 2018

eBook ISBN : 978-3-319-27347-1 Published: 15 March 2016

Edition Number : 2

Number of Pages : VI, 405

Number of Illustrations : 14 b/w illustrations, 16 illustrations in colour

Topics : Pharmacology/Toxicology , Pharmacy , Pharmacotherapy

  • Publish with us

Policies and ethics

  • Find a journal
  • Track your research
  • Remote Access
  • Save figures into PowerPoint
  • Download tables as PDFs

Infectious Diseases: A Case Study Approach

4:  Pharyngitis

Frank S. Yu; Jonathan C. Cho

  • Download Chapter PDF

Disclaimer: These citations have been automatically generated based on the information we have and it may not be 100% accurate. Please consult the latest official manual style if you have any questions regarding the format accuracy.

Download citation file:

  • Search Book

Jump to a Section

Patient presentation.

  • Full Chapter
  • Supplementary Content

Chief Complaint

“Mommy, my throat is on fire!”

History of Present Illness

JT is a 7-year-old Chinese American female, accompanied by her mother, who presents to the community pharmacy with complaints of sore throat and fever, looking for medications to take to relieve her symptoms. She is fussy and describes the pain when she swallows as feeling if her throat is “on fire.” Her symptoms began yesterday morning, and she has only tried drinking a pei pa koa syrup containing medicinal herbs (main active herb is elm bark) and honey to relieve the sore throat. This provided some relief but the pain has been getting worse. She did not have a temperature taken, but her forehead was hot to the touch. She was not given any medications to relieve the fever. She was dressed with additional clothing and blankets to “sweat the fever out,” but the fever still persisted. She reports that there may have been other sick classmates. She denies a prior history of sore throat.

Past Medical History

Attention-deficit disorder, recurrent otitis media (resolved)

Surgical History

Family history.

Non-contributory

Social History

Ear tubes at age 2

Amoxicillin (throat swelling, difficulty breathing)

Home Medications

Methylphenidate ER 18 mg PO daily

Physical Examination

Vital signs.

Temp 101.9°F (oral), Ht 4′4″, Wt 29.55 kg

Appears uncomfortable, tired, grimacing when swallowing

Anterior cervical lymph nodes enlarged and tender; tonsils moist, red, with white exudates

Point-of-Care GAS Rapid Antigen Detection Test

1. What is the most common pathogen responsible for acute bacterial pharyngitis in children?

A. Corynebacterium diphtheriae

B. Neisseria gonorrhoeae

C. Group C streptococcus

D. Group A streptococcus

2. What signs and symptoms in this patient definitely discriminate between GAS pharyngitis rather than viral pharyngitis?

A. Tonsils with white exudates

B. Temperature 101.9°F

C. Pain on swallowing

D. All of the above

3. If GAS is suspected, what age range is typically excluded for testing for GAS?

Pharmacy Case Studies for Pharmacists & Medical Sciences Students

Pharmacists and healthcare practitioners are required to demonstrate knowledge and understanding of the application of therapeutics in clinical practice. Pharmacists must ensure patient safety and achieve desired health outcomes through effective decision-making. The idea of designing these case studies is to meet the needs and challenges of a modern pharmacy undergraduate curriculum. Case studies are increasingly used in pharmacy undergraduate as well as postgraduate education.

Each chapter contains five case studies, increasing in complexity from those we would expect first-year students to complete (Level 1) through to cases designed for fourth-year/pre-registration students (Level M). The chapters have been designed to follow approximately the British National Formulary chapters for ease of use. Case study scenarios include both community and hospital pharmacy situations as suited to the disease and pharmaceutical care provision.

This section is only for Bangladeshi Pharmacy/Medical Students & Professionals !

Cardiovascular case studies by Narinder Bhalla

Case study level 1 – Angina Case study level 2 – Hypertension Case study level 3 – Atrial fibrillation Case study level Ma – Heart failure Case study level Mb – Myocardial infarction

Respiratory system case studies by Soraya Dhillon and Andrzej Kostrzewski

Case study level 1 – Asthma – community Case study level 2 – Asthma – acute on chronic Case study level 3 – Chronic obstructive pulmonary disease (COPD) with co-morbidity Case study level Ma – COPD Case study level Mb – Brittle asthma

Obstetrics, gynaecology and UTI case studies by Alka Mistry

Case study level 1 – Primary dysmenorrhoea Case study level 2 – Urinary tract infections in pregnancy Case study level 3 – Pelvic inflammatory disease Case study level Ma – Endometriosis management in secondary care Case study level Mb – Management of severe pre-eclampsia/ eclampsia

Liver disease case studies by Caron Weeks and Mark Tomlin

Case study level 1 – Alcoholic cirrhosis; alcohol withdrawal Case study level 2 – Alcoholic cirrhosis; management of bleeding risk and treatment for the maintenance of alcohol abstinence Case study level 3 – Hepatic encephalopathy and ascites Case study level Ma – Pulmonary tuberculosis Case study level Mb – Liver failure  
  • Case study for pharmacist
  • Pharmacy case study

You might be interested in

pharma case studies

Liver disease case studies: Case study level 1 – Alcoholic cirrhosis; alcohol withdrawal

Cardiovascular case studies: case study level mb – myocardial infarction, cardiovascular case studies : case study level 3 – atrial fibrillation.

Nice sources

How to read all case studies?

Leave a Reply Cancel reply

Your email address will not be published.

{{#message}}{{{message}}}{{/message}}{{^message}}Your submission failed. The server responded with {{status_text}} (code {{status_code}}). Please contact the developer of this form processor to improve this message. Learn More {{/message}}

{{#message}}{{{message}}}{{/message}}{{^message}}It appears your submission was successful. Even though the server responded OK, it is possible the submission was not processed. Please contact the developer of this form processor to improve this message. Learn More {{/message}}

Submitting…

This site uses Akismet to reduce spam. Learn how your comment data is processed .

BioPharma Asia Convention Terrapinn

Press Release: 4 Symposiums at BioPharma Asia Convention 2013, Singapore

bioinformatics notes

Notes on Bioinformatics Basic

case study examples pharmacology

Login into your account

Please enter username and password bellow!

Forgotten Password

Don't have an account? Register here

Our Pharmacy Blog

How to answer pharmacy case studies.

how to answer pharmacy case studies

How to Answer Pharmacy Case Studies

Whether you’re a student of pharmacy, medicine or nursing – you cannot underestimate the value of case studies.

Case studies bring together the essential strands of your professional study. They are the means through which you analyse a patient, their condition, the medicines they are taking and the impact that additional medicines have on their circumstances.

Studying subjects separately – whether it be pharmacology, pharmaceutical chemistry, pharmaceutics or clinical pharmacy – has value, but it’s though practicing case studies that bring the essential, relevant facts into sharp relief.

Reaching the next level

Case studies are what elevate your status from general student into prospective clinical professional. The more case studies you review, the more rounded your knowledge becomes. Your critical thinking skills become more refined and more focussed.

The link between chemistry, how it translates into a meaningful pharmacological impact, and how these twin forces inform your real-life clinical judgement, suddenly begins to make sense.

To begin with, case studies are challenging, difficult and can dampen your confidence. In time, case studies become enjoyable, insightful and a positive force that serves to nourish clinical judgement.

It’s through persistence and hard work that you reap the long-term benefits of case studies. Knowing how to approach case studies , then, is important. With the wrong approach and an improper mindset, you’ll never extract the value that case studies bring.

How to approach case studies

When faced with a case study, approach it with the following points in mind:

Review the case study more than once . Never rush into a case study, picking and choosing facts you understand. Instead, approach the case study with an open mind. Be prepared to accept that you might not know the answer straight-off. The study may require some careful consideration, time and reflection.

Evaluate the clinical significance of each statement . If told that the patient’s blood pressure is 148/90, relate this fact to every other detail in the question. Is hypertension a side effect of any of the patient’s medicines? Does the presence of hypertension mean that the patient’s dosing is incorrect? Is hypertension a cause of a drug-drug interaction?

Perhaps hypertension is not caused by any medicine but is instead related to the patient’s lifestyle. In other words, you need to approach each statement from a multi-dimensional perspective – reviewing every possible angle and dismissing those angles that cannot possibly be true.

Take the patient’s age into consideration . Knowing the patient’s age is extremely important. When informed of the patient’s age, think about:

  • Whether the patient is a child. Dosing is lower in children. Many medicines cannot be administered to children (think of aspirin and tetracyclines, for example).
  • Older patients are more likely to experience side effects. For example – CNS effects, such as drowsiness or dizziness, are invariably worse. Older patients are more susceptible to infection, too. The risk of bleeding increases too, a factor that should inform your judgement of anticoagulants, such as warfarin.
  • Age and the onset of biological change. Female patients over the typical menopausal age may be taking bisphosphonates to ward off (or treat) the onset of osteoporosis. Male patients over of the age of 40 produce less antidiuretic hormone, meaning they urinate more.
  • How senescence impacts drug metabolism. Older patients metabolise drugs at a slower rate than younger patients, impacting their adverse effect profile.

Of course, these are just some of the factors you should consider. At this point, we must appreciate why age is, when evaluating case studies, is an important consideration.

How their medical history informs the debate . An older patient may be admitted to the hospital with an acute bacterial infection. Five years ago, they patient may have been treated for deep vein thrombosis. As a result, anticoagulant therapy should be considered. There are three reasons:

  • First, the patient is older meaning that risk of recurring DVT increases
  • The patient has a history of DVT
  • The patient will be relatively immobile during hospital admission, increasing risk

The relationship between past disease and current treatment is important.

Understand diagnostic parameters . As a prospective clinical professional, you must understand the various diagnostic factors at play.

For example – sodium, potassium, white blood cell and cholesterol ranges; what is normal, what are considered low values and what high values mean? Understand the relationship between these factors, the medicines the patient is taking and the how their underlying disease impacts both factors.

For example – proton-pump inhibitors reduce gastric acid levels. Because ketoconazole is best absorbed in acidic conditions, proton-pump inhibitors impact absorption and therapeutic efficacy. High gastric pH also reduces metabolism of essential nutrients, such as vitamin B 12 .

Know side effects and drug interactions . It’s challenging to learn the major side effects and drug interactions. It doesn’t come overnight – it takes time. That’s why studying pharmacy case studies is important; they give you the opportunity to apply the learning you’ve done so far and extract the detail that you have yet to learn. Take two minutes to revise our compilation of major side effects .

Build your knowledge of side effects over time. Often, knowing the drug’s mechanism of action is enough to inform your judgement of either potential side effects or drug interactions.

For example – knowing that opioids, such as buprenorphine, causes respiratory depression should trigger alarm bells if the patient is co-prescribed another medicine, such as alprazolam; a benzodiazepine which itself causes respiratory depression.

Practice, practice, practice!

Knowing how to answer pharmacy case studies effectively is invariably down to practice, too. The more practice, the more knowledge gaps are plugged and the greater the degree of context you construct around specific clinical topics.

At PharmaFactz, we have developed over pharmacy case studies for you to practice . These case studies cover just about every aspect of clinical pharmacy – major organ systems, diagnostics, blood test results, side effects and drug interactions, and the essential clinical facts you need to know – helping you acquire the knowledge and expertise you need to excel at pharmacy case studies.

  • Anticancer Pharmacology
  • Antimicrobial Drugs
  • Cardiovascular Pharmacology
  • Clinical Case Study
  • Clinical Pharmacy
  • General Pharmacology
  • GI Pharmacology
  • Immune System Pharmacology
  • Nervous System Pharmacology
  • Respiratory Pharmacology
  • Study Tips and Tricks

Join Our Mailing List For Even More Facts!

Don't stop learning now, you may also like, fluoroquinolones pharmacology, beta-2 agonists pharmacology, ssris pharmacology.

  • Search Menu
  • Sign in through your institution
  • Volume 2024, Issue 8, August 2024 (In Progress)
  • Volume 2024, Issue 7, July 2024
  • Case of the Year
  • MSF Case Reports
  • Audiovestibular medicine
  • Cardiology and cardiovascular systems
  • Critical care medicine
  • Dermatology
  • Emergency medicine
  • Endocrinology and metabolism
  • Gastroenterology and hepatology
  • Geriatrics and gerontology
  • Haematology
  • Infectious diseases and tropical medicine
  • Medical ophthalmology
  • Medical disorders in pregnancy
  • Paediatrics
  • Palliative medicine
  • Pharmacology and pharmacy
  • Radiology, nuclear medicine, and medical imaging
  • Respiratory disorders
  • Rheumatology
  • Sexual and reproductive health
  • Sports medicine
  • Substance abuse
  • Author Guidelines
  • Submission Site
  • Open Access
  • Reasons to publish with us
  • Editorial Board
  • Advertising and Corporate Services
  • Journals Career Network
  • Self-Archiving Policy
  • Journals on Oxford Academic
  • Books on Oxford Academic

Article Contents

Answer to part 1, answer to part 2, answer to part 3, answer to part 4, answer to part 5.

  • < Previous

Educational Case: A 57-year-old man with chest pain

Contributed equally.

  • Article contents
  • Figures & tables
  • Supplementary Data

Nikhil Aggarwal, Subothini Selvendran, Vassilios Vassiliou, Educational Case: A 57-year-old man with chest pain, Oxford Medical Case Reports , Volume 2016, Issue 4, April 2016, Pages 62–65, https://doi.org/10.1093/omcr/omw008

  • Permissions Icon Permissions

This is an educational case report including multiple choice questions and their answers. For the best educational experience we recommend the interactive web version of the exercise which is available via the following link: http://www.oxfordjournals.org/our_journals/omcr/ec01p1.html

A 57 year-old male lorry driver, presented to his local emergency department with a 20-minute episode of diaphoresis and chest pain. The chest pain was central, radiating to the left arm and crushing in nature. The pain settled promptly following 300 mg aspirin orally and 800 mcg glyceryl trinitrate (GTN) spray sublingually administered by paramedics in the community. He smoked 20 cigarettes daily (38 pack years) but was not aware of any other cardiovascular risk factors. On examination he appeared comfortable and was able to complete sentences fully. There were no heart murmurs present on cardiac auscultation. Blood pressure was 180/105 mmHg, heart rate was 83 bpm and regular, oxygen saturation was 97%.

What is the most likely diagnosis?

AAcute coronary syndrome
BAortic dissection
CEsophageal rupture
DPeptic ulceration
EPneumothorax

An ECG was requested and is shown in figure 1.

How would you manage the patient? (The patient has already received 300 mg aspirin).

AAtenolol 25 mg, Atorvastatin 80 mg, Clopidogrel 75 mg, GTN 500 mcg
BAtenolol 25 mg, Clopidogrel 75 mg, GTN 500 mcg, Simvastatin 20 mg
CAtorvastatin 80 mg, Clopidogrel 300 mcg, GTN 500 mcg, Ramipril 2.5 mg
DAtorvastatin 80 mg, Clopidogrel 75 mg, Diltiazem 60 mg, Oxygen
EClopidogrel 300 mg, Morphine 5 mg, Ramipril 2.5 mg, Simvastatin 20 mg

30 minutes later the patient's chest pain returned with greater intensity whilst waiting in the emergency department. Now, he described the pain as though “an elephant is sitting on his chest”. The nurse has already done an ECG by the time you were called to see him. This is shown in figure 2.

ECG on admission.

ECG on admission.

ECG 30 minutes after admission.

ECG 30 minutes after admission.

What would be the optimal management for this patient?

AAdminister intravenous morphine
BIncrease GTN dose
CObserve as no new significant changes
DProceed to coronary angiography
EThrombolyse with alteplase

He was taken to the catheterization lab where the left anterior descending coronary artery (LAD) was shown to be completely occluded. Following successful percutaneous intervention and one drug eluding stent implantation in the LAD normal flow is restored (Thrombosis in myocardial infarction, TIMI = 3). 72 hours later, he is ready to be discharged home. The patient is keen to return to work and asks when he could do so.

When would you advise him that he could return to work?

A1 week later
B3 weeks later
C6 weeks later
DNot before repeat angiography
ENot before an exercise test

One week later, he receives a letter informing him that he is required to attend cardiac rehabilitation. The patient is confused as to what cardiac rehabilitation entails, although he does remember a nurse discussing this with him briefly before he was discharged. He phones the hospital in order to get some more information.

Which of the following can be addressed during cardiac rehabilitation?

ADiet
BExercise
CPharmacotherapy
DSmoking cessation
EAll of the above

A - Acute coronary syndrome

Although the presentation could be attributable to any of the above differential diagnoses, the most likely etiology given the clinical picture and risk factors is one of cardiac ischemia. Risk factors include gender, smoking status and age making the diagnosis of acute coronary syndrome the most likely one. The broad differential diagnosis in patients presenting with chest pain has been discussed extensively in the medical literature. An old but relevant review can be found freely available 1 as well as more recent reviews. 2 , 3

C - Atorvastatin 80 mg, Clopidogrel 300 mcg, GTN 500 mcg, Ramipril 2.5 mg,

In patients with ACS, medications can be tailored to the individual patient. Some medications have symptomatic benefit but some also have prognostic benefit. Aspirin 4 , Clopidogrel 5 , Atenolol 6 and Atorvastatin 7 have been found to improve prognosis significantly. ACE inhibitors have also been found to improve left ventricular modeling and function after an MI. 8 , 9 Furthermore, GTN 10 and morphine 11 have been found to be of only significant symptomatic benefit.

Oxygen should only to be used when saturations <95% and at the lowest concentration required to keep saturations >95%. 12

There is no evidence that diltiazem, a calcium channel blocker, is of benefit. 13

His ECG in figure 1 does not fulfil ST elevation myocardial infarction (STEMI) criteria and he should therefore be managed as a Non-STEMI. He would benefit prognostically from beta-blockade however his heart rate is only 42 bpm and therefore this is contraindicated. He should receive a loading dose of clopidogrel (300 mg) followed by daily maintenance dose (75 mg). 14 , 15 He might not require GTN if he is pain-free but out of the available answers 3 is the most correct.

D - Proceed to coronary angiography

The ECG shows ST elevation in leads V2-V6 and confirms an anterolateral STEMI, which suggests a completely occluded LAD. This ECG fulfils the criteria to initiate reperfusion therapy which traditionally require one of the three to be present: According to guidance, if the patient can undergo coronary angiography within 120 minutes from the onset of chest pain, then this represents the optimal management. If it is not possible to undergo coronary angiography and potentially percutaneous intervention within 2 hours, then thrombolysis is considered an acceptable alternative. 12 , 16

≥ 1 mm of ST change in at least two contiguous limb leads (II, III, AVF, I, AVL).

≥ 2 mm of ST change in at least two contiguous chest leads (V1-V6).

New left bundle branch block.

GTN and morphine administration can be considered in parallel but they do not have a prognostic benefit.

E - Not before an exercise test

This patient is a lorry driver and therefore has a professional heavy vehicle driving license. The regulation for driving initiation in a lorry driver following a NSTEMI/ STEMI may be different in various countries and therefore the local regulations should be followed.

In the UK, a lorry driver holds a category 2 driving license. He should therefore refrain from driving a lorry for at least 6 weeks and can only return to driving if he completes successfully an exercise evaluation. An exercise evaluation is performed on a bicycle or treadmill. Drivers should be able to complete 3 stages of the standard Bruce protocol 17 or equivalent (e.g. Myocardial perfusion scan) safely, having refrained from taking anti-anginal medication for 48 hours and should remain free from signs of cardiovascular dysfunction during the test, notably: angina pectoris, syncope, hypotension, sustained ventricular tachycardia, and/or electrocardiographic ST segment shift which is considered as being indicative of myocardial ischemia (usually >2 mm horizontal or down-sloping) during exercise or the recovery period. 18

For a standard car driving license (category 1), driving can resume one week after successful intervention providing that no other revascularization is planned within 4 weeks; left ventricular ejection fraction (LVEF) is at least 40% prior to hospital discharge and there is no other disqualifying condition.

Therefore if this patent was in the UK, he could restart driving a normal car one week later assuming an echocardiogram confirmed an EF > 40%. However, he could only continue lorry driving once he has passed the required tests. 18

E - All of the above

Cardiac rehabilitation bridges the gap between hospitals and patients' homes. The cardiac rehabilitation team consists of various healthcare professions and the programme is started during hospital admission or after diagnosis. Its aim is to educate patients about their cardiac condition in order to help them adopt a healthier lifestyle. This includes educating patients' about their diet, exercise, risk factors associated with their condition such as smoking and alcohol intake and finally, about the medication recommended. There is good evidence that adherence to cardiac rehabilitation programmes improves survival and leads to a reduction in future cardiovascular events.​ 19 , 20

Oille JA . Differential diagnosis of pain in the chest . Can Med Assoc J . 1937 ; 37 (3) : 209 – 216 . http://www.ncbi.nlm.nih.gov/pmc/articles/PMC536075/ .

Google Scholar

Lee TH , Goldman L . Evaluation of the patient with acute chest pain . N Engl J Med . 2000 ; 342 (16) : 1187 – 1195 . http://www.nejm.org/doi/full/10.1056/NEJM200004203421607 .

Douglas PS , Ginsburg GS . The evaluation of chest pain in women . N Engl J Med . 1996 ; 334 (20) : 1311 – 1315 . http://www.nejm.org/doi/full/10.1056/NEJM199605163342007 .

Baigent C , Collins R , Appleby P , Parish S , Sleight P , Peto R . ISIS-2: 10 year survival among patients with suspected acute myocardial infarction in randomised comparison of intravenous streptokinase, oral aspirin, both, or neither. the ISIS-2 (second international study of infarct survival) collaborative group . BMJ . 1998 ; 316 (7141) : 1337 – 1343 . http://www.ncbi.nlm.nih.gov/pmc/articles/PMC28530/ .

Yusuf S , Zhao F , Mehta S , Chrolavicius S , Tognoni G , Fox K . Clopidogrel in unstable angina to prevent recurrent events trail investigators . effects of clopidogrel in addition to aspirin in patients with acute coronary syndromes without ST-segment elevation . N Engl J Med . 2001 ; 345 (7) : 494 – 502 . http://www.nejm.org/doi/full/10.1056/NEJMoa010746#t=articleTop .

Yusuf S , Peto R , Lewis J , Collins R , Sleight P . Beta blockade during and after myocardial infarction: An overview of the randomized trials . Prog Cardiovasc Dis . 1985 ; 27 (5) : 335 – 371 . http://www.sciencedirect.com/science/article/pii/S0033062085800037 .

Schwartz GG , Olsson AG , Ezekowitz MD et al.  . Effects of atorvastatin on early recurrent ischemic events in acute coronary syndromes: The MIRACL study: A randomized controlled trial . JAMA . 2001 ; 285 (13) : 1711 – 1718 . http://jama.jamanetwork.com/article.aspx?articleid=193709 .

Pfeffer MA , Lamas GA , Vaughan DE , Parisi AF , Braunwald E . Effect of captopril on progressive ventricular dilatation after anterior myocardial infarction . N Engl J Med . 1988 ; 319 (2) : 80 – 86 . http://content.onlinejacc.org/article.aspx?articleid=1118054 .

Sharpe N , Smith H , Murphy J , Hannan S . Treatment of patients with symptomless left ventricular dysfunction after myocardial infarction . The Lancet . 1988 ; 331 (8580) : 255 – 259 . http://www.sciencedirect.com/science/article/pii/S0140673688903479 .

Ferreira JC , Mochly-Rosen D . Nitroglycerin use in myocardial infarction patients . Circ J . 2012 ; 76 (1) : 15 – 21 . http://www.ncbi.nlm.nih.gov/pmc/articles/PMC3527093/ .

Herlitz J , Hjalmarson A , Waagstein F . Treatment of pain in acute myocardial infarction . Br Heart J . 1989 ; 61 (1) : 9 – 13 . http://www.ncbi.nlm.nih.gov/pmc/articles/PMC1216614/ .

Task Force on the management of ST-segment elevation acute myocardial infarction of the European Society of Cardiology (ESC), Steg PG, James SK, et al . ESC guidelines for the management of acute myocardial infarction in patients presenting with ST-segment elevation . Eur Heart J . 2012 ; 33 (20) : 2569 – 2619 . http://eurheartj.oxfordjournals.org/content/33/20/2569 .

The effect of diltiazem on mortality and reinfarction after myocardial infarction . the multicenter diltiazem postinfarction trial research group . N Engl J Med . 1988 ; 319 (7) : 385 – 392 . http://www.nejm.org/doi/full/10.1056/NEJM198808183190701 .

Jneid H , Anderson JL , Wright RS et al.  . 2012 ACCF/AHA focused update of the guideline for the management of patients with unstable angina/Non–ST-elevation myocardial infarction (updating the 2007 guideline and replacing the 2011 focused update) A report of the american college of cardiology foundation/american heart association task force on practice guidelines . J Am Coll Cardiol . 2012 ; 60 (7) : 645 – 681 . http://circ.ahajournals.org/content/123/18/2022.full .

Hamm CW , Bassand JP , Agewall S et al.  . ESC guidelines for the management of acute coronary syndromes in patients presenting without persistent ST-segment elevation: The task force for the management of acute coronary syndromes (ACS) in patients presenting without persistent ST-segment elevation of the european society of cardiology (ESC) . Eur Heart J . 2011 ; 32 (23) : 2999 – 3054 . http://eurheartj.oxfordjournals.org/content/32/23/2999.long .

O'Gara PT , Kushner FG , Ascheim DD et al.  . 2013 ACCF/AHA guideline for the management of ST-elevation myocardial infarction: Executive summary: A report of the american college of cardiology foundation/american heart association task force on practice guidelines . J Am Coll Cardiol . 2013 ; 61 (4) : 485 – 510 . http://content.onlinejacc.org/article.aspx?articleid=1486115 .

BRUCE RA , LOVEJOY FW Jr . Normal respiratory and circulatory pathways of adaptation in exercise . J Clin Invest . 1949 ; 28 (6 Pt 2) : 1423 – 1430 . http://www.ncbi.nlm.nih.gov/pmc/articles/PMC439698/ .

DVLA . Https://Www.gov.uk/current-medical-guidelines-dvla-guidance-for-professionals-cardiovascular-chapter-appendix .

British Heart Foundation . Http://Www.bhf.org.uk/heart-health/living-with-heart-disease/cardiac-rehabilitation.aspx .

Kwan G , Balady GJ . Cardiac rehabilitation 2012: Advancing the field through emerging science . Circulation . 2012 ; 125 (7) : e369–73. http://circ.ahajournals.org/content/125/7/e369.full .

Author notes

  • knowledge acquisition
Month: Total Views:
December 2016 1
January 2017 46
February 2017 45
March 2017 32
April 2017 55
May 2017 35
June 2017 71
July 2017 1
August 2017 4
September 2017 2
October 2017 10
November 2017 25
December 2017 127
January 2018 161
February 2018 150
March 2018 194
April 2018 262
May 2018 308
June 2018 221
July 2018 197
August 2018 207
September 2018 297
October 2018 317
November 2018 486
December 2018 347
January 2019 501
February 2019 596
March 2019 887
April 2019 1,123
May 2019 1,057
June 2019 859
July 2019 1,045
August 2019 1,010
September 2019 1,290
October 2019 1,415
November 2019 1,238
December 2019 996
January 2020 1,017
February 2020 1,649
March 2020 1,204
April 2020 990
May 2020 931
June 2020 1,247
July 2020 1,128
August 2020 1,021
September 2020 1,536
October 2020 1,454
November 2020 1,534
December 2020 1,488
January 2021 1,263
February 2021 1,232
March 2021 1,723
April 2021 1,685
May 2021 1,343
June 2021 1,477
July 2021 1,119
August 2021 1,469
September 2021 2,203
October 2021 2,429
November 2021 2,176
December 2021 1,900
January 2022 1,631
February 2022 1,755
March 2022 2,089
April 2022 1,825
May 2022 1,452
June 2022 1,045
July 2022 749
August 2022 944
September 2022 1,412
October 2022 1,677
November 2022 1,463
December 2022 1,134
January 2023 1,180
February 2023 1,474
March 2023 1,791
April 2023 1,389
May 2023 1,349
June 2023 927
July 2023 876
August 2023 849
September 2023 1,204
October 2023 1,534
November 2023 1,524
December 2023 1,021
January 2024 1,247
February 2024 1,702
March 2024 1,971
April 2024 1,546
May 2024 1,786
June 2024 811
July 2024 1,020
August 2024 286

Email alerts

Citing articles via, affiliations.

  • Online ISSN 2053-8855
  • Copyright © 2024 Oxford University Press
  • About Oxford Academic
  • Publish journals with us
  • University press partners
  • What we publish
  • New features  
  • Open access
  • Institutional account management
  • Rights and permissions
  • Get help with access
  • Accessibility
  • Advertising
  • Media enquiries
  • Oxford University Press
  • Oxford Languages
  • University of Oxford

Oxford University Press is a department of the University of Oxford. It furthers the University's objective of excellence in research, scholarship, and education by publishing worldwide

  • Copyright © 2024 Oxford University Press
  • Cookie settings
  • Cookie policy
  • Privacy policy
  • Legal notice

This Feature Is Available To Subscribers Only

Sign In or Create an Account

This PDF is available to Subscribers Only

For full access to this pdf, sign in to an existing account, or purchase an annual subscription.

Case Reviews, Study Guides, and Step Exams: Pharmacology

  • Cases and Self-Assessment
  • Anatomy, Histology, and Physiology
  • Emergency Medicine
  • Family Medicine
  • Internal Medicine
  • Microbiology & Biochemistry
  • Neurology/Neuroscience
  • Obstetrics & Gynecology

Pharmacology

Reviews at a glance.

  • LWW Medical Education Exam Review Texts
  • LWW Clerkship/Clinical Rotations Exam Review Texts
  • AccessMedicine Self-Assessment
  • Katzung & Trevor's Pharmacology: Examination & Board Review Review Questions Free AccessMedicine personal account required.

Cover Art

  • << Previous: Pediatrics
  • Next: Psychiatry >>
  • Last Updated: Jul 10, 2024 8:45 PM
  • URL: https://med-fsu.libguides.com/cases

Maguire Medical Library Florida State University College of Medicine 1115 W. Call St., Tallahassee, FL 32306 Call 850-644-3883 (voicemail) or Text 850-724-4987 Questions? Ask us .

We use cookies to enhance our website for you. Proceed if you agree to this policy or learn more about it.

  • Essay Database >
  • Essays Samples >
  • Essay Types >
  • Case Study Example

Pharmacology Case Studies Samples For Students

39 samples of this type

WowEssays.com paper writer service proudly presents to you an open-access directory of Pharmacology Case Studies designed to help struggling students tackle their writing challenges. In a practical sense, each Pharmacology Case Study sample presented here may be a guidebook that walks you through the important phases of the writing process and showcases how to develop an academic work that hits the mark. Besides, if you need more visionary help, these examples could give you a nudge toward an original Pharmacology Case Study topic or encourage a novice approach to a threadbare theme.

In case this is not enough to slake the thirst for effective writing help, you can request personalized assistance in the form of a model Case Study on Pharmacology crafted by an expert from scratch and tailored to your specific directives. Be it a simple 2-page paper or a profound, extended piece, our writers specialized in Pharmacology and related topics will submit it within the stated timeframe. Buy cheap essays or research papers now!

Cardiovascular Disorder Case Studies Example

Affiliate university.

Question 1. Ans . A Question 2 His triglycerides levels are above the normal rate, which should be less than 150mg/dl. Triglycerides are stored type of fats which may pose a risk of stroke, heart attack, diabetes and other terminal diseases that my endanger his health (Kohli and Cannon, 2012). His Pattern B LDL need to be reduced as large percentage of Pattern B LDL increases the risk of coronary diseases.

They regulate the amount of cholesterol level, reduces the amount of triglycerides and lipoprotein in the body (Jafri et al., 2009).

Big Pharma's Marketing Tactics Case Study Sample

Example of chiquita: an excruciating dilemma between life and law case study.

Don't waste your time searching for a sample.

Get your case study done by professional writers!

Just from $10/page

Political Legal And Ethical Dilemmas In The Global Pharmaceutical Industry Case Study Example

Case study on nursing cancer care.

Caner care and the complications of chemotherapy have always created concerns in the nursing profession. Arguments have been that these complications are preventable since evidence based research has discovered strategies to limit burns and other adverse effects of the treatment. The following pages of this document present a case study of a patient suffering from chemotherapy complication. A synthesis of evidence based research interventions that have been recommended as prophylactic measures will be advanced as an appropriate nursing care is designed for this patient.

Free Case Study On Glenmark Pharmaceuticals Company

Executive summary p.3.

Introduction p.4 Generic drugs vs drug discovery businesses pp.4-6 Specific constraints in high-risk R&D, being faced by emerging markets pp.6-7 Glenmark business model pp.7-8 R&D action plan pp.8-9 Conclusion p.9

References pp.10-11

Case study on this work contains brief analysis of salix pharmaceuticals inc current market position, executive summary.

On the basement of the information obtained a new strategy was proposed. Strategy formulation, implementation and contingency plan were outlined. Perspectives of further development of the company were proposed.

Introduction

Example of pharmacy industry and products case study, good case study on merck open for innovation, sample case study on ptsd research paper, free adhd case study case study sample, pain assessment: a top-quality case study for your inspiration.

Part A: Assessment Cause Hank’s pain comes from a two-year-old surgery and radiotherapy for his left lung mesothelioma. Objective Data Hank is alert, oriented and can respond to questions in an appropriate manner. Hank’s skin appears pale with slight perspiration. The VAS assessment kit shows an intensity score of 2/10. Furthermore, Hank demonstrates distress on movement with visible restrictions on trunk movements. Finally, Hank’s chest is clear though with reduced air entry.

Subjective Data

Medication chart assessment case study.

Q. 1 Assess Mrs Daps’ medication chart. Discuss three (3) significant risks involved with Mrs Dap’ home and/or inpatient use of medicines.

Example Of Case Study On Stemi Patient Care

Expert evidence case studies example, differential diagnosis: a top-quality case study for your inspiration.

Objective: Patient appears well-groomed and devoid of nervousness or anxiety. Vital signs are within normal limits (WNL), except for mild fever. Abdomen: Normal bowel sounds, no rebound or guarding, no masses, presence of mild epigastric tenderness on palpation.

Sample Case Study On Diagnosis And Management Of Gastrointestinal Disorders

Good case study on the practice case study, a brief account of the patient/client assessment, hormone replacement therapy case study example, example of case study on food matters official trailer, summary of the movie, spiritual needs assessment and reflection case studies examples.

I interviewed TJ, a 59-year old male of Hispanic descent and a staunch believer of the Buddhist religion. The following is the transcript of the interview and the responses from my respondent and a patient a facility where I had been working at some point.

Do you think that spirituality has a significant or any influence on physical health?

Perfect model case study on drug dilemmas, business ethics, expertly written case study on pulmonary heart disease to follow, case study on sierra capital partners, about the case:, pharmacology case study case studies examples, physical exam:.

You will be expected to write up your response as follows: List of Differential Diagnoses [rank ordered] A. Congestive Heart Failure B. Angina C. Hypercholesterolemia D. Myocardial Infarction

E. Diabetes Mellitus

F. with Grade II diastolic heart failure.

G. Hypertriglyceridemia

J. hypertension.

Leading hypothesis : Congestive Heart Failure Active Alternative hypotheses Must not miss diagnoses [critical rule outs] Congestive Heart Failure Angina Hypertriglyceridemia Hypercholesterolemia Most common diagnoses [reasonable alternatives] GERD Hypertension Diabetes Mellitus Presumptive Diagnosis [assumption] Congestive Heart Failure

Treatment Plan

Free “prescribed drugs with cams” case study example.

This paper addresses a pharmacological management plan for Mr. NX, including consideration of possible contraindications for CAMs, prescriptive and non-prescriptive recommendations for management of acute pain and other ongoing disease processes, followed by evaluation strategies.

Pharmacological Management Plan

Sample case study on acute gastrointestinal bleed and nsaids, patient information, example of life, death, and property rights: the pharmaceutical industry faces aids in africa case study, article summaries, good example of pharmacotherapy for gastrointestinal and hepatobiliary disorders case study, good casey: a major depressive disorder case study example, haiers refrigerator strategy case studies example, good bajaj: case study example, ischaemic stroke case study example, ms. wong's case study sample, free case study on bayer ag: anthrax and cipro, discussion of the case, type 2 diabetes with foot neuropathy case study, description of the health issue, case study on intro to microbiology - pathogenic & non-pathogenic microorganism, example of case study on contingent liabilities according to paragraph 10 of ias standards, report to the parent company in the united kingdom, developed vision and mission statement for johnson and johnson pharmaceutical organization case study.

This paper gives an in depth analysis of the Johnson and Johnson, commonly referred to as J&J. Johnson and Johnson is an international, diversified pharmaceutical and medicinal producer. Besides, it is one of the largest health care related service providers and has drastically reformed people’s perception towards health care. It is indeed a top achiever in the health care industry as well as in general human health and happiness. In fact, its success is attributed to a well defined vision, mission, objectives and core strategies. Its existing vision, mission, objectives and core strategies are discussed as under.

Password recovery email has been sent to [email protected]

Use your new password to log in

You are not register!

By clicking Register, you agree to our Terms of Service and that you have read our Privacy Policy .

Now you can download documents directly to your device!

Check your email! An email with your password has already been sent to you! Now you can download documents directly to your device.

or Use the QR code to Save this Paper to Your Phone

The sample is NOT original!

Short on a deadline?

Don't waste time. Get help with 11% off using code - GETWOWED

No, thanks! I'm fine with missing my deadline

Case-based learning: palliative and end-of-life care in community pharmacy

Community pharmacists encounter patients at all stages in their life; however, patients who require palliative care require dedicated time and special consideration.

Case-based learning: palliative care

JL / Shutterstock.com

The World Health Organization (WHO) defines palliative care as an approach that improves the quality of life of patients and their families facing the problem associated with life-threatening illness. This is achieved through the prevention and relief of suffering by means of early identification and impeccable assessment, and treatment of pain and other problems [1] .

Palliative care is often considered to be only for patients with cancer, but it also incl udes patients with conditions such as organ failure (e.g. heart failure , COPD, renal and hepatic failure), neurological conditions (e.g. multiple sclerosis , Parkinson’s disease and motor neurone disease), dementia, frailty , stroke and HIV/AIDS [2] , [3] , [4] .

The same can be said when considering symptoms (see Table) [5] . Being symptom-free is one of the most important factors for patients when considering end-of-life care [6] . Pharmacists have much to offer, not only in providing medicines required for symptom management, but also supporting patients and their carers to receive the right care at the right time [2] , [7] , [8] .

case study examples pharmacology

Table: an estimate of the prevalence of physical symptoms in palliative patients based on results from 143 studies

Source: J Pain Symptom Manage [5]

Symptom management

How symptoms are treated may change over time and may depend on many factors, including the symptom being treated, the patient’s ability to swallow (owing to disease process causing fatigue and weakness), and level of consciousness. Although most symptoms can be treated with oral preparations, it is likely that the oral route will become less available and a switch to parenteral preparations may be required.

The Gold Standards Framework recommends considering anticipatory prescribing of subcutaneous formulations for pain, nausea and vomiting, agitation/restlessness and excess respiratory secretions (also known as ‘death rattle’) [9] , [10] . The anticipatory prescribing of these medicines is part of advance care planning (i.e. the conversation between the patient’s families and carers about their future wishes and priorities for care) [11] , [12] .

The way symptoms are treated in patients receiving palliative care may change over time; however, several factors may influence this. For example, the Gold Standards Framework recommends considering subcutaneous formulations for:

  • Nausea and vomiting;
  • Agitation/restlessness;
  • Excess respiratory secretions [12] .

Pharmacy schemes across the UK exist to act as stockists for locally selected anticipatory medicines, including alternative anti-emetics, opiates and steroids. See the Local Pharmaceutical Committee website for details (see Useful resources ). Be aware of the local pharmacies enrolled in these schemes, to ensure the pharmacy team can advise patients or carers if needed.

The Gold Standards Framework also suggests five steps for healthcare professionals to consider when advance care planning for patients, which are:

  • Thinking about the future and what is important to the patient;
  • Talking with family and friends about the patient’s choices so they are aware of them and can act as a spokesperson should the patient become unable to do this for themselves;
  • Recording the patient’s wishes;
  • Discussing plans with healthcare teams involved in the patient’s care (this may include conversations about resuscitation);
  • Sharing information with the people who will need to be aware of it [12] .

An advance care plan can include many details, such as the desired extent of active treatment; the patient’s preferred place of care at the end of life; preferences for symptom management (e.g. the level of sedation a patient finds acceptable); people they would like to be present at end of life; spiritual and religious needs; and thoughts around funeral planning.

The Palliative Care Formulary  provides excellent information and guidance on dosing and routes, which are often outside of the product license [10] . The National Institute for Health and Care Excellence (NICE) has also published several documents to support healthcare professionals in their approach to palliative care [13] (see Useful resources ).

Role of the pharmacist

Towards the end of a patient’s life, drug treatments should be as minimised when possible, with pharmacists reviewing and advising on deprescribing [7] , [14] . Pharmacists should also support patients in fulfilling their wishes to die in the place they choose with their symptoms well managed [8] .

The role of pharmacists in palliative care should be much greater than a supply function, although the importance of stocking anticipatory medicines should not be underestimated. Patients and their carers value more than the basic level of service from the pharmacy, and appreciate friendliness from staff who are familiar with them and can provide support with medicines and symptom management [15] .

The complex and often changing medicines regimens for palliative care patients can be challenging for medicines optimisation. Providing clear instructions on regimens and general communication can help (see Box 1 ) [10] . It is important to establish what medicines the patient is taking and when, and support and simplify their regimen if possible. For example, a patient is instructed to take paracetamol 1,000mg four times a day at 08:00, 12:00, 16:00 and 20:00, but wakes during the night with pain. Instead, the patient could be advised to take the paracetamol at equal intervals throughout the 24-hour period, (e.g. every six hours) [10] . However, this decision would need to take into consideration the patients waking hours and preferences.

The following case describes how pharmacists can assist patients nearing end of life with management of pain, nausea and constipation, and anticipatory prescribing.

Box: Considerations to take into account when communicating with palliative care patients and carers

Aims of communication :

  • Information sharing;
  • Reducing uncertainty;
  • Facilitating choices and joint decision making;
  • Creating, developing and maintaining relationships .

Getting started:

  • Make time for the conversation, even if it is only for a few uninterrupted minutes;
  • Ensure privacy;
  • Introduce yourself;
  • Indicate clearly that you have time to listen (e.g. offer to sit down with them);
  • Use open questions to obtain information and allow the patient to talk (e.g. “what do you think this medicine is for?”);
  • Engage using active listening (e.g. nodding and summarising to check information).

Be aware of non-verbal communication, such as:

  • Facial expression;
  • Eye contact;
  • Posture (whether sitting or standing);
  • Pitch and pace of voice;
  • Use of appropriate touch.

Try to recognise barriers to good communication, for example:

  • Language (patient’s own language or the use of jargon);
  • Poor hearing, feeling generally unwell, fatigued, distracted by symptoms, fear and anxiety;
  • Poor transfer of care information;
  • Lack of time and privacy.

case study examples pharmacology

Figure: the three-step analgesic ladder

Source: World Health Organization [16]

This cancer pain management ladder provides a general guide to treatment based on the severity of pain

Palliative care part 1: pain management

Mary*, aged 54 years, comes into the pharmacy and asks to speak to the pharmacist about her prescribed medicines. She was recently diagnosed with metastatic pancreatic cancer, and her GP and palliative care team are working to manage her pain.

Two weeks ago, Mary was started on paracetamol 1,000mg four times per day and morphine sulphate liquid 5mg (2.5mL) every two to four hours as required. She was previously prescribed codeine 30–60mg four times per day, but this did not help to relieve her symptoms. On average, she has been taking six doses of morphine sulphate 5mg in each 24-hour period, and although this helps, it does not completely alleviate her pain.

Today, Mary has been prescribed morphine sulphate (MST; Napp Pharmaceuticals) 15mg twice daily and has been told to continue with her paracetamol and morphine sulphate liquid (for breakthrough pain). Mary explains that her GP said there may be a need to add a medicine for nerve pain. However, Mary goes on to say that she is worried that she is taking “too much” medicine and she is reluctant to take all of these medicines, especially if another may be added for nerve pain.

Actions and recommendations

The pharmacist should consider the following during the consultation:

  • Communication — open questions should be used to determine how much has been explained to Mary about pain management. It may be that she has been given an explanation, but it is often worth covering these again;
  • World Health Organization pain ladder — explain the use of the ladder (see Figure), including the need to use regular paracetamol;
  • Analgesia — patients are often concerned by using more than one opiate. Patients may need to be encouraged to use their breakthrough analgesia when required, explaining how the dose is determined by the background analgesia may help [17] ;
  • Keeping a diary — Mary should be encouraged to keep a pain diary. This can be a benefit as keeping a record of breakthrough doses may help when it comes to dose titration;
  • Addressing patient concerns — explain to Mary that neuropathic agents may be used as an adjuvant if opiates fail to control the pain [18] ;
  • Resources — recommend available online resources to Mary through the British Pain Society and the National Institute for Health and Care Excellence (see Useful resources) [17] , [19] .

Explain to Mary that as her opiate dose is increased, breakthrough doses will also increase. These should be one-sixth of the total daily opiate dose. For example, a dose of MST 30mg twice daily (60mg/24 hours), will require morphine sulphate (liquid or tablet) 10mg every 2–4 hours as required for breakthrough pain [10] , [14] . 

When to refer

Mary should be referred back to her GP or the palliative care team if she requires frequent prescriptions for breakthrough medicines, which would indicate that her pain is not adequately managed, or she is experiencing unacceptable side effects (e.g. nausea or severe itch).

Palliative care part 2: nausea and constipation

Mary returns to the pharmacy ten days after starting on MST and her background dose is currently 30mg twice daily. She says that her pain is controlled, but she has been experienced nausea since the MST dose was increased two days ago. She also explains that she has not had “a proper bowel movement” for five days. Mary says she feels generally uncomfortable and has a feeling of fullness in her abdomen.

As a result, Mary is considering stopping the MST capsules. When questioned on her diet, Mary says she understands that dietary intake influences her bowels; however, she explains that her appetite is reduced owing to the abdominal discomfort. Mary would like to know if there is anything she can purchase that will help.

Constipation is the most commonly reported side effect of opiate treatment (up to 80% of patients) and can result in reduced quality of life and discontinuation of the offending medicine, which could result in a resurgence of pain [10] , [20] . When speaking with Mary about management, consider the following:

  • Ascertain bowel pattern — the Bristol Stool chart can be used to support the consultation, but it is important to determine what is ‘normal’ for the patient and any other symptoms (e.g. colic) [21] . Mary confirms that her bowels feel sluggish and she is passing small, hard stools, which requires a lot of effort.;
  • Maintaining a healthy diet — Mary’s dietary intake is reduced, but with resolution of constipation, this may improve. Encourage a healthy diet that includes whole grains, fruit and vegetables, and a good fluid intake [22] ;
  • Laxatives — regular administration of laxatives to avoid constipation is advised [14] . The National Institute for Health and Care Excellence recommends the use of: stimulant laxatives (e.g. Senna [7.5–15mg at night, increasing to 30mg maximum in 24 hours] and sodium picosulphate [5–10mg at night]) and osmotic laxatives (e.g. lactulose [15–45mL daily in three divided doses] and macrogol compound [1–3 sachets daily in divided doses]) [22] . Docusate (up to 500mg daily in divided doses) is an alternative as both a softener and weak stimulant [10] . If tolerance to constipation does not occur, treatment with laxatives is advised for the duration of opiate use [20] . If titration of laxative is required then this occurs every 1–2 days according to response before switching to an alternative [10] ;
  • Nausea — this common side effect occurs on initiation and dose increase, but is normally transient, lasting only a few days [15] , [23] , [24] . If patients are aware of this, they may feel more prepared and willing to accept it for a short period of time making the use of an antiemetic unnecessary [20] ;
  • Non-pharmacological measures — pharmacy teams can recommend removal of certain stimuli (e.g. sight and smell of nausea-provoking foods) that lead to nausea and massage to help manage constipation [25] , [26] .

It is important to explain to Mary that opioid analgesics are a common cause of constipation, alongside other contributing factors, such as poor fluid, and dietary intake [27] .

Compliance with laxatives may be limited by patient factors, such as palatability, volume required and undesirable effects (e.g. flatulence and colic). Mary’s preference should be considered, especially as there is limited evidence for the use of laxatives, and management is generally dictated by best practice and expert opinion [14] .

The underlying aetiology of nausea and/or vomiting should be considered; gastric dysmotility and stimulation of the chemoreceptor trigger zone are most common with opiates, and can be managed with prokinetics (e.g. domperidone, metoclopramide) or a D2 receptor antagonist (e.g. haloperidol) [10] , [18] , [19] .

When the maximum licensed dose of laxatives is reached without adequate result, then there is a need to refer the patient.

If an antiemetic is considered necessary (e.g. owing to persistent or intolerable nausea ), the patient requires referral for thorough assessment [25] .

Palliative care part 3: Anticipatory prescribing

Mary’s husband, John*, brings a prescription for anticipatory medicines into the pharmacy. He is visibly upset and asks to speak to the pharmacist. He explains that the GP surgery called and asked for this prescription to be collected following a home visit from Mary’s community nurse.

During the visit, the nurse asked difficult questions about where Mary would like to be cared for when she becomes less well, and whether she would want to be resuscitated. The nurse had also suggested that Mary can stop her simvastatin, which she has been taking for the past 17 years.

John’s concerns

Although John is concerned because he knows Mary is unwell, he thinks these conversations suggest that Mary is nearing the end of her life.

When speaking with John, be empathetic while providing information. For example, explain:

  • That anticipatory prescribing is part of advance care planning and does not signify that a patient is imminently dying. Medicines are prescribed in anticipation of symptoms, and should be put in place well in advance to enable rapid symptom relief [13] , [28] ;
  • The possible use for each of the medicines (pain, nausea and vomiting, excess respiratory secretions, agitation and restlessness) [29] . It is unlikely that all would be needed, and possibly none at all. These medicines can be used for a more acute symptom, such as nausea or pain owing to infection, rather than only for use in the last few days of life;
  • Stopping unnecessary medicines (i.e. those for long-term risk, such as statins for cardiovascular disease) should be considered to reduce tablet burden and potential side effects [7] ;
  • These conversation can be distressing, but it provides an opportunity to consider serious issues during a time that is less critical and that some patients and carers may find comfort in the planning [30] .

After speaking with and explaining these points to John he appears to be more content and less worried. It is important to explain to him that the pharmacy team is available to speak to him or his wife if they have any further concerns.

*Case study is fictional

Useful resources

  • Local Pharmaceutical Committee: lpc-online.org.uk/

[1] World Health Organization. WHO definition of palliative care. 2019. Available at: https://www.who.int/cancer/palliative/definition/en/ (accessed October 2019)

[2] World Health Organization. Palliative care. 2018. Available at: https://www.who.int/news-room/fact-sheets/detail/palliative-care (accessed October 2019)

[3] The Gold Standards Framework. PIG – Proactive identification guidance registration form. 2016. Available at: http://www.goldstandardsframework.org.uk/PIG (accessed October 2019)

[4] Marie Curie. Triggers for palliative care. 2015. Available at: https://www.mariecurie.org.uk/globalassets/media/documents/policy/policy-publications/june-2015/triggers-for-palliative-care-full-report.pdf (accessed October 2019)

[5] Moens K, Higginson IJ & Harding R. Are there differences in the prevalence of palliative care-related problems in people living with advanced cancer and eight non-cancer conditions? A systematic review. J Pain Symptom Manag 2014;48(4):660–677. doi: 10.1016/j.jpainsymman.2013.11.009

[6] Kobewka D, Ronksley P, McIssac D et al. Prevalence of symptoms at the end of life in an acute care hospital: a retrospective cohort study. CMAJ Open 2017;5(1):E222–E228. doi: 10.9778/cmajo.20160123

[7] Walker KA, Scarpaci L & McPherson ML. Fifty reasons to love your palliative care pharmacist. American J Hosp Palliat Care 2010;27(8):511–513. doi: 10.1177/1049909110371096

[8] Macmillan Cancer Support. The Final Injustice: variation in end of life care in England. 2017. Available at: https://www.macmillan.org.uk/_images/MAC16904-end-of-life-policy-report_tcm9-321025.pdf (accessed October 2019)

[9] The Gold Standards Framework. Examples of Good Practice Resource Guide. Just in Case Boxes. 2006. Available at: https://www.goldstandardsframework.org.uk/cd-content/uploads/files/Library%2C%20Tools%20%26%20resources/ExamplesOfGoodPracticeResourceGuideJustInCaseBoxes.pdf (accessed October 2019)

[10] Twycross R, Wilcock A & Howard P. Palliative Care Formulary 6th Ed (PCF6). 2016. Available at: https://about.medicinescomplete.com/publication/palliative-care-formulary (accessed October 2019)

[11] National Palliative and End of Life Care Partnership. Ambitions for palliative and end of life care. 2015. Available at: http://endoflifecareambitions.org.uk/wp-content/uploads/2015/09/Ambitions-for-Palliative-and-End-of-Life-Care.pdf (accessed October 2019)

[12] Gold Standard Framework. Advance Care Planning. 2018. Available at: https://www.goldstandardsframework.org.uk/advance-care-planning (accessed October 2019)

[13] National Institute for Health and Care Excellence (NICE). Care of dying adults in the last days of life. NICE guideline [NG31]. 2015. Available at: https://www.nice.org.uk/guidance/ng31 (accessed October 2019)

[14] Royal Pharmaceutical Society. British National Formulary 76 . London: Pharmaceutical Press; 2001 

[15] Edwards Z, Blenkinsopp A, Ziegler L & Bennett MI. How do patients with cancer pain view community pharmacy services? An interview study. Health Soc Care Community 2018;26(4):507–518. doi: 10.1111/hsc.12549

[16] World Health Organization. WHO’s cancer pain ladder for adults. 2019. Available at: https://www.who.int/cancer/palliative/painladder/en/ (accessed October 2019)

[17] National Institute for Health and Care Excellence (NICE). Palliative care for adults: strong opioids for pain relief. NICE guideline [CG140] 2016. Available at: https://www.nice.org.uk/guidance/cg140/ifp/chapter/Managing-side-effects (accessed October 2019)

[18] The British Pain Society. Cancer pain management. 2010. Available at: https://www.britishpainsociety.org/static/uploads/resources/files/book_cancer_pain.pdf (accessed October 2019) 

[19] The British Pain Society. Patient publications. Available at: https://www.britishpainsociety.org/british-pain-society-publications/patient-publications/ (accessed October 2019) 

[20] Rogers E, Mehta S, Shengelia R, & Carrington Reid M. Four strategies for managing opioid-induced side effects in older adults. Clin Geriatr 2013;21(4). PMID: 25949094

[21] National Institute for Health and Care Excellence (NICE). Clinical knowledge summary. Constipation in adults. 2019. Available at: https://cks.nice.org.uk/constipation#!scenarioRecommendation:1 (accessed October 2019)

[22] National Institute for Health and Care Excellence (NICE). Constipation in children and young people: diagnosis and management. Clinical guideline [CG99]. 2017. Available at: https://www.nice.org.uk/guidance/cg99/resources/cg99-constipation-in-children-and-young-people-bristol-stool-chart-2 (accessed October 2019)

[23] NHS. Morphine. 2018. Available at: https://www.nhs.uk/medicines/morphine/# (accessed October 2019) 

[24] Cherny N, Ripamonti C, Pereira J et al. Strategies to manage the adverse effects of oral morphine: An evidence-based report. J Clin Oncol 2001;19(9):2542–2554. doi: 10.1200/JCO.2001.19.9.2542

[25] Chand S. Nausea and vomiting in palliative care. Pharm J 2014;292(7799)240. doi: 10.1211/PJ.2014.11135047

[26] National Institute for Health and Care Excellence (NICE). Clinical Knowledge Summary. Palliative care — nausea and vomiting. 2016. Available at: https://cks.nice.org.uk/palliative-care-nausea-and-vomiting (accessed October 2019)

[27] Watson M, Lucas C, Hoy A & Back I. Oxford Handbook of Palliative Care. Oxford University Press; 2005. 

[28] National Institute for Health and Care Excellence (NICE). Care of dying adults in the last days of life. Quality standard [QS144]. 2017. Available at: https://www.nice.org.uk/guidance/qs144 (accessed October 2019)

[29] National Institute for Health and Care Excellence (NICE). Medicines guidance: prescribing in palliative care. Available at: https://bnf.nice.org.uk/guidance/prescribing-in-palliative-care.html (accessed October 2019)

[30] National Institute for Health and Care Excellence (NICE). End of life care for adults. 2017. Quality Standard [QS13]. Available at: https://www.nice.org.uk/guidance/qs13 (accessed October 2019)

You might also be interested in…

Illustration of an hourglass with sand in the shape of a pharmacy that is running out.

PJ view: Restore pharmacy opening hours to improve end-of-life care

Illustration of a hand opening a nursing home like a box, in which is a supply of pharmaceuticals

Controlled drug stock licensing is working against high-quality palliative care for care home patients

Home carer supporting old woman to stand up from the armchair at care home

How palliative care pharmacists in community practice are improving access to medicines

IMAGES

  1. Case 14

    case study examples pharmacology

  2. Pharmacology Case Study

    case study examples pharmacology

  3. SOLUTION: Pharmacology case study 2 docx 1

    case study examples pharmacology

  4. ⇉Pharmacology Lab Report Essay Example

    case study examples pharmacology

  5. Pharmacology Case Studies for Nurse Prescribers PDF Free Download

    case study examples pharmacology

  6. Pharmacology Case Study- student

    case study examples pharmacology

COMMENTS

  1. 50+ Pharmacy Case Studies for Students!

    As a qualifying pharmacist, case studies bring together the threads of study over the past four years. This includes your study of subjects such as: Pharmacology. Pharmaceutical chemistry. Pharmaceutics. Clinical pharmacy practice. In practice, pharmacists are expected to draw on this knowledge and clinically apply it where necessary.

  2. PDF Pharmacology/Toxicology Case Studies

    Pharmacology/Toxicology Case Studies + 132 Pages + 60 Case Studies + Each case includes multiple questions followed by a detailed explanation

  3. PDF Case Studies

    Case Studies Case Study 1: A Pain in the Knee Principles of Pharmacology 2002 Case 1:1 Case Author: John D. Yee, M.D., HMS '91 Case Editors: Kelly Fuksa, HMS '04 and Prasanna Jagannathan, HMS '04 Case Study 1: A Pain in the Knee David Robertson is a 15 year old high school student from the Bronx, New York City,

  4. Clinical Case Studies with Answers For Pharm.D

    c) Endocrine System Case Studies with answers: Diabetes: Case Study 17 and Case Study - 18. Hypothyroidism : Case Study -19 and Case Study -20. Other Thyroid Disorders : Case Study -21 and Case Study -22. Oral Contraceptive Use: Case Study- 23 and Case Study- 24. Hormone Replacement Therapy: Case Study- 25 and Case Study- 26.

  5. (PDF) 100 Cases in Clinical Pharmacology Therapeutics and Prescribing

    100 Cases in Clinical Pharmacology Therapeutics and Prescribing 1st edition. ... We describe a case series of pulmonary embolism associated with severe pulmonary tuberculosis. In our first case pulmonary embolism (PE) occurred within 6months of pulmonary tuberculosis diagnosis.Patient took ATT for 6 months and after being diagnosed as a case of ...

  6. Cases

    007 Case 007-Dyslipidemia Case # 1. 008 Case 008-Dyslipidemia Case # 2. 009 Case 009-Stable Ischemic Heart Disease. 010 Case 010-Acute Coronary Syndromes. 011 Case 011-Cardiovascular Respiratory Decompensation. 012 Case 012-Chronic Heart Failure Case # 1. 013 Case 013-Chronic Heart Failure Case # 2. 014 Case 014-Venous Thromboembolism-Pulmonary ...

  7. Cases

    Harrison's Pathophysiology Animations. Cases. Top 125 Drug Card Case Quiz. Katzung Pharmacology Cases. Pharmacotherapy Casebook and Care Plans. Standardized Patient Cases. Pharmacogenomics Cases. Transitions of Care Cases. Pharmacy Practice and Tort Law.

  8. Case Studies

    Case Studies. Case one. GB is a 42-year-old man who comes to the clinic for a scheduled visit. GB is overweight and admits to exercising very infrequently. He has a medical history of hypertension and hyperlipidemia for which he takes hydrochlorothiazide and rosuvastatin, respectively. Recent laboratory results show GB has a fasting blood ...

  9. PDF Example Case Studies

    Example Case Studies Activation Date: September 2014 Revision Date: Dec 2020 8 . Clinical Pharmacy Guide: Cancer Drug Treatment Assessment and Review 5 th Edition . Subsequent Doses . Cycle 2: First occurrence of low counts, no febrile neutropenia . D.N. returns on June 17 for her second cycle of BRAJFEC. ...

  10. Tuberculosis

    Read chapter 34 of Infectious Diseases: A Case Study Approach online now, exclusively on AccessPharmacy. AccessPharmacy is a subscription-based resource from McGraw Hill that features trusted pharmacy content from the best minds in the field.

  11. Clinical Pharmacology: Current Topics and Case Studies

    In addition, in-depth insights into clinical drug research and trial methodology are presented on the basis of concrete case studies. This updated book makes a valuable contribution to the field of Clinical Pharmacology and serves as a must-have guide for professors, researchers and advanced students from academia and pharmaceutical industry.

  12. Case Study: A Patient With Type 2 Diabetes Working With an Advanced

    The following case study illustrates the pharmacotherapeutic challenges of diabetes with other comorbidities, which can lead to potential drug-drug and drug-disease interactions. ... For example, near-normal glycemic control and normalization of blood pressure (endpoints) would be necessary to significantly reduce the risk of end-stage renal ...

  13. Pharmacology Case Studies for Nurse Prescribers

    This new edition of the popular Pharmacology case studies for nurse prescribers has been thoroughly revised in the light of the latest research and guidance from NICE, the British National Formulary (BNF), the Royal Pharmaceutical Society, the Nursing and Midwifery Council and the Royal College of Nursing. While the first edition was aimed at students undertaking the non-medical prescribing ...

  14. Pharyngitis

    Read chapter 4 of Infectious Diseases: A Case Study Approach online now, exclusively on AccessPharmacy. AccessPharmacy is a subscription-based resource from McGraw Hill that features trusted pharmacy content from the best minds in the field.

  15. Case 24-2020: A 44-Year-Old Woman with Chest Pain, Dyspnea, and Shock

    A 44-year-old woman presented with cough, dyspnea, and chest pain. On examination, she had tachycardia and hypotension. Evaluation revealed SARS-CoV-2 RNA in a nasopharyngeal swab, as well as eleva...

  16. Pharmacology Case Study Workbook

    Pharmacology Case Study Workbook. Kathy Putman, Kathy Latch Putman. Jones & Bartlett Learning, Oct 25, 2010 - Medical - 452 pages. Written by a distinguished nursing professional with over 25 years of experience, Pharmacology Case Study Workbook provides nursing students with an opportunity to apply pharmacology concepts to real-world situations.

  17. Pharmacy Case Studies for Pharmacists & Medical Sciences Students

    This section is only for Bangladeshi Pharmacy/Medical Students & Professionals ! Cardiovascular case studies by Narinder Bhalla. Case study level 1 - Angina. Case study level 2 - Hypertension. Case study level 3 - Atrial fibrillation. Case study level Ma - Heart failure. Case study level Mb - Myocardial infarction.

  18. How to Answer Pharmacy Case Studies

    When faced with a case study, approach it with the following points in mind: Review the case study more than once. Never rush into a case study, picking and choosing facts you understand. Instead, approach the case study with an open mind. Be prepared to accept that you might not know the answer straight-off. The study may require some careful ...

  19. Educational Case: A 57-year-old man with chest pain

    A 57 year-old male lorry driver, presented to his local emergency department with a 20-minute episode of diaphoresis and chest pain. The chest pain was central, radiating to the left arm and crushing in nature. The pain settled promptly following 300 mg aspirin orally and 800 mcg glyceryl trinitrate (GTN) spray sublingually administered by ...

  20. Case Reviews, Study Guides, and Step Exams: Pharmacology

    Case Files Pharmacology by Eugene C. Toy; David S. Loose; Shelley A. Tischkau; Anush S. Pillai Publication Date: 2013 To access quizzes and additional content, create a free, personal MyAccess account.

  21. Pharmacology Case Study Examples That Really Inspire

    39 samples of this type. WowEssays.com paper writer service proudly presents to you an open-access directory of Pharmacology Case Studies designed to help struggling students tackle their writing challenges. In a practical sense, each Pharmacology Case Study sample presented here may be a guidebook that walks you through the important phases of ...

  22. Case Study: Treating Hypertension in Patients With Diabetes

    Studies that have compared antihypertensive treatment in patients with diabetes versus placebo have shown reduced cardiovascular events. The United Kingdom Prospective Diabetes Study (UKPDS), which followed patients with diabetes for an average of 8.5 years, found that patients with tight BP control (< 150/< 85 mmHg) versus less tight control (< 180/< 105 mmHg) had lower rates of myocardial ...

  23. Case-based learning: palliative and end-of-life care in community

    For example, a patient is instructed to take paracetamol 1,000mg four times a day at 08:00, 12:00, 16:00 and 20:00, but wakes during the night with pain. Instead, the patient could be advised to take the paracetamol at equal intervals throughout the 24-hour period, (e.g. every six hours) [10]